Download as pdf or txt
Download as pdf or txt
You are on page 1of 165

106/3A & 106/2, Postal Nagar, Bodhupatty (Post), Namakkal – 637 003.

Ph: 04286 – 234410, 234411, 234413, 234414 Cell : 95009 79461, 95009 79462
E- mail : gpccnamakkal@gmail.com Website : www.gpccnamakkal.com
Our Branches :
GPCC @ SSVM Girls Matric Hr. Sec. School Campus, Karur Green Park Academy, Chennai
GPCC @ Green Garden Matric Hr. Sec. School Campus, Erode Green Park Career Academy, Kovai

.SPP – 1 to 5.
Important Instructions :
1) This test is 3 hours 20 min duration and Maximum mark is 720.
2) This test booklet contains 200 questions. There are four parts in the question paper, consisting
Physics, Chemistry, Botany and Zoology. Each part consists of 50 questions.
3) Each part consists of two sections, Section - A and Section - B.
4) Section - A of each subject consists of 35 questions and Section - B consists of 15 questions.
Candidates have to answer for all the 35 questions from Section - A and only 10 questions out of 15
questions from Section - B.
5) Candidates are allowed to answer maximum of 180 questions only from all four parts.
6) There will be only one correct choice in the given four choices for each question. For each question
4 marks will be awarded for correct choice, 1 mark will be deduced for incorrect choice and zero mark
will be awarded for unattempted question.
7) Rough work is to be done on the space provided for this purpose in the Test Booklet only.
8) Use blue/black ballpoint pen only to darken the appropriate circle.
9) Dark only one circle for each question. Mark should be dark and should completely fill the circle.
10) Any textual, printed or written material, mobile phones, calculator etc. is not allowed for the students
appearing for the test.
11) On completion of the test, the candidate must handover the Answer Sheet to the invigilator before leaving
the Room / Hall. The candidates are allowed to take away this Test Booklet with them.
12) The candidates should ensure that the Answer Sheet is not folded. Do not make any stray marks on the
Answer Sheet. Do not write your Roll No. anywhere else except in the specified space in the Test
Booklet / Answer Sheet.
13) Use of white fluid for correction is NOT permissible on the Answer Sheet.
14) No candidate, without special permission of the Superintendent or Invigilator, would leave his/her seat.

Stay Cool … Think Twice … Analyse Well … Confirm the Answer …


Success is Assured !
1

SPP – 1
Max. Marks: 720 Duration : 3 Hrs. 20 Min.
PHYSICS
SECTION – A (35 questions)
(Answer all the questions)

1. An electric field of 1000 V/m is applied to an 4. In a young’s double slit experiment, 16 fringes
electric dipole at an angle of 45°. The Value of are observed in a certain segment of the screen
electric dipole moment is 10-29 Cm. What is when light of wave length 700 nm is used. If
the potential energy of the electric dipole the wavelength of light is changed to 400 nm
the number of fringes observed in the same
1) -20×10-8J 2) -7 ×10-27J
segment of the screen would be
3) -10 × 10-29J 4) -9 ×10-29J
1) 24 2) 30
2. A potentiometer wire PQ of 1 m length is 3) 18 4) 28
connected to a standard cell E1. Another cell
5. The aperture diameter of telescope is 5m. The
E2 of emf 1.02 V is connected with a separation between the moon and earth is
resistance “r” and switch ‘s’ (as shown in 4 x 105 km. With light of wavelength of 5500
figure). With switch S open the null position is Å the minimum separation between objects on
obtained at a distance of 49 cm from Q. The the surface of moon so that they are just
potential gradient in the potentiometer wire resolved is close to
is:- 1) 60 m 2) 10 m
3) 200m 4) 600 m
6. A galvanometer of resistance G is converted
into a volt meter of range 0- 1V by connecting
a resistance R1 in series with it. The
additional resistance that should be connected
in series with R1 to increase the range of the
voltmeter to 0-2V will be
1) 0.02 V/cm 2) 0.01 V/cm 1) G 2) R1
3) 0.03 V/cm 4) 0.04 V/cm 3) R1+ G 4) R1G
3. The magnetic field of a plane electromagnetic 7. The time period of revolution of electron in its
wave is B =3×10-8 sin (200π(y + ct) î T where ground state orbit in a hydrogen atom is
c =3×108 ms-1 is speed of light. 1.6 x 10-16 s. The frequency of revolution of
the electron in its first excited state is (in s-1)
The corresponding electric field is
1) 1.6 ×1014 2) 7.8 ×1014
1) E = 9sin[200π(y+ct)]kɵ V/m
3) 6.2 × 1015 4) 5.6 × 1012
2) E =10−6 sin[200π(y+ct)]kɵ V/m 8. An electron, a doubly ionized helium (He++)
and a proton are having the same kinetic
3) E = 3 ×10−8 sin[200π(y+ct)]kɵ V/m energy. The relation between their respective
4) E = − 9sin[200π(y+ct)]kɵ V/m de-Broglie wavelengths λe, λHe++ and λp is
1) λe > λHe++ > λp 2) λe < λHe++ = λp
3) λe > λp >λHe++ 4) λe < λp < λHe++
2
9. In a photoelectric effect experiment the graph 14. A double convex lens has power P and same
of stopping potential V versus reciprocal of radii of curvature R of both surfaces. The
wave length obtained is shown in the figure. radius of curvature of a surface of a plano –
As the intensity of incident radiation is convex lens made of the same material with
increased power 1.5 P is
R
1) 2 R 2)
2
3R R
3) 4)
2 3
1) Straight line shifts to right 15. A 50 kg girl wearing high heel shoes balances
2) Slope of the straight line get – more steep on a single heel. The heel is circular with a
3) Straight line shifts to left diameter 1.0cm. What is pressure exerted on
4) Graph does not change the horizontal floor?
10. With increasing biasing voltage of a 1) 6.24 × 106 Pa 2) 6.24 × 104 Pa
photodiode, the photo current magnitude
3) 6.24 × 108 Pa 4) 6.24 × 1010 Pa
1) remains constant
16. An electric heater supplies heat to a system at
2) increases initially and after attaining certain a rate of 100W. If the system performs work
valve, it decrease
at a rate of 75 J/s. At what rate is the internal
3) increases linearly energy increasing
4) increases initially and saturate finally 1) 175 J/ sec 2) 25 W
11. Which of the following gives a reversible
3) 50 W 4) 75 J/s
operation.
17. The stress-strain graph for materials A and B
1) 2) are shown in fig (a) and fig (b). The graphs are
drawn to the same scale

3) 4)

12. In the figure, potential difference between A


and B is
10 k Which of the two is the stronger
A
1) Material A 2) Material B
10 k 10 k 3) Both A & B 4) None
30 V
18. The power radiated by a black body is P and it
B radiates maximum energy at wavelength λ0. If
1) 10 V 2) 5 V the temperature of the black body is now
3) 15 V 4) zero changed so that it radiates maximum energy of
An AC circuit has R = 100 Ω, C = 2µF and
λ0
13. wavelength , the power radiated by it
L = 80 mH connected in series. The quality 4
factor of the circuit is become nP. The valve of ‘n’ is _____
1) 2 2) 0.5 1) 4 2) 16
3) 20 4) 400 3) 64 4) 256
3
19. A molecule in a gas container hits a horizontal
wall with speed 200 ms-1 and angle 30° with
the normal and rebounds with the same speed
then the collision is
v v
1) elastic 1) 2)
2 4
2) inelastic
v v
3) perfectly inelastic 3) 4)
8 16
4) momentum is not conserved
23. If speed V, area A and force F are choosen as
20. The bob of a pendulum is released from
fundamental units, then the dimension of
horizontal position A as shown in the figure.
young’s modulus will be
If the length of the pendulum is 1.5 m, what is
1) FA2 V-1 2) FA2 V-3
the speed with which the bob arrives at the
lowest point B, given that it dissipates 5 % of 3) FA2V-2 4) FA-1V0
its initial energy against air resistance 24. A clock has a continuously moving second’s
hand of 0.1 m length. The average
acceleration of the tip of the hand is of the
order of (in units of ms-2)
1) 10-3 2) 10-4
3) 10-2 4) 10-1
1) 2.3 m/s 2) 5.3 m/s 25. A driver in a car approaching a vertical wall
3) 9.3 m/s 4) 14.3 m/s notices that the frequency of his car horn, has
21. A particle moves in one dimension from rest changed from 440 Hz to 480 Hz, when it gets
under the influence of a force that varies with reflected from the wall. If the speed of sound
the distance travelled by the particle as shown in air is 345 m/s, then the speed of the car is.
in the figure. The kinetic energy of the particle 1) 54 m/s 2) 36 m/s
after it has travelled 3 m is 3) 15 m/s 4) 24 m/s
1
26. A carnot engine having an efficiency of is
10
being used as a refrigerator. If the work done
on the work done on the refrigerator is 10 J,
the amount of heat absorbed from the reservoir
at lower temperature is:
1) 4 J 2) 2.5 J 1) 99 J 2) 100 J
3) 6.5 J 4) 5 J 3) 1 J 4) 90 J
22. Blocks of masses m, 2m, 4m and 8 m are 27. An ideal fluid flows (laminar flow) through a
arranged in a line on a frictionless floor. pipe of non uniform diameter. The maximum
Another block of mass m moving with speed and minimum diameters of the pipes are 6.4
is along the same line (see figure) collides cm and 4.8 cm respectively. The ratio of the
with mass m in perfectly in elastic manner. minimum and maximum velocities of fluid in
All the subsequent collisions are also perfectly this pipe is
inelastic. Velocity of the last block of mass
8 m after collision is.
4
9 3 32. If λ1 and λ2 are wavelengths of the first
1) 2)
16 2 members of the lyman and paschen series
3 81 respectively, then λ1:λ2 is
3) 4)
4 256 1) 1 : 3 2) 1 : 30
28. The electromagnetic radiation in the ascending 3) 7 : 50 4) 7 : 108
order of wavelength are 33. Three identical particles each of mass 1 kg are
1) IR rays, Radio waves, X-rays placed with their centres on a straight line. Their
2) Radio waves, IR rays, X-rays centres are marked A, B and C respectively. The
distance of centre of mass of the system from A is
3) X-rays, Radio waves, IR rays
4) X-rays, IR rays, Radio waves
AB + AC + BC AB + AC
29. A ray of light is incident 60° on a prism of 1) 2)
refracting angle 30°. The emergent ray is at 3 3
an angle 30° with incident ray. The value of AB + AC AB + BC
3) 4)
refractive index of the prism is 2 3
34. In the network shown in circuit, potential at A
3 3
1) 2) and B are equal at an instant then the value of
4 2
di
3) 3 4) 2 3 is
dt
30. Two equal and opposite charges of masses m1
and m2 are accelerated in an uniform electric
field through the same distance. What is the
ratio of their accelerations if the ratio of their 1) 400 A/s 2) 300 A/s
m 3) 800 A/s 4) 200 A/s
masses is 1 = 0.5?
m2
35. The given plot represent the I – V
a1 a1 characteristics for a semiconductor device.
1) =2 2) = 0.5
a2 a2 Which of the following statements is correct?
a1 a1
3) =3 4) =1
a2 a2
31. The de-Broglie wavelength of an electron
moving with a velocity of 1.5 x 108 m/s is
equal to that of a photon. The ratio of kinetic
energy of the electron to that of the photon (C 1) It is I – V characteristics of solar cell
= 3 × 108 m/s) 2) It is I – V characteristics of photodiode
1) 2 2) 4 connected in forward bias
1 1 3) It is I – V characteristics of LED in forward
3) 4)
2 4 bias
4) It is I – V characteristics of photodiode in
reverse bias
5
SECTION – B (15 questions)
Section – B consists of 15 questions. Candidates are allowed to answer any 10 questions only out of 15 questions
given. In case if candidates attempts more than 10 questions, first 10 attempted questions only considered for
marking.
36. Concentric metallic hollow spheres of radii R 1) 1 A/m 2) 4 A/m
and 4R hold charges Q1 and Q2 respectively. 3) 2.25 A/m 4) 0.75 A/m
Given that surface charge densities of the 40. A wire A1 bent in the shape of an arc of a
concentric spheres are equal, the potential
circle, carrying a current of 2A and having
difference V(R) - V(4R) is
radius 2cm and another wire B also bent in the
3Q1 3Q 2
1) 2) shape of arc of a circle, carrying a current of
16πE 0 R 4πE 0 R 3A and having radius of 4 cm, are placed as
Q2 3Q1 shown in the figure. The ratio of the magnetic
3) 4)
µπE 0 R 4πE 0 R fields due to the wires A and B at the common
37. In the circuit shown in the figure, the total centre “O” is
change is 750µC and voltage across capacitor
C2 is 20V. Then the charge on capacitor C2 is
O
A B
90o
60o

1) 4 : 6 2) 6 : 4
1) 450µC 2) 590µC 3) 2 : 5 4) 6 : 5
3) 160µC 4) 650µC 41. A concave mirror for face viewing has focal
38. In a meter bridge experiment ‘S’ is a standard length of 0.4 m. The distance at which you
resistance. R is a resistance of wire. It is found hold the mirror from your face in order to see
that balancing length is l = 25 cm. If R is your image upright with a magnification of
replaced by a wire of half length and half 5 is _____
diameter that of R of same material, then the 1) 0.24 m 2) 1.60 m
balancing distance will now is ______. 3) 0.32 m 4) 0.16 m
42. A hoop of radius 2 m weighs 100 kg. It rolls
along a horizontal floor so that its centre of
mass has a speed of 20 cm/sec. How much
work has to be done to stop it?
1) 2 J 2) 4 J
3) 8 J 4) zero
1) 40 cm 2) 20 cm
43. Four point masses, each of mass m, are fixed
3) 25 cm 4) 60 cm
at the corners of a square of side l. The square
39. A paramagnetic sample shows a net
is rotating with angular frequency ω, about an
magnetization of 6 A/m when it is placed in an
external magnetic field of 0.4T at a axis passing through one of the corners of the
temperature of 4K. When the sample is placed square and parallel to its diagonal, as shown in
in an external magnetic field of 0.37 T at a the figure. The angular momentum of the
temperature of 24K then the magnetization square about this axis is
will be
6

3) 4)

47. A block of mass 5 kg pushed by a force


1) ml2 W 2) 4 ml 2W F = 20 N making an angle of 30° with the
3) 3 ml2W 4) 2 ml2W horizontal as shown in the figure. The
44. In a resonance tube experiment when the tube coefficient of friction between the block and
is filled with water upto a height of 17 cm floor is µ = 0.2 The acceleration of the block is:
from bottom it resonates with a given tuning
fork. When the water level is raised the next 30o
resonance with the same tuning fork occurs at
a height of 24.5 cm. If the velocity of sound F = 20 N
in air is 330 m/s, the tuning fork frequency is 2
1) 1.06 m/s 2) 2.06 m/s2
1) 2200 Hz 2) 550 Hz
3) 3.0 m/s2 4) 4.06 m/s2
3) 1100 Hz 4) 3300 Hz
48. The acceleration due to gravity on the earth’s
45. The displacement time graph of a particle surface at the pole is g and angular velocity of
executing SHM is given in figure( sketch is earth about the axis passing through the pole is
schematic and not to scale). Which of the W. An object is weighted at the equator and at a
following statement is wrong for this motion height h above the poles by using a spring
balance. If the weights are found to be same then
h is ( h<<R, where R is the radius of the earth)
R 2W2 R 2W2
1) 2)
2g g
3T
1) The force is zero at t =
4 R 2W2 R 2W2
3) 4)
2) The acceleration is maximum at t = T 4g 8g
T 49. Two bodies of masses 4 kg and 9 kg are
3) The speed is maximum at t = placed at distance ‘r’ apart. The gravitational
4
T potential at the position where the
4) The PE is equal to KE of the oscillation at t gravitational field due to them is zero
2
4G 25G
46. A thermodynamic cycle xyzx is shown on a 1) − 2) −
V-T diagram. The P-V diagram that best r r
describes this cycle is (Diagrams are 16G 9G
3) − 4) −
schematic and not to scale) r r
50. In the following figure, which of the following
sample A (or) B has shorter mean life

P
x y
1) 2) 1) B 2) A
z
V 3) A and B both 4) neither A nor B
7
CHEMISTRY
SECTION – A (35 questions)
(Answer all the questions)
51. The equivalent weight of MnSO4 is half of its Which of the above statements are correct?
molecular weight, when it is converted to 1) a and b only 2) b and d only
1) Mn2O3 2) MnO2 3) b and c only 4) a, b, c and d
-
3) MnO4 4) MnO4-2 56. The correct statements regarding C2 molecule
52. Under critical conditions, the compressibility based on molecular orbital theory.
factor for a gas is 1) C2 molecule is diamagnetic
3 8
1) 2) 2) Bond order of C2 = 2
8 3
3) The two bonds in C2 are Pi bonds
1
3) 1 4) 4) All are correct
4
57. The table gives the first four ionization
53. The suggested Mechanism for the reaction
energies in kJ mol-1 of four elements (The
CHCl3 (g) + Cl2(g) → CCl4(g) + HCl (g) is
K
letters are not the symbols for the elements).
Cl2 ↽ K 1 ⇀ 2Cl• (Fast) Which element occurs in group 13 of the
2

• periodic table?
CHCl3 + Cl• 
K3
→ HCl + C Cl3 (Slow) I.E1 I.E2 I.E3 I.E4

C Cl3 + Cl• 
K4
→ CCl4 (Fast) Element P 502 4569 6919 9550
The experimental rate law consistent with the Element Q 526 7305 11822 11950
mechanism is Element R 584 1823 2751 11584
1) Rate = K3 [CHCl3] [Cl2] Element S 796 1583 3238 4362
2) Rate = K4[CCl3] [Cl]
1) Element P 2) Element Q
3) Rate = Keq [CHCl3][Cl2]
1 1
3) Element R 4) Element S
4) Rate = K 3 K eq2 [CHCl3 ] [ Cl2 ] 2 58. Select the incorrect statement
1) Mg2C3 react with H2O forms propyne gas,
54. Which of the following is not a characteristic
of chemisorption? C3−4 ion contains 2σ bonds and 2π bonds
1) Adsorption is irreversible 2) Density of Mg is less than ‘Ca’
2) Adsorption is multi layered 3) Be, Sn and Ga are amphoteric metals.
3) Adsorption is specific 4) CaO2 is more stable than MgO2
4) Adsorption increases with increase of 59. In context of the Lanthanoids, which of the
surface area of adsorbent. following statements is not correct?
55. Consider the following statements about 1) There is a gradual decrease in the radii of
carbohydrates. the members with increasing atomic
a) Bromine water can be used to differentiate number in the series.
aldose from a ketose. 2) All the members exhibit ‘+3’ oxidation
b) All monosaacharides, whether aldoses or state.
ketoses, are reducing sugars. 3) Because of similar properties the separation
c) Sucrose is reducing sugar of Lanthanoids is not easy.
d) A pair of diastereomeric aldoses which differ 4) Availability of 4f electrons result in the
in configuration about C – 2 termed as pair of formation of compound in +4 state for all
epimers. the members of the series.
8
60. Which is the correct sequence in the 66. 10.8 g Ag is deposited at cathode when ‘X’
following properties? For the correct order amperes of current is passed through aqueous
mark (T) and for the incorrect order mark (F) solution of AgNO3 for 193 seconds. The value
a) Reducing nature : NH3 < PH3 < AsH3 < of ‘X’ is
SbH3 < BiH3 1) 25 amp 2) 50 amp
b) Boiling point : NH3 < SbH3 < AsH3 < PH3 3) 10.8 amp 4) 193 amp
67. Which one of the following orders represents
c) Lewis basic strength :
the correct order for the properties indicates
against them?
d) Dipole moment order : NH3 > PH3 > AsH3 1) H2O < H2S < H2Se < H2Te – acidic character
> SbH3> BiH3 2) H2O < H2S < H2Se < H2Te – thermal stability
1) TTTT 2) FTTT 3) H2S > H2Se < H2Te < H2O – reducing character
3) TFTT 4) TFFT 4) H2O < H2S < H2Se < H2Te – boiling point
61. In oxygen masks for emergency breathing, 68. Half life period for a substance following first
source of oxygen used is order reaction is 10 days. After how many
1) Li2O 2) Na2CO3 days, 75% of the reactant got reacted?
3) KO2 4) BaO 1) 10 days 2) 20 days
3) 90 days 4) 120 days
62. Which structure of protein is having β-pleated
69. Which of the following is incorrect regarding
sheet shape?
deviation from Raout’s law?
1) Primary 2) Secondary
1) Ethanol-acetone, +ve deviation
3) Tertiary 4) Quaternary
2) CS2-acetone, +ve deviation
63. Which could be the major product of the
3) Chloform-acetone, -ve deviation
following reaction?
4) Phenol-aniline, +ve deviation
70. The velocities of two particles A and B are

Cl 2
→ product
AlCl3 0.05 and 0.02 ms-1 respectively. The mass of B
is five times the mass of A. The ratio of their
de-Broglie’s wavelength is
1) 2)
1) 2 : 1 2) 1 : 4
3) 1 : 1 4) 14 : 1
71. Molecular shapes of SF4, CF4, XeF4 are
3) 4) 1) The same with 2, 0 and 1 lone pair of
electron respectively
64. Lassaigne's test for the detection of nitrogen 2) The same with 1, 1 and 1 lone pair of
will fail in the case of electron respectively
1) NH2CONH2 2) NH2 CONH NH2.HCl 3) Different with 0, 1 and 2 lone pair of
electrons respectively
3) NH2NH2.HCl 4) C6H5NHNH2. 2HCl
4) Different with 1, 0 and 2 lone pair of
65. The change in internal energy of a given electron respectively
reaction at 300K is -908 kJ mol-1. Then the 72. The hardness of water sample (in terms of
enthalpy change is equivalents of CaCO3) containing 10-3 M
4NH3(g) + 5O2(g) → 4NO(g) + 6H2O(l)
CaSO4 is (molar mass of CaSO4=136 g mol-1
1) -920.47 kJ 2) -908.0 kJ
1) 10ppm 2) 50ppm
3) 0 kJ 4) -454.0 kJ
3) 90ppm 4) 100ppm
9
73. Arrange the following compounds in order of
3) 4)
decreasing acidity
81. Strong acid among HNO3, HClO4 and H2SO4
in water
1) HNO3 2) HClO4
3) H2SO4 4) All are equal
82. Which of the following does not give
iodoform test?
1) II > IV > I > III 2) I > II > III > IV 1) CH3CHO 2) CH3CH2OH
3) III > I > II > IV 4) IV > III > I > II 3) CH3COCH3 4) CH3COOH
74. Ozone in stratosphere is depleted by
83. The increasing order of wavelengths of
1) CF2Cl2 2) C2F6
absorption for the complex ions
3) C6H6Cl6 4) C6F6
I) [Cr(NH3)6]3+ II) [CrCl6]3-
75. Which of the following is incorrect about
their thermal decomposition? III) [Cr(H2O)6]3+ IV) [Cr(CN)6]3- is
1) NH4NO3 ∆ 1) IV < II < III < I 2) IV < III < II < I
→ N2O
3) IV < I < III < II 4) II < III < I < IV
2) NH4NO2 ∆
→ N2
∆ N 84. Which of the following incorrect about
3) (NH4)2Cr2O7 → 2
polymers
1
4) LiNO3 ∆
→ LiNO2 + O2 1) Terylene, glyptal ⇒ condensation &
2 polyesters
76. Which of the following can be both
2) Nylon-6,6, Nylon-2,6 ⇒ condensation &
bactericidal and broad spectrum antibiotic
1) Pencilin G 2) Chloramphenicol polyamides
3) Ofloxacin 4) All of these 3) Teflon, PVC ⇒ addition & plastics
77. Which of the following is incorrect 4) Orlon, Styrene ⇒ condensation & fibres
1) Ge, Si, Ga, are refined by zone refining
2) Ni, Ti, Zr are refined by vapour phase
refining 85.
3) Cu, Ag, Au are refined by electrolysis
4) Sn, Pb are refined by distillation C 
O O →D
|| ||
78. Find the incorrect statement about noble gases CH3 − C −O −C − CH3

1) All are monoatomic Product ‘D’ would be?


2) Colourless and chemically inert (less reactive) NH2 OCOCH3
3) Insoluble in water
4) Inflammable and does not conduct electricity 1) 2)
79. Which of the following complexes show both
geometrical and optical isomerism COCH3 NO2
1) Ma3b3 2) Ma4b2
NH2
3) M(aa)3 4) M(ab)3
80. Which of the following is more stable alkene 3) 4)
based on hyperconjugation
1) 2) NHCOCH3
10
SECTION – B (15 questions)
Section – B consists of 15 questions. Candidates are allowed to answer any 10 questions only out of 15
questions given. In case if candidates attempts more than 10 questions, first 10 attempted questions only
considered for marking.
86. A quantity of 10 g of a mixture of C2H6 and 90. Rank in order of increasing rate of reaction
C5H10 occupy 4480 mL at 1 atm and 273 K. towards electrophilic aromatic substitution
The percentage of C2H6 by mass, in the with bromine in the presence of FeBr3
mixture is
1) 30% 2) 70%
a) b) c)
3) 50% 4) 60%
87. Standard Entropies of x2, y2 and xy3 are 60, 40 1) b < a < c 2) c < a < b
and 50 J k-1 mol-1 respectively. At what 3) a < b < c 4) a < c < b
temperature the reaction will be in 91. If the colour absorbed by [Ni(H2O)2(en)2]+2 is
equilibrium. For the reaction. blue – purple then what would be the colour
1 3
x 2(g) + y 2(g ) → xy3 (g) ∆ H 0 = − 30 kJ absorbed by [Ni(en)3]+2 ?
2 2
1) Green 2) Red
1) 1000 K 2) 1250 K
3) Yellow 4) Violet
3) 500 K 4) 750 K
92. In which of the following silicates only two
88. Phenol dimerizes in Benzene, if the observed corners per tetrahedron are shared?
molecular mass of phenol in solution is 120, I. Pyro silicate II. Cyclic silicate
It’s degree of dimerization is
III. Single chain silicate IV. 3D silicate
1) 0.600 2) 0.433
1) I only 2) III only
3) 0.277 4) 0.866
3) Both II & III 4) Both I and IV

89. 93.  i)O3


ii) Zn + H 2 O
→ A+B

A is 
dil.NaOH

→C
The product ‘C’ is
1)

1)
2)
2)

3)
O
3) Ph C
4)
4)
11
2+ 2+
94. The emf of the cell Mg | Mg (0.01 M) || Sn OC6H5
(0.1 M) | Sn at 298 K is (Given, E oMg +2 |Mg =
1) 2)
-2.34 V, E o
Sn +2 |Sn
= −0.14V)
NH2
1) 2.23 V 2) 1.86 V
OH
3) 1.56 V 4) 3.26 V
C 6H 5
3) 4)

95. 
NBS
→ X 
alc.KOH
→ Y;
1eq. ∆ NH2
98. For a concentrated solution of weak
Product Y is electrolyte AxBy of concentration ‘C’ The
degree of dissociation α is given by (if α <<< 1)
K eq CK eq
1) 2) 1) α = 2) α =
C(x + y) xy
1
 K  x+y K eq
3) α =  x + y −1 eq x y  4) α =
C . x .y  Cxy

3) 4) 99. Two moles of PCl5 is heated in a closed vessel


of 2 L capacity. When the equilibrium is
attained 40% of it has been found to be
96. If NaCl is doped with 4 × 10-2 mol % of dissociated. What is the Kc in mol/dm3?
CaCl2, concentration of cation vacancy per 1) 0.532 2) 0.266
mol is 3) 0.133 4) 0.174
1) 3.12 × 1020 O O
20
2) 6.02 × 10
100.
3) 1.204 × 1019
N N N O
4) 2.408 × 1020
H CH3 H
97. For the reaction
(I) (II) (III)
Which among these can exhibit tautomerism
1) only I
2) only III
3) I and III
The major product formed is 4) I, II and III
12
BOTANY
SECTION – A (35 questions)
(Answer all the questions)
101. Identify the incorrect combination 109. Termination of polypeptide chain is brought
1) Unicellular fungi – Fragmentation about by
2) Amoeba – Binary fission 1) UUG, UAG and UCG
3) Yeast – Budding 2) UAA, UAG and UGA
4) Planaria – True regeneration 3) UUG, UGC and UCA
102. Exogenous asexual spores and endogenous 4) UCG, GCG and ACC
sexual spores are found in 110. Identify the correct statement regarding
1) Ascomycetes 2) Basidiomycetes antibiotics
3) Phycomycetes 4) Deuteromycetes i) Very effective against viral diseases
103. Which of the following are related to prions? ii) Produced by microbes and used to kill all
1) Abnormally folded proteins pathogens
2) Cr - Jacobdisease iii) Used to treat Plague, Leprosy
3) Mad cow disease 1) i and iii 2) ii and iii
4) All of the above 3) iii only 4) i, ii and iii
111. Mathematical expression of arithmetic growth is
104. Gymnosperms differ from grasses in
1) lack of xylem tracheids 1) Lt = Lo + rt 2) W1 = Wo ert
2) absence of pollen tubes 3) Lo = Lt + rt 4) Wo = W1 ert
112. Find out incorrect statement w.r.t asexual
3) formation of endosperm
reproduction
4) production of seeds
i) Offspring are morphologically identical but
105. Basic structure of DNA can be studied by
not genetically
1) X-ray crystallography
ii) Uniparental process
2) Paper chromatography
iii) Usually occurs in lower organisms.
3) Ultracentrifuge
iv) Requires syngamy
4) Light microscope
1) i and iii 2) i and iv
106. Unequivocal experimental proof that the DNA
3) ii and iii 4) i, ii and iv
act as genetic material was given by
113. The common event occurs during cyclic and
1) Hershey and Chase
non-cyclic electron transport is
2) Watson and Crick
1) Photolysis of water 2) Phosphorylation
3) Franklin and Wilkins +
3) Reduction of NADP 4) Both 2 and 3
4) Gamow and Nirenberg
114. Plants absorb nitrogen from the soil mainly in
107. During DNA replication, strands separation is
the form of
done by
1) ammonia 2) Nitrogen
1) DNA polymerase 2) Ligase
3) nitrite 4) nitrate
3) Helicase 4) Primase
115. Which of the following elements is central
108. Which of the following Bt genes that control atom of chlorophyll?
corn borers?
1) Carbon 2) Nitrogen
1) cry I Ac 2) cry II Ab
3) Magnesium 4) Zinc
3) cry I Ab 4) Both (1) and (2)
13
116. Identify the mismatch from the following. 124. Crossing over occurs between
1) Simple diffusion – Transport saturation 1) Sister chromatids of homologous
2) Facilitated diffusion – Downhill transport chromosomes.
3) Active transport – Transport saturation 2) Non-sister chromatids of homologous
4) Simple diffusion – Pumps are not involved chromosomes.
117. Which of the following cellular constituent 3) Sister chromatids of non-homologous
provides cell motility, mechanical support and chromosomes.
maintenance of the shape of the cell? 4) Non-sister chromatids of non-homologous
1) Cell wall chromosomes.
2) Flagella 125. The genotype commonly found in F1 and F2

3) Cytoskeleton offspring of Mendel’s dihybrid cross is


4) Endoplasmic reticulum 1) RRYY 2) RrYy
118. Find out the mis-match from the following
3) rryy 4) Rryy
126. Which of the following palindromic sequence
1) Ribosomes – Protein synthesis
can be easily cut by some particular restriction
2) Nucleolus – rRNA synthesis
endonuclease?
3) SER – Lipid synthesis
4) Lysosomes – DNA synthesis 1) 2)
119. Insulin is a
1) polysaccharide
3) 4)
2) polymer of aminoacids
127. Which of the following is true w.r.t
3) phytohormone
transformed cells?
4) lipid
1) all transformed cells contain rDNA
120. Identify the incorrect statement
2) all transformed cells contain gene of
1) Starch forms helical secondary structure
interest
2) Exoskeleton of arthropods have chitin
3) all transformed cells contain vector
3) Generally protein is a heteropolymer
4) all non recombinants contain rDNA
4) The right end of glycogen is called non-
128. Plasmids are most suitable vectors for gene
reducing end
cloning because
121. Aestivation without overlapping of perianth
1) These are small circular DNA molecules which
members is
can integrate with host chromosomal DNA
1) Vexillary 2) Imbricate
2) These are small, circular, DNA molecules
3) Valvate 4) Twisted with their own ori site
122. In mango, the fruit is developed from
3) These can shuttle between prokaryotic and
1) Monocarpellary and inferior ovary eukaryotic cells
2) Monocarpellary and superior ovary 4) They carry antibiotic resistance genes in all
3) Bicarpellary and superior ovary bacteria
4) Bicarpellary and inferior ovary 129. Transgenic mice are being developed for
123. In which phase of mitosis chromosomes are 1) Vaccine safety testing
aligned on the equator of the spindle? 2) Molecular diagnosis
1) Anaphase 2) Prophase 3) Production of human protein enriched milk
3) Metaphase 4) Telophase 4) Production of humulin
14
130. Which of the following is a biopesticide? 133. Which of the following characters of pea plant
1) Nucleopolyhedrovirus (NPV) was not studied by Mendel?
2) Azolla 1) Flowers colour 2) Fruit colour
3) Rhizobium 3) Seed shape 4) Pod length
4) Agrobacterium 134. Percentage of “t” gametes produced by F1 pea

131. Which of the following is an unique feature of plant in monohybrid cross is


non-archegoniate spermatophytes? 1) 25% 2) 50%
1) Naked seeds 3) 75% 4) 0%
2) Multicellular female gametophyte 135. Which of the following prokaryote has high

3) Double fertilization rate of biomass production and used as SCP?


4) Secondary growth 1) Methylophilus methylotrophus
132. The ratio of ribosomes and mRNA in a 2) E.coli
polysome respectively is 3) Chlorella
1) 1 : 1 2) 1 : n 4) Yeast
3) n : 1 4) 1 : 2
SECTION – B (15 questions)
Section – B consists of 15 questions. Candidates are allowed to answer any 10 questions only out of 15
questions given. In case if candidates attempts more than 10 questions, first 10 attempted questions only
considered for marking.
136. Which of the following is a non-vascular 3) Hydrophily occurs in water hyacinth and
embryophyte? water lily
1) Marchantia 2) Equisetum 4) The primary endosperm nucleus is triploid
3) Selaginella 4) Pinus in angiosperms
137. Match the codons with their respective amino 139. Perisperm differs from endosperm in
acids and choose the correct answer 1) being fertilized product
Column – I Column – II 2) being a haploid tissue
a) UUU 1) Methionine 3) having no reserve food
b) GGG 2) Phenylalanine 4) being a diploid tissue
c) CCC 3) Glycine 140. Electron transport system is located in

d) AUG 4) Proline mitochondrial A and protons are


accumulated in B during the ETS.
a b c d
1) 2 3 4 1 A B
2) 3 1 4 2 1) Inter membrane Matrix
3) 3 4 2 1 space
4) 2 4 1 3 2) Matrix Inner membrane
138. Study the following statements and select the 3) Inner Matrix
incorrect option. membrane
1) Tapetum nourishes the developing pollen 4) Inner Inter membrane
grains
membrane space
2) Hilum represents the junction between
ovule and funicle
15
141. Match the column I with column – II and D) Seed coats of legumes
choose the correct option. 1) A and C 2) B and D
Column – I Column – II 3) B, C and D 4) A, B, C and D
A) Human urine I) Ethylene 146. Arrange the following parts developed during
B) Gibberella II) Abscisic acid secondary growth in dicot stem, in centripetal
fujikuroi manner
i) Cork cambium ii) Phelloderm
C) Fruit ripening III) Auxin
iii) Secondary phloem iv) Phellem
D) Closure of stomata IV) GA v) Secondary xylem vi) Vascular cambial ring
A B C D 1) v, vi, iii, ii, i, iv 2) v, vi, ii, I, iii, iv
1) IV III II I 3) iv, i, ii, iii, vi, v 4) iv, i, ii, v, vi, iii
2) III IV I II 147. Human gene could not be expressed in
3) II I III IV bacteria because
4) I II IV III 1) human gene have intrans
142. How many of the given statements are correct 2) genetic code is not inverse
w.r.t C4 plants? 3) human protein is sensitive to bacteria
A) They have a special type of leaf anatomy 4) all of the above
B) They tolerate to higher temperatures. 148. Identify the true statement regarding genetic
C) They show response to high light intensities engineering.
D) They generally lack photorespiration 1) a gene from eukaryotic cell can be made to
E) They have greater productivity of biomass express in a prokaryote
1) Two 2) Three 2) a gene can be synthesized artificially
3) Four 4) Five 3) a gene from animal cell can be made to
143. Observe the following diagram of three cells A, express in a plant cell
B, C and select the correct set of answers 4) all of the above
related to movement of water between the cells. 149. Statement – I : Transfer of alien gene from
one organism to another is made only by a
vector.
Statement – II : Alien genes can also be
transferred through methods like
electroporation, genegun, biolistic and
microinjection.
1) Both the statements are correct
A B 2) Both the statements are incorrect
1) 2) 3) Statement – I is correct, statement – II is
C incorrect
A B 4) Statement – I is incorrect, statement - II is
3) 4)
C correct
144. Semilunar patch of sclerenchymatous 150. Which among the following is an exception to
pericycle is found in cell theory
1) Monocot root 2) Dicot root 1) Infectious agent that causes madcow
3) Dicot stem 4) Monocot stem disease
145. Sclereids are commonly found in
2) Causative organism of cholera
A) Pericarp of nuts 3) Parasite that causes citrus canker
B) Pulp of fruits of guava and sapota 4) Microbe used in biogas production
C) Leaves of tea
16

ZOOLOGY
SECTION – A (35 questions)
(Answer all the questions)
(A): Class - Osteichthyes includes
151. Assertion 156. Statement-I: All connective tissues except
both marine and freshwater bony fishes. blood, the cells secrete fibres of structural
Reason (R): Cartilaginous fishes have proteins called collagen or elastin.
persistent notochord. Statement - II: The excess of nutrients which
1) Both A and R are true and R is the correct are not used immediately are converted into
explanation of A fats.
2) Both A and R are true and R is the not 1) Both Statements I and II are correct
correct explanation of A 2) Statement I is correct, statement II is
3) A is true but R is false incorrect
4) A is false but R is true 3) Statement I is incorrect, statement II is
correct
152. A common characteristic of all vertebrates
without exception is 4) Both statements I and II are incorrect
157. Which of the following statements are not
1) the division of body into head, neck, trunk
and tail correct w.r.t cockroach?
i) Blood vascular system is closed type
2) their body is covered with an exoskeleton
ii) Blood vessels are poorly developed and
3) they have two pairs of functional
open into haemocoel.
appendages
iii) Alary muscles are help in blood circulation
4) presence of skull and vertebral column
iv) Blood from sinuses enter the heart through
153. The true metazoans with diploblastic body, ostia and is pumped posteriorly to sinuses
exhibiting radial symmetry are again
1) flatworms 2) sponges 1) i and ii only 2) ii and iii only
3) roundworms 4) cnidarians 3) iii and iv only 4) i and iv only
154. Find the correct statement w.r.t sponges. 158. The undigested food enters into the caecum
1) Fertilization is external through
2) Development is indirect 1) cardiac sphincter
3) Larval stage is morphologically similar to 2) ileo-caecal valve
that of the adult 3) pyloric sphincter
4) Eggs and sperms are produced by two 4) sphincter of Oddi
different sexes 159. At which thoracic vertebrae does the trachea
155. Find the similarities between cockroach and divide into right and left primary bronchi?
bird. 1) 5 2) 6
1) Organ system level of organization 3) 9 4) 4
2) Digestive tract has crop and gizzard 160. Choose the options which correctly fills the

3) Presence of exoskeleton blanks to complete the analogy.


4) All of these Reptiles : pulmonary respiration
Aquatic arthropods : X
17
1) Tracheal respiration 165. A part of myofibril without thin filaments is
2) Branchial respiration called
3) Cutaneous respiration 1) Z - line 2) H - zone
4) Bronchial respiration 3) M - zone 4) H - line
161. Select the correct statement. 166. The rib cage of human includes

1) Blood is the most commonly used body 1) 1st - 6th vertebrae


fluid by all organisms including humans 2) 8th - 19th vertebrae
2) T-lymphocytes only responsible for 3) 12th - 22nd vertebrate
immune responses of the human body 4) 7th - 12th vertebrae
3) RBCs have an average life span of 120 167. The _____ is attached to the tympanic
days after which they are destroyed in the membrane and the _____ is attached to the
bone marrow oval window of the cochlea.
4) Eosinophils are the type of granulocytes 1) Malleus, Incus 2) Incus, Stapes
which can resist infections 3) Malleus, Stapes 4) Stapes, Malleus
162. Assertion (A): Spleen is called graveyard of
168. A projecting ridge of semicircular canal is
WBCs. called
Reason (R): WBCs are destroyed in spleen 1) Crista 2) Macula
after completing their average life span. 3) Organ of Corti 4) Saccule
1) Both A and R are true and R is the correct 169. Statement-I: Hormones which interact with
explanation of A
membrane bound receptors normally do not
2) Both A and R are true and R is the not enter the target cell but generate second
correct explanation of A messengers.
3) A is true but R is false Statement - II: Hormones which interact with
4) Both the A and R are false intracellular receptors mostly regulate gene
163. Match the following columns and select the expression/chromosome function by interaction
correct option. of hormone receptor complex with the genome.
Column - I Column - II 1) Both Statements I and II are correct
A. length of kidney i) 2 - 3 cm 2) Statement I is correct, statement II is
incorrect
B. weight of kidney ii) 5 - 7 cm
3) Statement I is incorrect, statement II is
C. thickness of kidney iii) 120 - 170 gm correct
D. width of kidney iv) 10 - 12 cm 4) Both statements I and II are incorrect
1) A - i, B - ii, C - iii, D - iv 170. Choose the correct option for type of hormone
2) A - iv, B - i, C - iii, D - ii which do not generate second messengers to
3) A - iv, B - iii, C - i, D - ii regulate cellular metabolism
4) A - i, B - iii, C - ii, D - iv 1) protein 2) peptide
164. How much amount of CO2 removed by our
3) amino acid derivatives 4) iodothyronines
lungs? 171. The human embryo at 8 - 16 celled stage is

1) 200 mL/day 2) 200 mL/hour called as


3) 200 mL/min 4) 25 - 30 mL/min 1) morula 2) blastula
3) blastocyst 4) trophoblast
18
172. Match the column - I and column - II. 1) A - v, B - ii, C - i, D - iv, E - iii
Column - I Column - II 2) A - i, B - iii, C - ii, D - iv, E - v
A. Head i) Mitochondria 3) A - iii, B - iv, C - ii, D - i, E - v
4) A - iv, B - v, C - iii, D - ii, E - i
B. Acrosome ii) Haploid nucleus +ve
177. A Rh man (heterozygous) is married a Rh-ve
C. Middle piece iii) Sperm motility
lady. They had four children only first and the
D. Tail iv) Filled with enzymes last are Rh+ve then the child with
1) A - ii, B - iv, C - i, D - iii erythroblastosis foetalis / HDNB among the
2) A - iii, B - ii, C - iv, D - i four children probably is
3) A - iv, B - i, C - ii, D - iii 1) 2nd child
4) A - iv, B - ii, C - i, D - iii 2) 3rd child
173. How many haploid spermatids are formed
3) both 2nd and 3rd child
from 16 secondary spermatocytes after second 4) 4th child
meiotic division? 178. W.r.t erythroblastosis foetalis, which of the

1) 4 2) 8 following will pass from the mother through


3) 16 4) 32 placenta into the foetus?
174. Match the following columns and choose the
1) Rh antigens 2) Rh antibodies
correct option. 3) Agglutinins 4) ABO antibodies
179. A pleiotropic gene causes
Column - I Column - II
1) single phenotype and multiple genotypes
A. Gonorrhea i) HIV
2) single phenotype and single genotype
B. Syphilis ii) Neisseria gonorrhoeae 3) multiple phenotypes and single genotype
C. Genital warts iii) Treponema pallidum 4) multiple phenotypes and multiple genotypes
D. AIDS iv) Human papilloma virus 180. The process of evolution of different species

1) A - iv, B - iii, C - ii, D - i in a given geographical area starting from a


2) A - ii, B - iii, C - iv, D - i point and literally radiating to other habitats is
called
3) A - i, B - ii, C - iv, D - iii
1) convergent evolution
4) A - iii, B - i, C - ii, D - iv
2) adaptive radiation
175. MTP is considered relatively safe during
3) saltation
1) second trimester 2) 24 week of pregnancy
4) founder effect
3) first trimester 4) third trimester
181. Tendrils of cucurbita and thorns of
176. Match the following.
Bougainvillea explains
Column - I Column - II 1) Analogy
A. ZIFT i) Zygote intra fallopian transfer 2) Convergent evolution
B. IUT ii) gamete intra fallopian transfer 3) Homology
C. GIFT iii) Intra uterine transfer 4) Fossil history
182. Industrial melanism is an example of
D. IUI iv) Intra uterine insemination
1) mutation 2) Lamarckism
v) Intra cytoplasmic sperm
E. ICSI 3) natural selection 4) saltation
injection
19
183. Statement-I: If a person is physically fit then 184. Which of the following is not a common
he is healthy. symptom in typhoid?
Statement - II: Health is a state of complete 1) Sustained high fever (39 to 40oC)
physical, mental and social well being. 2) Weakness
1) Both Statements I and II are correct 3) Stomach pain
2) Statement I is correct, statement II is 4) Intestinal perforation
incorrect 185. Which of the following is one of the most
3) Statement I is incorrect, statement II is common infectious disease in man?
correct 1) Ascariasis 2) Amoebiosis
4) Both statements I and II are incorrect 3) Ringworm 4) Malaria
SECTION – B (15 questions)
Section – B consists of 15 questions. Candidates are allowed to answer any 10 questions only out of 15 questions
given. In case if candidates attempts more than 10 questions, first 10 attempted questions only considered for
marking.
186. Assertion (A): Organs cannot be taken from 188. A population has less young individuals
just any source for transplantation. compared to the older individuals. What
Reason (R): Tissue matching, blood group would be the status of the population after
matching are essential before undertaking any some years?
transplant. 1) It will decline
1) Both A and R are true and R is the correct 2) It will stable
explanation of A 3) It will first decline and then stabilize
2) Both A and R are true and R is the not 4) It will increase
correct explanation of A 189. Cuckoo lays eggs in the nest of crow during
3) A is true but R is false breeding season. This is an example for
4) A is false but R is true 1) Brood parasitism 2) Mutualism
187. Match the following. 3) Endoparasitism 4) Commensalism
190. Assertion (A): Earthworm being referred to as
Column - I Column - II
the farmer’s friend.
A. Artificial Reason (R): Earthworm helps in the
i) Hisardale
insemination breakdown of complex organic matter as well
B. MOET ii) Overcome STIs as in loosening of the soil.
iii) High quality meat yielding 1) Both A and R are true and R is the correct
C. Mule explanation of A
bulls
2) Both A and R are true and R is the not
iv) Progeny of two different
D. Cross breed correct explanation of A
related species
3) A is true but R is false
1) A - iv, B - ii, C - i, D - iii
4) A is false but R is true
2) A - ii, B - iii, C - i, D - iv
191. What kind of pyramid would you get when
3) A - i, B - ii, C - iv, D - iii number of insects feeding on big tree?
4) A - ii, B - iii, C - iv, D - i 1) Spindle shaped 2) Upright pyramid
3) Inverted pyramid 4) both 1 and 2
20
192. More than 70% of all the species recorded are Statement - II: Chemical manufacturing
1) plants 2) animals industries releases heavy metals with density >
3) fungi 4) bacteria 5 g/cm3.
193. Which is the most important cause of 1) Both Statements I and II are correct
biodiversity losses? 2) Statement I is correct, statement II is
1) Habitat loss and fragmentation incorrect
2) Over exploitation 3) Statement I is incorrect, statement II is
3) Alien species invasions correct
4) Co-extinction 4) Both statements I and II are incorrect
194. Match the following. 198. Statement-I: Landfills used to treat municipal
wastes pollute underground water resources.
Column - I Column - II
Statement - II: Thermal waste water
A. CFCs i) Refrigerants eliminates or reduces the number of organisms
B. Dobson units ii) Thickness of ozone sensitive to high temperature.
C. Ozone hole iii) Thinned ozone layer 1) Both Statements I and II are correct
D. Snow blindness iv) UV-B 2) Statement I is correct, statement II is
incorrect
1) A - i, B - ii, C - iii, D - iv
3) Statement I is incorrect, statement II is
2) A - iv, B - i, C - ii, D - iii correct
3) A - iv, B - ii, C - iii, D - i
4) Both statements I and II are incorrect
4) A - i, B - iv, C - iii, D - ii 199. Who is involved in showing exemplary
195. Algal bloom causes courage by hugging a tree and daring king’s
1) deterioration of the water quality men to save trees?
2) fish mortality 1) Amrita Devi
3) reduced algal growth 2) Ramesh Chandra Dagar
4) both 1 and 2 3) Sundarlal Bahuguna
196. Identify the wrong one. 4) Ahmed khan
1) A few toxic substances, often present in 200. Statement-I: Temperature affects the kinetics
industrial waste waters can undergo of enzymes and other physiological functions
biological magnification of the living organisms.
2) Eutrophication is natural aging of a lake Statement - II: A few organisms are restricted
3) DDT causes thinning of egg shell and to a narrow range of temperatures such
premature breaking of eggs in mammals organisms are called stenothermal.
4) The natural aging of a lake may span 1) Both Statements I and II are correct
thousands of years based on various factors 2) Statement I is correct, statement II is
197. Statement-I: The prime contaminants of incorrect
accelerated eutrophication are nitrates and 3) Statement I is incorrect, statement II is
sulphates. correct
4) Both statements I and II are incorrect
21

SPP – 2
Max. Marks: 720 Duration : 3 Hrs. 20 Min.
PHYSICS
SECTION – A (35 questions)
(Answer all the questions)

1. Dimensions of ε0 are 4. Four projectiles are fixed with the same


1) [M-1L-3T4A2] 2) [M0L-3T3A3] velocities at angles 25o, 40o, 55o and 70o with
the horizontal. The range of projectile will be
3) [M-1L-3T3A] 4) [M-1L-3TA2]
largest for the one projected at angle
2. The least count of the main scale of a vernier
1) 25o 2) 40o
calipers is 1 mm. Its vernier scale is divided
3) 55o 4) 70o
into 10 divisions and coincide with 9 divisions
of main scale. When jaws are touching each 5. In the given figure, all surfaces are
other, the 7th division of vernier scale T
frictionless, the ratio of 1 is
coincides with a division of main scale and the T2
zero of the vernier is lying right side of the
zero of main scale. When this vernier is used
to measure length of a cylinder the zero of the
vernier scale between 3.1 cm and 3.2 cm and 1) 3:2 2) 1 : 3
4th VSD coincides with a main scale division.
3) 1 : 5 4) 5 : 1
The length of the cylinder is (VSD is vernier
6. Two bodies A and B have masses 20 kg and
scale division)
5 kg respectively. Each one is acted upon by a
1) 3.2 cm 2) 3.21 cm force of 4 kgwt. If they acquire the same
3) 3.07 cm 4) 2.99 cm t
kinetic energy in times tA and tB then A is
3. Which of the following options is correct for tB
the object having a straight line motion
2
represented by the following graph? 1) 2 2)
5
5 1
3) 4)
6 2
7. The x and y coordinates of the position of the
centre of mass for a planar body of uniform
mass distribution as shown in the figure are
1) the object moves with constantly increasing
velocity from 0 to A and then it moves
with constant velocity
2) velocity of the object increases uniformly
3) average velocity is zero
4) the graph shown is impossible
22
9 9 13. Heat energy absorbed by a system in going
1) 0 m, m 2) m, 0 m
11 11 through a cyclic process shown in the figure is
20 20
3) m,0m 4) 0 m, m
11 11
8. The moment of inertia in rotational motion is
equivalent to
1) Angular velocity of linear motion
2) Mass of linear motion 1) 107 πJ 2) 104 πJ
3) Frequency of linear motion 3) 102 πJ 4) 10-3 πJ
4) Current 14. The temperature of argon, kept in a vessel is
9. Three particles of mass m are placed at the raised by 1oC at a constant volume. The total
vertices of an equilateral triangle of side l. The heat supplied to the gas is a combination of
work required in placing the particles at the translational and rotational energies. Their
vertices of an equilateral triangle of side 2l is respective shares are
Gm 2 Gm 2 1) 50 % and 50 % 2) 60 % and 40 %
1) 2) 3) 75 % and 25 % 4) 100 % and 0 %
2l l
15. Displacement versus time curve for a particle
3Gm 2 2Gm 2
3) 4) executing SHM is shown in the figure.
2l l Identify the points marked at which the
10. A wooden bar is subjected to a tensile stress of velocity of the oscillator is zero
5 MPa. The value of normal stress across a
section, which makes an angle of 25o with the
direction of the tensile stress is
[cos-1(0.6425)= 50o]
1) 0.9 Mpa 2) 1.9 Mpa 1) A, C, E, G 2) B, D, F, H
3) 2.9 Mpa 4) 4.9 Mpa 3) A, B, C, D 4) E, F, G, H
11. A liquid wets glass. It’s angle of contact with 16. Two closed pipes have the same fundamental
glass is frequency. One is filled with oxygen and the
π π other with hydrogen at the same temperature.
1) less than 2)
2 2 Ratio of their lengths respectively is
3π 1) 1 : 4 2) 4 : 1
3) 4) π
2 3) 1 : 2 4) 2 : 1
12. Steam at 100oC is passed into 1.1 kg of water 17. The maximum electric field intensity on the
contained in a calorimeter of water equivalent axis of a uniformly charged ring of charge ‘q’
0.02 kg at 15oC till the temperature of the and radius R is
calorimeter and its contents rises to 80oC. The 1 q 1 2q
1) 2)
mass of the steam condensed (in kg) is 4πε0 3 3R 2
4πε0 3R 2
1) 0.130 2) 0.065 1 2q 1 2q
3) 4)
3) 0.260 4) 0.135 4πε 0 3 3R 2 4πε 0 2 2R 2
23
18. If the potential function is given by 22. The coil of galvanometer consists of 100 turns
V = 4x + 3y then the magnitude of electric and effective area of 1 sq.cm. The restoring
field intensity at the point (2, 1) (in N/C) is couple is 10-8Nm rad-1. The magnetic field
1) 11 2) 5 between the pole pieces is 5T. The current
sensitivity of this galvanometer is
3) 7 4) 1
1) 5 x 104 rad/µA
19. What will be the equivalent resistance
between the two points A and D? 2) 5 x 10-6 rad/µA

10 3) 2 x 10-7 rad/µA
10 10
A C 4) 5 rad/µA
23. If area vector A = 3iˆ + 2ˆj + 5kˆ m2, flux density
10 10
vector B = 5iˆ + 10ˆj + 6kˆ Wb/m2. The magnetic
B D
10 10 10 flux linked with the coil is
1) 10 Ω 2) 20 Ω 1) 31 Wb 2) 9000 Wb
3) 30 Ω 4) 40 Ω 3) 65 Wb 4) 100 Wb

20. Which of the following statement is false? 24. The coefficients of self inductance of two coils
are 0.01 H and 0.03 H respectively. When they
1) Heat produced in a conductor is
are connected in series so as to support each
proportional to its resistance
other, then the resultant self inductance
2) Heat produced in a conductor is becomes 0.06 H. The value of coefficient of
proportional to the square of current mutual inductance will be
3) Heat produced in a conductor is 1) 0.02 H 2) 0.05 H
proportional to charge 3) 0.01 H 4) zero
4) Heat produced in a conductor is 25. An electric bulb and a capacitor are connected
proportional to the time for which current in series with an AC source. On increasing the
is passed frequency of the source, the brightness of the
21. A length of wire carries a steady current. It is bulb
bent first to form a circular coil of one turn. 1) increases
The same length is now bent more sharply to 2) decreases
give a double loop of smaller radius. The 3) remains unchanged
magnetic field at the centre caused by the 4) none of the above
same current is
26. The potential difference across the plates of a
1) a quarter of its first value capacitor is charging at the rate of 100 Vs-1.
2) unaltered The displacement current through the
3) four times of its first value capacitor is drawn to be 10-1mA. The value of
4) half of its first value capacitance is
1) 10-3F 2) 10-6 F
3) 10-9 F 4) 10-2F
24
27. In the figure, an air lens of radius of curvature ν ν
1) 2)
10 cm (R1 = R2 = 10 cm) is cut in a cylinder of 6 3
glass (µ = 1.5). The focal length and the nature 2ν 4ν
3) 4)
of the lens is 3 3

32. As per Bohr model, the minimum energy (in


eV) required to remove an electron from the
ground state of doubly ionized Li atom (Z = 3)
is
1) 15 cm, concave 1) 1.51 2) 13.6
2) 15 cm, convex 3) 40.8 4) 122.4
3) ∝ neither concave nor convex 33. The radioactivity of a given sample of whisky
4) 0, concave due to tritium (half life 12.6 years) was found
28. The focal lengths of objective and eye-piece of to be only 3.125% of that measured in a
a telescope are respectively 100 cm and 2 cm. recently purchased bottle marked 7 years old.
The moon subtends an angle of 0.5o at the eye. The sample must have been prepared about
It is looked through the telescope, the angle 1) 220 years back
subtended by the moon’s image will be 2) 300 years back
1) 100o 2) 50o 3) 400 years back
o o
3) 25 4) 10 4) 70 years back
29. Light is incident on a glass surface at 34. Two ideal diodes are connected to a battery as
polarizing angle of 57.5o, then the angle shown in the circuit. The current supplied by
between the reflected ray and the refracted ray the battery is
is
1) 57.5o 2) 115o
3) 65o 4) 90o
30. If the kinetic energy of particle is increased by
16 times, the percentage change in de-Broglie
wavelength of the particle is
1) 25 % 2) 75 %
3) 60 % 4) 50 % 1) 0.53 A 2) 0.35 A
31. When a certain metal surface is illuminated 3) 0.23 A 4) zero
with light of frequency ν, the stopping 35. A common emitter amplifier has a voltage
potential for photo electric current is V0. gain of 50, an input impedance of 100 Ω and
When the same surface is illuminated by light an output impedance of 200 Ω. The power
ν V gain of amplifier is
of frequency , the stopping potential is 0 .
2 4 1) 500 2) 1000
The threshold frequency for photoelectric 3) 1250 4) 100
emission is
25
SECTION – B (15 questions)
Section – B consists of 15 questions. Candidates are allowed to answer any 10 questions only out of 15 questions
given. In case if candidates attempts more than 10 questions, first 10 attempted questions only considered for
marking.
36. If 100 N force is applied to 10 kg block as 41. A sample of 10 g of oxygen is heated through
shown in the diagram, the acceleration of 40 100oC at a constant pressure then, choose the
kg slab is correct option is (Take, CV= 5.03 cal mol-1K-1,
and R = 1.98 cal mol-1K-1)
1) Heat supplied is 62 cal
2) Work done by gas is 219 cal
3) Change in internal energy of gas is 157 cal
4) No work is done by the gas
1) 1.65 m/s2 2) 0.98 m/s2 42. A wire of variable mass per unit length
3) 0.5 m/s2 4) 0.25 m/s2 µ = µ0y is hanging from the ceiling as shown
37. A stone of mass 2 kg tied to a light in the figure, the length of the wire is L0. A
5 small transverse disturbance is produced at its
inextensible string of length m is whirling in
3 lower end. The time after which the
a circular path in a vertical plane. If the ratio disturbance will reach the other end is
of maximum tension to the minimum tension
in the string is 4, then the speed of the stone at
the highest point of the circle is (g = 10 m/s2)
1) 20 m/s 2) 10 3 m/s
3) 50 m/s 4) 10 m/s
38. A light rope is wound around a hollow
cylinder of mass 4 kg and radius 40 cm. If the 2 L0 8L0
1) 2)
rope is pulled with a force of 40 N, its angular g g
acceleration is
L0 12L0
1) 0.4 rad/s2 2) 0.25 rad/s2 3) 4)
2g g
3) 25 rad/s2 4) 40 rad/s2
39. Two bodies of equal masses are some distance 43. Two infinitely long parallel wires having
apart. If 20 % of mass is transferred from first linear charge densities λ1 and λ2 respectively
body to the second body, then the gravitational are placed at a distance of R meters, the force
force between them per unit length on either wire will be
1) increases by 4 % 2) increases by 14%  1 
k = 
3) decreases by 4% 4) decreases by 14%  4πε0 
40. Water in a lake starts freezing at t = 0 when 2λ1λ 2 2λ1λ 2
the outside temperature is -θoC. The time 1) k 2) k
R2 R
intervals in which thickness changes from 0 to
λ1λ 2 λ1λ 2
y, from y to 2y, from 2y to 3y will be in ratio 3) k 4) k
R2 R
1) 1 : 2 : 3 2) 1 : 4 : 9
3) 1 : 3 : 5 4) 1 : 9 : 25
26
44. A capacitor of capacitance C1 = 1µF can
withstand maximum voltage V1 = 6 kV and
another capacitor of capacitance C2 = 3µF can
withstand maximum voltage V2 = 4 kV. When
the two capacitors are connected in series, the
combined system can withstand a maximum
voltage of 1) 0.5 cms-1 2) 1 cms-1
1) 4 kV 2) 6 kV 3) 2 cms-1 4) 4 cms-1
3) 8 kV 4) 10 kV 48. Find the velocity of image at the instant as
45. A potentiometer wire of length L and shown in the figure.
resistance 10 Ω is connected is series with a
battery of emf 2.5 V and a resistance in its
primary circuit. The null point corresponding
to a cell of emf 1 V is obtained at a distance
L
. If the resistance in primary circuit is 1) 1 cm/s in leftward direction
2
doubled then the position of new null point 2) 2 cm/s in rightward direction
will be 3) 1 cm/s in rightward direction
1) 0.4 L 2) 0.5 L 4) 4 cm/s in leftward direction
3) 0.6 L 4) 0.8 L 49. In a two slit experiment with monochromatic
+ + +2
46. Three ions H , He and O having same light, fringes are obtained on a screen placed
kinetic energy passing through a region in at some distance from the slits. If screen is
which there is a uniform magnetic field moved by 5 x 10-2 m towards the slits, then
perpendicular to their velocity, then change in fringe width is 3 x 10-5 m. If the
1) H+ will be least deflected distance between slits is 10-3m, then
2) He+ and O+2 will be deflected equally wavelength of light used will be

3) O+2 will be deflected most 1) 4000 Ao 2) 6000 Ao

4) All will be deflected equally 3) 5890 Ao 4) 8000 Ao

47. A square metal of loop PQRS of side 10 cm 50. An energy of 24.6 eV is required to remove
first electron from a neutral helium atom. The
and resistance 1 Ω is moved with a constant
energy (in eV) required to remove both the
velocity partly inside a magnetic field of 2
electrons from a neutral helium atom is
Wbm-2, directed into the paper as shown in
figure. This loop is connected to a network 1) 79.0 2) 51.8
ABCD of five resistors each of value 3Ω. If a 3) 49.2 4) 38.2
steady current of 1 mA flows in the loop, then
the speed of loop is
27
CHEMISTRY
SECTION – A (35 questions)
(Answer all the questions)
51. In an organic compound of molar mass 58. When a sample of hard water is passed
108 g/mol C, H and N atoms are present in through the layer of sodium zeolite resulting
9 : 1 : 3.5 by mass molecular formula can be which of the following ions will not be present
1) C6H8N2 2) C7H10N in the resulting sample of water obtained?
3) C5H6N3 4) C4H18N3 1) Mg2+, Ca2+ 2) Ca2+, Na+
52. Which of the following is not permissible 3) Mg2+, CO32− 4) CO32−, Cl−
arrangement of electrons in an atom? 59. The correct order of thermal stability of
1) n = 5, l = 3, m = 0, s = + ½ hydroxides is
2) n = 3, l = 2, m = –3, s = – ½ 1) Ba(OH)2 < Ca(OH)2 < Sr(OH)2 < Mg(OH)2
3) n = 3, l = 2, m = –2, s = – ½ 2) Mg(OH)2 < Sr(OH)2 < Ca(OH)2 < Ba(OH)2
3) Mg(OH)2 < Ca(OH)2 < Sr(OH)2 < Ba(OH)2
4) n = 4, l = 0, m = 0, s = – ½
4) Ba(OH)2 < Sr(OH)2 < Ca(OH)2 < Mg(OH)2
53. Electron affinity is positive for
60. Among the following substituted silanes the
1) O(g) + e− → O−(g) 2) S(g) + e− → S−(g)
one which will give rise to cross linked
3) O+(g) + e− → O(g) 4)O−(g) + e− → O2−(g) silicone polymer on hydrolysis is
54. In which of the following ionization processes 1) R4Si 2) R2SiCl2
the bond energy has increased and also the 3) RSiCl3 4) R3SiCl
magnetic behavior has changed from 61. The correct IUPAC name of the following
paramagnetic to diamagnetic? compound?
1) NO → NO+ 2) N2 → N2+
3) C2 → C2+ 4) O2 → O2+
55. The work done by a system is 8 Joule, when
40 Joule of heat is supplied to it, increase in 1) 4-Methyl-3-ethylhexane
internal energy of system. 2) 3-Ethyl-4-methylhexane
1) 25 J 2) 30 J 3) 3,4-Ethylmethylhexane
3) 32 J 4) 28 J 4) 4-Ethyl-3-methylhexane
56. Following reaction occurs in an automobile. 62.
2C8H18(g) + 2SO2(g) → 16CO2(g) + 18H2O(g)
The sign of ∆H, ∆S and ∆G? The two compounds are
1) Identical 2) Enantiomers
1) +, −, + 2) −, +, −
3) Diastereomers 4) Regiomers
3) −, +, + 4) +, +, −
63. The two compounds A and B obtained from
57. Which of the following do not show
1-butyne can be distinguished by
disproportionation reaction?
ClO4−, F2, Cl2, ClO2−, ClO−, P4, S8, ClO−
1) ClO2−, ClO4−, ClO− 2) F2 only 1) NaHSO3 2) Litmus solution
3) F2 and ClO4− 4) ClO4− only 3) Iodoform test 4) 2, 4 - DNP
28
70. The reaction of A2 and B2 follows the equation
A2(g) + B2(g) → 2AB(g)
the following data were observed
64. Initial rate of appearance
[A2]0 [B2]0
of AB(g) (ms−1)
0.10 0.10 2.5 x 10-4
0.20 0.10 5 x 10-4
Identify the position where Electrophilic
substitution reaction is most favourable 0.20 0.20 10-3
1) A 2) B The value of rate constant for the above reaction is
3) C 4) A & C 1) 2.5 x 10-4 2) 2.5 x 10-2
-2
3) 1.25 x 10 4) 1.25 x 10-4
71. Arrange the following electrolytes in the
increasing order of coagulating power for the
65.
ferric hydroxide sol
Reagent A may be I) Na3PO4 II) KCl
1) H2O/H+ III) K2SO4 IV) NaCl
2) BH3, THF / H2O2 - OH− 1) I < II < III < IV 2) II = IV < III < I
3) Hg(OCOCH3)2, H2O / NaBH4, NaOH 3) II = IV < I < III 4) II = III < IV = I
4) All are possible 72. Which of the following pair is incorrectly
66. Which of the following can control the photo matched?
chemical slog? 1) Magnetite − Fe3O4
A) Use of catalytic converters in automobiles 2) Copper glance − Cu2S
B) Plantation of trees like pinus, pyrus vitis etc
3) Calamine − ZnCO3
C) Using less sulphur containing fossil fuels
4) Zincite − ZnS
1) A and C 2) B
3) A and B 4) A, B and C 73. A + H2O → B + HCl ; B + H2O → C + HCl

67. The ionic radii of X and Y− ions are 146 and


+ Compound A, B and C will be
216 pm respectively. The probable type of 1) PCl5, POCl3, H3PO3
structure shown by it is 2) PCl5, POCl3, H3PO4
1) CsCl type 2) Rock salt type 3) SOCl2, POCl3, H3PO3
3) ZnS type 4) CaF2 type 4) PCl3, POCl3, H3PO4
68. A 0.001 molal solution of PtCl4.4NH3 in water 74. P4O10 is not used to dry NH3 gas because
had a freezing point depression of 0.0054°C.
1) P4O10 reacts with moisture in NH3
If Kf for water is 1.80, the correct formulation
for the above molecule is 2) P4O10 is not a drying agent
1) [Pt(NH3)4Cl3]Cl 2) [Pt(NH3)4Cl2]Cl2 3) P4O10 is acidic and NH3 is basic
3) [Pt(NH3)4Cl]Cl3 4) [Pt(NH3)4Cl4] 4) P4O10 is basic and NH3 is acidic
69. A current of 2.0 A passed for 5 hrs through a 75. The deep blue colour produced on adding
molten metal salt deposits 22.2g of metal (At. excess of ammonia to copper sulphate is due
wt = 177). The oxidation state of the metal in to presence of
the metal salt is 1) Cu2+ 2) [Cu(NH3)4]2+
1) +1 2) +2 3) [Cu(NH3)6] 2+
4) [Cu(NH3)2]2+
3) +3 4) +4
29
76. Which of the following form of the sulphur [Nu− = (A) PhO− (B) AcO− (C) HO−
shows paramagnetic behavior? 1) A > B > C 2) B > C > A
1) S8 2) S6 3) C > A > B 4) C > B > A
3) S2 4) All of these
82.
77. Of Cr(VI) as Cr2O7 and CrO42− which is
2−

better oxidizing agent?


1) CrO42− - basic medium
2) Cr2O72− - basic medium Compound (C) is
2−
3) Cr2O7 - acidic medium
4) CrO3 – basic medium 1)
78. The complex given is
2)

3)
a) non-super imposable on its mirror image
b) optically active 4)
c) rotate plane polarized light
d) planar 83. Cannizzaro’s reaction is not given by
1) (a) and (b) 2) (a) and (d)
3) (a), (b) and (c) 4) only (d) 1) 2)
79. Which of the following carbonyls will have
3) CH3CHO 4) HCHO
the strongest C−O bond?
1) [Mn(CO)6]+ 2) [Cr(CO)6]
3) [V(CO)6]− 4) [Fe(CO)5]
80. Knowing that the chemistry of lanthanoids 84.

(Ln) is dominated by its +3 oxidation state,


which of the following is incorrect? Product (B) in this reaction is
1) The ionic size of Ln(III) decrease in general Br
with increasing atomic number Br
2) Ln (III) compounds are generally colourless 1) 2)
3) Ln (III) hydroxide are mainly basic in
character
4) Because of the large size of the Ln (III) ions Br
the bonding in its compounds is
predominantly ionic in character
81. CH3Br + Nu− → CH3−Nu + Br−
The decreasing order of the rate of the above 3) 4)
reaction with nucleophiles (Nu−) A to D is
30
85. The monomer of the polymer
1) 2) CH3CH=CHCH3

is 3) CH3CH=CH2 4) (CH3)2C=C(CH3)2

SECTION – B (15 questions)


Section – B consists of 15 questions. Candidates are allowed to answer any 10 questions only out of 15
questions given. In case if candidates attempts more than 10 questions, first 10 attempted questions only
considered for marking.
86. When 2.5g of a sample of Mohr’s salt reacts 1) 2035 kJ/mol 2) –2035 kJ/mol
N 3) 2167 kJ/mol 4) –2167 kJ/mol
completely with 50mL of KMnO4
10 90. If CuSO4.5H2O(s) ⇌ CuSO4.3H2O(s) + 2H2O(g)
solution. The % purity of the sample of
Mohr’s salt is Kp = 1.086 x 10-4 atm2 at 25°C. The
1) 78.4 2) 70 efflorescent nature of CuSO4.5H2O can be
noticed when the vapour pressure of H2O in
3) 37 4) 40
atmosphere is
87. Molecule AB has a bond length of 1.61 Å and
1) > 9.72 atm 2) < 7.92 atm
a dipole moment of 0.38 D. The fractional
charge on each atom (absolute magnitude) is 3) > 7.92 atm 4) < 11.92 atm
(e0 = 4.802 x 10-10 esu) 91. How many grams of CaC2O4 will dissolve in
1) 0.5 2) 0.05 1 litre of saturated solution. Ksp (CaC2O4) =
2.5 x 10-9 mol2/lit2
3) 0 4) 1.0
1) 0.0064g 2) 0.0128g
88. Which gas shows real behaviour?
3) 0.0032g 4) 1.28g
1) 16g O2 at 1 atm and 273K occupies 11.2L
92. aK2Cr2O7 + bKCl + cH2SO4 → xCrO2Cl2 +
2) 1g H2 in 0.5L flask exerts pressure of 24.63
atm at 300K yKHSO4 + zH2O
3) 1 mole NH3 at 300K and 1 atm occupies The above equation balances when
volume 22.4L 1) a = 2, b = 4, c = 6 and x = 2, y = 6, z = 3
4) 5.6L of CO2 at 1 atm and 273K is equal to 2) a = 4, b = 2, c = 6 and x = 6, y = 2, z = 3
11g 3) a = 6, b = 4, c = 2 and x = 6, y = 3, z = 2
89. Diborane is a potential rocket fuel which 4) a = 1, b = 4, c = 6 and x = 2, y = 6, z = 3
undergoes combustion according to the 93. The shape of transition state is
equation
B2H6(g) + 3O2(s) → B2O3(s) + 3H2O(g) Nu− + → [Transition state]
Calculate the enthalpy change for the
combustion of diborane given.
3 →
a) 2B(s) + O2(g) → B2O3(s) ∆H =-1273 kJ/mol
2
1 1) Triangular planar 2) Square pyramidal
b) H2(g) + O2(g) → H2O(l) ∆H =-286 kJ/mol
2 3) Trigonal bipyramidal 4) Tetrahedral
c) 2B(s) + 3H2(g) → B2H6(g) ∆H = 36 kJ/mol
31
94. Predict the product (A) of the following reaction. 98. Incorrect statement regarding following
reaction is

1) 2)
1) X is explosive
2) Y is an oxyacid of xenon
3) Both are examples of non-redox reaction
3) 4)
4) XeF6 can undergo partial hydrolysis
95. Henrys law constant K of CO2 in water at 99. The final product obtained in the reaction
25°C is 3 x 10-2 mol/L/atm. The mass of CO2
present in 100L of soft drink bottled with a
partial pressure of CO2 of 4 atm at the same
temperature is
1) 5.28 g 2) 12.0 g
3) 428 g 4) 528 g
96. A graph was plotted between molar conductivity D
of various electrolytes (NaCl, HCl and NH4OH)
and C (in mol/L). Correct set is
1) 2)

CH2Cl
Br

1) I : NaCl, II : HCl, III : NH4OH


3) 4)
2) I : HCl, II : NaCl, III : NH4OH
3) I : NH4OH, II : NaCl, III : HCl
4) I : NH4OH, II : HCl, III : NaCl CH2OD
97. ∆Gf vs T plot in the Ellingham diagram slopes 100. Arrange the following in the order of decreasing
downward for the reaction acidity.
1 CH2
1) Mg(s) + O2(g) → MgO(s)
2
1 A) B) C)
2) 2Ag(s) + O2(g) → Ag2O(s)
2
1 1) C > A > B 2) C > B > A
3) C(s) + O2(g) → CO(g)
2 3) A > C > B 4) A > B > C
1
4) CO(g) + O2(g) → CO2(g)
2
32
BOTANY
SECTION – A (35 questions)
(Answer all the questions)
101. Which one of the following taxonomic aid is 107. Which of the following is not a feature of
analytical in nature? Angiosperms?
1) Herbarium 2) Key 1) Reticulate venation
3) Museum 4) Catalogue 2) Double fertilization
102. Plant breeding is undertaken for improved 3) Absence of sieve tubes
nutritional quality of plants with the objective 4) Presence of tracheids and vessels
of improving all, except 108. Select the incorrect match from the following.
1) Protein content and quality 1) Adenosine = Sugar + Adenine
2) Oil content and quality 2) Guanosine = Sugar + Guanine
3) Cholesterol content 3) Cytidine = Sugar + Cytosine
4) Vitamin content 4) Thymine = Sugar + Thymidine
103. Match the following 109. Find the correct statement from the
Column - I Column – II following?
A) Flagellated i) Entamoeba 1) DNA and histone are positively charged
protozoans 2) DNA and histone are negatively charged
B) Ciliated ii) Trypanosoma 3) DNA is negatively charged and histones are
protozoans positively charged
C) Sporozoans iii) Paramoecium 4) DNA is positively charged and histones are
negatively charged
D) Amoeboid iv) Plasmodium
110. The number of genotypes and phenotypes are
protozoans vivax
possible from ABO blood grouping in human
1) A – iv, B – ii, C – i, D – iii beings respectively
2) A – iii, B – ii, C – iv, D – i 1) 3 and 6 2) 4 and 6
3) A – ii, B – iii, C – iv, D – i 3) 6 and 3 4) 6 and 4
4) A – iv, B – iii, C – ii, D – i 111. Which statement is incorrect about bacterial
104. An association of fungi with roots of higher transcription?
plants is known as 1) It is the process of copying genetic
1) Mycorrhizae 2) Corolloid roots information from DNA to RNA
3) Lichen 4) All the above 2) Adenine is complementary to uracil instead
105. In which of the following fungi, asexual of Thymine
spores are not found but vegetative 3) σ factor initiates transcription
reproduction by fragmentation is common? 4) It takes place in 3′ → 5′ direction
1) Agaricus 2) Neurospora 112. Interphase includes all of the following, except
3) Trichoderma 4) Claviceps 1) G1 phase 2) Anaphase
106. Juvenile prostrate gametophyte and adult leafy
3) S phase 4) G2 phase
gametophyte are found in
113. Synapsis occurs in
1) Marchantia 2) Funaria
1) Leptotene 2) Zygotene
3) Polytrichum 4) Both 2 and 3
3) Pachytene 4) Diplotene
33
114. Chose the correct combination 121. Correct option related to photorespiration is
1) Aspergillus niger – Citric acid 1) Oxygen binds with RuBisCO
2) Trichoderma polysporum – Butyric acid 2) Occurs more in C4 plants than in C3 plants
3) Streptococcus – Pectinase 3) Uptake of CO2 and the release of oxygen
4) Monascus purpureus – Lactic acid 4) RuBisCO acts as carboxylase
115. Presence of filiform apparatus is characteristic 122. The translocation of sucrose in phloem is
feature of explained by
1) Antipodals 2) Synergids 1) Root pressure
3) Egg cell 4) Central cell 2) Cohesion – Tension
116. If the nucellus cell of an angiosperm contains 3) Pressure flow hypothesis
24 chromosomes, then the number of 4) Transpiration pull
chromosomes present in generative cell,
123. The adsorption of water by seeds and dry
endosperm cell and an embryo will be
respectively wood is called
1) 24, 36, 24 2) 12, 36, 24 1) Osmosis
3) 12, 24, 36 4) 24, 12, 12 2) Imbibition
117. The end products of oxidative phosphorylation
3) Plasmolysis
are 4) Active transport
1) NADH and Energy 2) O2 and H2O only 124. In plant nutrition, elements are classified as

3) ADP, O2 and H2O 4) ATP and H2O macro and micronutrients depending on
118. Choose the correct option by identifying the 1) their availability in the soil
given image. 2) their relative concentration in ash
3) the relative amounts required by the plants
4) their relative importance in plant growth
125. Hydroponics is a technique of growing plants
in
1) Distilled water 2) Nutrient solution
3) Green house 4) Nutrient rich soil
126. Identify the mis-match
1) Anton Von Leeuwenhoek – discovered live
1) D– reduced osmotic potential cell
2) C - Selectable markers 2) Robert Brown – discovered nucleus
3) A and B – antibiotic resistance genes
3) Schleiden – Proposed omins cellula – e –
4) A and C – restriction endonuclease sites cellula
119. Heterophyllous aquatic plant of the following
4) Ramachandran – Triple helical structure of
is collagen
1) Larkspur 2) Cotton 127. Find the odd one w.r.t membrane bound cell
3) Buttercup 4) Both (1) and (3) organelles?
120. Most vital event in sexual reproduction is
1) Lysosomes 2) Mitochondria
1) Gametogenesis 2) Fertilization 3) Centrioles 4) Vacuoles
3) Embryogenesis 4) Parthenogenesis
34
128. Identify the incorrect statement w.r.t plasmid 1) A – I; B – II; C – IV; D – III
DNA 2) A – II; B – I; C – IV; D – III
1) It confers certain unique phenotypic 3) A – II; B – IV; C – I; D – III
characters to bacteria
4) A – II; B – I; C – III; D – IV
2) It may have genes coding for antibiotic
133. Which of the following statement(s) is/are
resistance
correct about venation?
3) It is used as a vector for insertion of
i) The arrangement of veins and the veinlets in
foreign DNA into the host
the lamina of leaf is called venation.
4) It can replicate by depending on the main
ii) Reticulate venation is the characteristic of
genome
monocots.
129. Number of carbon atoms excluding carboxyl
iii) When the veinlets form a network, the
carbon in palmitic acid and arachidonic acid is
venation is termed as reticulate venation.
respectively
1) 20 and 16 2) 16 and 20 iv) When the veins run parallel to each other
within a lamina, the venation is termed as
3) 19 and 15 4) 15 and 19
parallel venation.
130. Over 95 percent of all existing transgenic
animals are 1) Only (i) 2) Both (i) and (ii)
1) Cows 2) Fish 3) (i), (iii) and (iv) 4) All of these
3) Mice 4) Rabbit 134. Which of the following is a modified stem for

131. Hydrolysis of lactose gives rise to


the protection of plants from browsing
animals?
1) Glucose + Fructose
1) Tendrils 2) Thorns
2) Glucose + Glucose
3) Glucose + Galactose 3) Rhizome 4) Tuber
135. Which one of following helps in grasses to
4) Galactose + Ribose
132. Match the following correctly w.r.t regenerate parts removed by the grazing
aestivation in corolla. herbivores?
1) Apical meristems
Column – I Column-II
2) Lateral meristems
(Type of aestivation) (Examples)
3) Secondary meristems
A Valvate I Cotton
4) Intercalary meristems
B Twisted II Mustard
C Imbricate III Beans
D Vexillary IV Gulmohur
SECTION – B (15 questions)
Section – B consists of 15 questions. Candidates are allowed to answer any 10 questions only out of 15
questions given. In case if candidates attempts more than 10 questions, first 10 attempted questions only
considered for marking.
136. Which of the following is a living mechanical 137. The
enzyme recombinase involved in
tissue? 1) Transcription
1) Collenchyma 2) Sclerenchyma 2) Centriole duplication
3) Xylem 4) Phloem 3) Crossing over
4) Poleward movement of sister chromatids
35
138. The ratio between hydrogen bonds and 144. Parthenogenesis is the development of new
phosphodiester bonds broken in each individual from
palindromic sequence of DNA when treated 1) fertilized female gamete
with EcoRI during the formation of cohesive 2) unfertilized female gamete
ends is 3) male gamete
1) 1 : 1 2) 1 : 2
4) zygote
3) 4 : 1 4) 1 : 4 145. RNAi is mediated by
139. In one turn of citric acid cycle how many
1) ssDNA 2) dsRNA
oxidations and decarboxylation reactions takes
3) ssRNA 4) tRNA
place respectively
146. The main challenge for the production of
1) Four and Four
humulin is
2) Eight and Four
1) Creating disulphide bonds
3) Four and Six
2) To produce all insulin chains
4) Four and Two
3) Creating peptide bonds
140. Transfer of DNA of one bacterium to the other
4) Synthesizing it as a prohormone
bacterium via a virus is called
147. Pollen grains are well preserved as fossils
1) Transduction
because of
2) Transfection
1) Their architecture
3) Transformation
2) Sporopollenin in exine
4) Microinjection
3) Pectocellulose in intine
141. Statement – I : Ochoa’s enzyme is used to
4) Variability in viability
synthesize RNA with defined sequences in a
148. E.coli with completely radioactive DNA was
template independent manner.
Statement – II : Sickle cell anaemia is an allowed to replicate in non-radioactive
example for point mutation. medium for two generations. Percentage of
bacteria with radioactive DNA is
1) Both the statements are correct
1) 100% 2) 50%
2) Both the statements are incorrect
3) 25% 4) 12.5%
3) Statement – I is correct, statement – II is
149. Unisexuality in plants prevents
incorrect
4) Statement – I is incorrect, statement - II is 1) Autogamy and geitonogamy
correct 2) Autogamy but not geitonogamy
142. In india, biopatent rights are granted by 3) both geitonogamy and xenogamy
1) IARI 2) GEAC 4) geitonogamy but not xenogamy
3) EFB 4) NBRI 150. In C4 plants, CO2 fixation by PEP carboxylase

143. When pH (or) temperature is plotted against


occurs in
enzyme activity the shape of curve obtained is 1) stroma of bundle sheath chloroplast
1) Sigmoid curve 2) stroma of mesophyll chloroplast
2) Bell shaped curve 3) cytoplasm of mesophyll cells
3) Rectangular hyperbola 4) chloroplast of both mesophyll cells and
4) Linear curve bundle sheath cells
36
ZOOLOGY
SECTION – A (35 questions)
(Answer all the questions)
151. Sucking and circular mouth without jaws is 1) A - iv, B - ii, C - iii, D - i
seen in 2) A - ii, B - iv, C - i, D - iii
1) Lamprey 2) Pristis
3) A - iv, B - ii, C - i, D - iii
3) Lancelet 4) Salpa
4) A - ii, B - iii, C - i, D - iv
152. Which of the following belongs to the phylum
157. Matchthe following columns and identify the
of Aschelminthes?
1) Ascaris 2) Fasciola correct option.
3) Planaria 4) Taenia Column - I Column - II
153. Find the marine animal with stream lined body A. Columnar epithelium i) inner lining of
and cartilaginous endoskeleton. with microvilli bronchioles
1) Asterias 2) Ascidia
B. Cubodial epithelium ii) inner lining of
3) Labeo 4) Carcharodon
with microvilli intestine
154. Camelus and Elephas
C. Columnar epithelium iii) wall of salivary
1) have dry skin without any glands
with cilia glands
2) do not have external ears
3) are poikilothermous D. Multicellular
iv) lining of PCT
glandular epithelium
4) have milk producing glands
155. Assertion (A): The skin of mammals is 1) A - iii, B - iv, C - i, D - ii
aglandular. 2) A - i, B - ii, C - iv, D - iii
Reason (R): The skin of mammals is unique 3) A - iv, B - ii, C - i, D - iii
in possessing hair. 4) A - ii, B - iv, C - i, D - iii
1) Both A and R are true and R is the correct
158. Identify the correct statement w.r.t the given
explanation of A
diagram.
2) Both A and R are true and R is the not
correct explanation of A
3) A is true but R is false
4) A is false but R is true
156. Match the following columns w.r.t cockroach
and select the correct option.
Column - I Column - II
i) Junction of midgut 1) Cartilage present in the tip of the nose
A. Crop 2) Areolar tissue present beneath the skin
and hindgut
ii) Junction of foregut 3) Dense regular connective tissue of tendon
B. Hepatic caecae
and mid gut 4) Specialised loose connective tissue to store
C. Malpighian tubules iii) Hind gut fats
D. Colon iv) Storage of food
37
159. Find the correct equation from the following. 2) Eosinophils secrete histamine, serotonin,
1) Sucrase 
sucrose
→ Glucose + Fructose heparin, etc.,
3) Neutrophils are the most abundant cells of
2) Starch 
Salivaryamylase
→ Maltose
pH 6.8 the total formed elements
3) Proteins 
trypsin
→ Disaccharides 4) Eosinophils resist infections
4) Lactose lactase
→ Glucose + Glucose 163. Assertion (A): The circulatory system in
arthropods is open type.
160. Mark the incorrect statement.
Reason (R): In which blood pumped by the
1) The branching network of bronchi,
heart passes through vessels into open spaces
bronchioles and alveoli comprise the lungs
called sinuses.
2) Lungs are covered by double layered pleura
1) Both A and R are true and R is the correct
with pleural fluid between them
explanation of A
3) The outer pleural membrane is in close
2) Both A and R are true and R is the not
contact with the thoracic lining whereas
correct explanation of A
inner pleural membrane is in contact with
3) A is true but R is false
the alveolar surface
4) Both the A and R are false
4) Dome - shaped diaphragm is present on the
164. Presence of glucose and ketone bodies in urine
lower side of thoracic chamber
are indicative of
161. Identify A, B, C and D.
1) albunuria 2) diabetes mellitus
3) nephritis 4) both 1 and 3
165. Select the incorrect statement.
1) Small amounts of nitrogenous wastes could
be eliminated through saliva
2) In kidney failure patients, urea can be
removed by a process called hemodialysis
3) In hemodialysis, blood drained from a
convenient vein is pumped into dialyzing
unit after adding an anticoagulant like
heparin
4) During urine formation, the tubular cells
secrete substances like H+, K+ and
1) A - Systemic arteries, B - Pulmonary vein, ammonia into the filtrate
C - Systemic veins, D - Pulmonary artery 166. Isotropic band of a myofibril is bisected by
2) A - Systemic veins, B - Pulmonary vein, 1) elastic fibre
C - Systemic arteries, D - Pulmonary artery 2) thin fibrous membrane
3) A - Systemic arteries, B - Pulmonary artery, 3) thick fibrous membrane
C - Systemic veins, D - Pulmonary vein 4) collagen fibre
4) A - Pulmonary vein, B - Systemic arteries, 167. Fused vertebrae in human are
C - Systemic veins, D - Pulmonary artery a) cervical b) lumbar c) sacral d) coccygeal
162. Select the correct statement. 1) a and b 2) b and c
1) RBCs are devoid of nucleus in most of the 3) a and c 4) c and d
mammals and are biconvex in shape
38
168. Which of the following are controlled by 171. Choose the correct option for hormone.
limbic system? 1) non nutrient chemicals
a) sexual behaviour 2) act as intercellular messengers
b) emotional reaction 3) produced in trace amounts
c) body balance 4) all the above
d) motivation 172. Which among the following secretes relaxin?
e) respiration 1) Placenta
1) a, b, c, d and e 2) a, b, d 2) Ovary
3) a, b, c, d 4) a, b, d, e 3) Cervix
169. Which of the following statements are not 4) both 1 and 2
correct about choroid? 173. Choose the correct statements from the
A) It is the most vascular structure. following.
B) It is bluish in colour. 1) Placenta possess both endocrine and
C) It is thin over the anterior two thirds of the exocrine function
eye ball. 2) During pregnancy, cortisol, thyroxine, etc
D) It becomes thick in posterior part to form are increased several folds in foetal blood
the ciliary body. 3) Immediately after implantation, inner cell
1) A and B only mass differentiates into germ layers
2) B and C only 4) Inner cell mass have no potency to give rise
3) C and D only to tissues and organs
4) A, B and D 174. Match the column - I and column - II.

170. i) over secretion of GH in children leads to Column - I Column - II


gigantism i) foetus is fully
ii) low secretion of GH in children results in A. By the end of 8 weeks
developed
pituitary dwarfism
B. By the end of 12
iii) excess secretion of GH in adults ii) eye lids separate
weeks
(especially in middle age) results in
C. By the end of 24 iii) development of
Acromegaly
weeks limbs and digits
iv) An impairment affecting synthesis (or)
release of ADH results in condition diabetes iv) external genital
D. By the end of 36
inspidus. organs are well
weeks
developed
v) MSH acts on melanocytes and regulates
synthesis of melatonin. 1) A - iii, B - iv, C - ii, D - i
Study the above given statements. 2) A - iii, B - ii, C - iv, D - i
Choose the correct option related given 3) A - iv, B - i, C - ii, D - iii
statements. 4) A - iv, B - ii, C - i, D - iii
1) Except statement v all are correct 175. Statement-I: Intra uterine devices are inserted
2) Except statement ii and v all are correct by doctors or expert nurses in the vagina.
3) Except statement iv and v all are correct Statement - II: IUDs increases phagocytosis
4) all the given statements are correct of sperms within the uterus.
39
1) Both Statements I and II are correct 179. Readthe following statements, find out the
2) Statement I is correct, statement II is wrong statements.
incorrect A) In honey bees, workers are developed by
3) Statement I is incorrect, statement II is the unfertilized egg by means of
correct parthenogenesis.
B) Sex determination in honey bee is based
4) Both statements I and II are incorrect
on the number of sets of chromosomes.
176. Match the following.
C) In haplo diploid sex determination, the
Column - I Column - II male do not have grandfather and thus
A. HIV i) Virus have grand sons
B. Trichomonas ii) Protozoan 1) A and B 2) B and C
3) A and C 4) A, B and C
C. Candida albicans iii) Fungi
180. Match the following columns and choose the
D. Nisseria gonorrheae iv) Bacteria correct option.
1) A - ii, B - iii, C - iv, D - i Column - I Column - II
2) A - iii, B - iv, C - i, D - ii A. Stabilisation i) Other than mean
3) A - i, B - ii, C - iii, D - iv selection character value
4) A - iv, B - iii, C - ii, D - i B. Directional selection ii) Mean character valve
177. Statement-I:Erythroblastosis foetalis can be iii) Peripheral character
prevented by injecting the ‘anti-B’ antibodies C. Disruptive selection
valve
to the baby immediately after the delivery.
D. Founders effect iv) Genetic drift
Statement - II: In HDNB the affected baby
1) A - iv, B - ii, C - i, D - iii
suffers with haemolytic jaundice.
2) A - ii, B - i, C - iii, D - iv
1) Both Statements I and II are correct
3) A - iii, B - iv, C - i, D - ii
2) Statement I is correct, statement II is
incorrect 4) A - iii, B - i, C - ii, D - iv
181. Assertion (A): Hardy Weinberg principle says
3) Statement I is incorrect, statement II is
that the allelic frequencies in a population are
correct
stable and is constant from generation to
4) Both statements I and II are incorrect generation.
178. Which of the following statements are correct Reason (R): Change of frequency of alleles in
w.r.t honey bee? a population would then interpreted as
A) Males have half the number of resulting in evolution.
chromosomes than that of female. 1) Both A and R are true and R is the correct
B) Males produce sperms by meiosis explanation of A
C) Female produce gametes by mitosis 2) Both A and R are true and R is the not
D) Male are diploid having 16 chromosomes correct explanation of A
3) A is true but R is false
1) A and B 2) B and C
4) A is false but R is true
3) A only 4) A and D
40
182. Among the ancestors of Homo sapiens the 1) Chickengunya
cranial capacity was more than 900 cc in 2) Dengue
1) Homo habilis 3) Amoebiosis
2) Neanderthal man 4) Common cold
3) Homo erectus 185. Assertion (A): More and more children in
4) Dryopithecus metro cities of India suffer from asthma.
183. Which plasmodium species is the most serious Reason (R): Due to protected environment
and can even be fatal? provided in their early life.
1) P. falciparum 1) Both A and R are true and R is the correct
2) P. malaria explanation of A
3) P. vivax 2) Both A and R are true and R is the not
4) P. falcipatum and P. malaria correct explanation of A
184. Which of the following is not a vector borne
3) A is true but R is false
disease? 4) A is false but R is true
SECTION – B (15 questions)
Section – B consists of 15 questions. Candidates are allowed to answer any 10 questions only out of 15
questions given. In case if candidates attempts more than 10 questions, first 10 attempted questions only
considered for marking.
186. Mucosal associated lymphoid tissue 189. Matchthe following columns and select the
constitutes about _____ % of the lymphoid correct option.
tissue in human body.
Column - I Column - II
1) 50 2) 25
A. Inland water i) 30 - 35 ppm
3) 10 4) 5
187. The chronic use of drugs and alcohol causes B. Sea ii) 5 ppm
1) cirrhosis 2) anaemia C. Hypersaline
iii) more than 100 ppm
3) haemophilia 4) colour blindness lagoons
188. Read the following and choose the correct
iv) less than 100 ppm
w.r.t poultry.
1) A - ii, B - i, C - iii
i) It is the class of domesticated fowl like
geese. 2) A - ii, B - i, C - iv
ii) It is refer to the meat of domesticated 3) A - i, B - iii, C - ii
birds 4) A - i, B - ii, C - iv
iii) Proper feed and water and hygiene and 190. Statement-I: Any species growing
health care are not important components exponentially under unlimited resource
iv) Selection of disease free and suitable conditions can reach enormous population
breeds is important
densities in a short time.
1) except iii and iv
Statement - II: No population of any species
2) except i and ii
in nature has at its disposal unlimited
3) all except iii
resources to permit exponential growth.
4) all except i
41
1) Both Statements I and II are correct 196. Identify the wrong statement.
2) Statement I is correct, statement II is 1) Reforestation can occur naturally in a
incorrect deforested area
3) Statement I is incorrect, statement II is 2) Industrial waste waters are often rich in
correct toxic chemicals
4) Both statements I and II are incorrect 3) One of the major effects of deforestation is
191. What parameters are used for tiger census in reduced CO2 concentration in the atmosphere
our country’s national parks? 4) Deforestation causes loss of biodiversity
1) Pug marks only 197. Identify the incorrect statement.
2) Faecal pellets only 1) The greenhouse effect is a naturally
3) Actual head counting occurring phenomenon
4) Pug marks and faecal pellets 2) Present average temperature (15oC) of
192. Which among the following is man - made earths’ surface is due to greenhouse effect
ecosystem?
3) Clouds and gases reflect about one fifth of
1) Grassland 2) Lake the incoming solar radiation
3) River 4) Aquarium 4) Without greenhouse effect the average
193. The pyramid of biomass in sea is temperature at earth’s surface would have
1) upright 2) inverted been a chilly - 18oC
3) spindle 4) both 1 and 2 198. Which of the expensive metal is not found in
194. Statement-I:
It has taken millions of years of catalytic converters?
evolution to accumulate rich diversity in 1) Platinum 2) Palladium
nature.
3) Rhodium 4) Gold
Statement - II: Biodiversity and its
199. Increasing concentration of toxic substance
conservation are now vital environmental
due to passage to the next trophic level in a
issues of international concern.
food chain is known as
1) Both Statements I and II are correct
1) Eutrophication
2) Statement I is correct, statement II is
incorrect 2) Bioremediation
3) Statement I is incorrect, statement II is 3) Biomagnification
correct 4) both 2 and 3
4) Both statements I and II are incorrect 200. Use of Inorganic fertilizers and pesticides has
195. The colonization of tropical Islands by increased manifold in the wake of
humans led to the extinction of more than 1) white revolution
1) 200 species of native birds 2) green revolution
2) 20 species of native birds 3) blue revolution
3) 2000 species of native birds 4) red revolution
4) 1000 species of native birds
42

SPP – 3
Max. Marks: 720 Duration : 3 Hrs. 20 Min.
PHYSICS
SECTION – A (35 questions)
(Answer all the questions)

1. The energy of a system as a function of time


‘t’ is given by E = A2e-αt where α = 0.2 sec.
The measurement of A has an error of 1.25%. 3) 4)
If the error in measurement of time is 1.5%,
then the maximum possible error in E at t = 5
sec is 4. A particle starts from rest. its acceleration (a)
1) 4 2) 3 versus time ‘t’ is as shown in figure. The
maximum speed of the particle will be
3) 3.75 4) 1.125
2. Match the following 10
a (m/s2)
Column - I Column - II
(Physical quantity) (Unit) 11
time (sec)
i) Capacitance a) Tesla
1) 110 m/s 2) 55 m/s
ii) Magnetic b) Columb/volt 3) 550 m/s 4) 660 m/s
induction 5. A block of mass 2 kg rests on a rough inclined
2 -1
iii) Inductance c) Columb – joule plane making an angle 30o with horizontal.
d) Henry The coefficient of friction between block and
plane is 0.7. The frictional force on block is
e) N sec/C-m
1) 9.8 N 2) 0.7 x 9.8 3 N
1) (i) → b, c ; (ii) → a, e ; (iii) → d
3) 9.8 3 N 4) 0.7 x 9.8 N
2) (i) → c ; (ii) → c, e ; (iii) → d, b
6. A car is moving in a circular horizontal track
3) (i) → e ; (ii) → b, c ; (iii) → d, a of radius 10 m with a constant speed 10 m/s.
4) (i) → b, c ; (ii) → a ; (iii) → d, e A plumb bob is suspended from the roof of the
3. A projectile is projected with velocity ‘U’ at car by a light rigid rod. The angle made by rod
an angle ‘θ’ with horizontal. For a fixed θ with vertical is
which of the graph shown in the following 1) zero 2) 30o
figure shows the variation of range R versus 3) 45o 4) 60o
‘U’ 7. A wind powered generator converts wind
energy into electrical energy. Assume that the
generator converts a fixed fraction of wind
energy intercepted by the blades into electrical
1) 2) energy. For wind speed ‘v’, the electrical
power output will be proportional to
1) v 2) v2
3) v3 4) v4
43
8. A block of mass 0.18 kg is attached to a spring the surface of the planet. The escape velocity
of force constant 2 N/m. The coefficient of from the surface of planet is
friction between the block and the floor is 0.1. v
Initially the block is at rest and spring is 1) 2) 2 v
2
unstretched. An impulse is given to the block
v
and slides a distance of 0.06 m and comes to 3) 2 v 4)
rest for 1st time. The initial velocity of block is 2
13. If the radius of earth shrinks by 1 % without
changing its mass then the acceleration due to
gravity on the earth’s surface would
1) 0.4 m/s 2) 0.16 m/s 1) increases 2) remain unchanged
3) 16 m/s 4) 32 m/s 3) decreases 4) be zero
9. The output Y, when all three inputs are first
14. The below graph shows the extension (∆f) of a
high and then low, will respectively be
wire of length 1m suspended from the top of a
roof at one end with a load W connected to the
other end. If the cross-section area of wire is
1) 1, 0 2) 1, 1 10-6 m2, young’s modulus of material of wire
3) 0, 0 4) 0, 1 is
10. A mass M is moving with a constant velocity
parallel to x-axis. Its angular momentum with
respect to the origin is
1) zero
2) remains constant
3) goes on increasing 1) 2 x 1011 N/m 2) 2 x 10-11 N/m
4) goes on decreasing 3) 3 x 1012 N/m 4) 3 x 1011 N/m
11. A rod is rotating as well as translating on a
15. A conducting rod of length l is falling with a
smooth horizontal surface. The velocities of
velocity ‘v’ perpendicular to uniform
it’s ends at any moment are as shown. If
horizontal magnetic field B. The potential
θ = 30o, then the value of vB is difference between its two ends will be
1) 2 Blv2 2) Blv
1 2 1
3) Bl v 4) Blv 2
2 2
16. When an ideal diatomic gas is heated at
1) 2 m/s 2) 3 m/s constant pressure, the fraction of heat energy
2 supplied which increases internal energy of
3) 2 3 m/s 4) m/s gas is
3
2 3
12. A bullet fired vertically up with velocity ‘v’ 1) 2)
from the surface of a spherical planet. When it 5 5
reaches its maximum height, its acceleration 3 5
3) 4)
1 7 7
due to planet’s gravity is th of it’s value at
4
44
17. Which of the following graph correctly 1) 300 Hz 2) 400 Hz

represents the variation of β =


( dV
dp )
with P 22.
3) 600 Hz 4) 150 Hz
A bus is moving towards a huge wall with a
V velocity of 5 m/s. The driver sounds a horn of
for an ideal gas at constant temperature? frequency 200 Hz. The frequency of the beats
heard by passenger of the bus will be (speed of
sound in air= 342 m/s)
1) 2) 1) 6 beats/sec 2) 1 beats/sec
P P 3) 2 beats/sec 4) 3 beats/sec
23. When a solid metallic sphere is heated, the
largest percentage increase is in it’s
3) 4) 1) volume 2) surface area
3) radius 4) cross-section area
P P
24. Two positive ions, each carrying a charge ‘q’,
18. The intensity of radiation emitted by sun has are separated by a distance ‘d’. If F is the force
it’s maximum value at a wavelength of of repulsion between the ions, the number of
510 nm and that emitted by the north star has electrons missing from each ion will be (e is
the maximum value at 350 nm. If these starts charge on electron)
behave like black bodies the ratio of the
surface temperatures of sun and north start is 4πε0 Fd 2 4πε0 Fd 2
1) 2)
1) 1.46 2) 0.69 q2 e2
3) 1.21 4) 0.83 4πε 0 Fe 2 4πε 0 Fd 2
3) 4)
19. At room temperature, the rms speed of certain d2 e2
gas is found to be 1930 m/s. The gas is 25. As per the Bohr model, the minimum energy
1) H2 2) F2 (in eV) required to remove an electron from
3) O2 4) Cl2 the ground state of doubly ionized Li atom
20. Two masses m1 & m2 are suspended together (Z = 3) is
by a massless spring of spring constant ‘k’. 1) 1.51 2) 13.6
When the masses are in equilibrium, m1 is 3) 40.8 4) 122.4
removed without disturbing the system. The 26. A conducting sphere of radius R and carrying
angular frequency and amplitude of oscillation charge q is joined to a conducting sphere of
of m2 are radius 2R, and carrying a charge -2q the
k m1g k m2g charge flowing between them will be
1) , 2) ,
m2 k m1 k q 2q
1) 2)
3 3
k m 2g k (m1 + m 2 )g
3) , 4) , 4q
m2 k m2 k 3) q 4)
3
21. An object of mass m is hung from a steel wire. 27. All six capacitors shown are identical. Each
The fundamental frequency of transverse wave in can withstand maximum 200 V between its
it is 300 Hz. A mass of 3m is added to it. The new terminals. The maximum voltage that can be
fundamental frequency of the system is safely applied between A and B is
45
31. Consider a thin square sheet of side L and
thickness ‘t’, made of a material of resistivity
‘ρ’ the resistance between two opposite faces
shown by shaded areas in the figure is

1) 1200 V 2) 400 V
3) 600 V 4) 2400 V
28. The voltage of clouds is 4 x 106 V with respect
to ground. In lightening strike lasting
1) directly proportional to L
100 ms, a charge of 4C is delivered to the
ground. The power of lightening strike is 2) directly proportional to t
1) 160 MW 2) 80 MW 3) Independent of L
3) 20 MW 4) 500 kW 4) Independent of t

29. In the circuit P ≠ R, the reading of 32. If kinetic energy of a body is doubled, what
will be the ratio of final to initial de-Broglie
galvanometer is same with switch S opened or
wavelength?
closed then
1) 2:1 2) 1 : 2
3) 1 : 2 4) 2 : 1
33. Current is flowing in south direction a power
line. The direction of magnetic field above the
power line is (neglect earth’s magnetic field)
1) south 2) north
1) IR = IG 2) IP = IG
3) east 4) west
3) IQ = IG 4) IQ = IR
34. The magnetic field at a distance ‘x’ on the axis
30. At time t = 0, a battery of 10 V is connected
of a ring of radius ‘r’ from its centre is
across points A and B in the given circuit. If
proportional to
the capacitors have no charge initially, at what
time does the voltage across the becomes 4V? r2 r
1) 2)
(ln 5 = 1.6 and ln 3 = 1.1) (r 2 + x 2 )3/ 2 (r + x 2 )3/ 2
2

r r2
3) 4)
(r + x 2 )1/ 2
2
(r 2 + x 2 )1/2
35. At a certain place, the horizontal component
of earth’s magnetic field is 3 times the
1) 2 sec 2) 5 sec vertical component. The angle of dip at that
3) 1 sec 4) 10 sec place is
1) 60o 2) 45o
3) 90o 4) 30o
46
SECTION – B (15 questions)
Section – B consists of 15 questions. Candidates are allowed to answer any 10 questions only out of 15 questions
given. In case if candidates attempts more than 10 questions, first 10 attempted questions only considered for
marking.
36. If a current of 5 A in a coil of self inductance 1) air and placed in air
2 mH is cut off in time 0.1 sec, the induced 2) air and immersed in L1
emf in the coil is 3) L1 and immersed in L2
1) 0.1 V 2) 0.01 V 4) L2 and immersed in L1
3) 0.2 V 4) 0.02 V 40. A ray of light undergoes deviation of 30o
37. Two solid spheres and of equal volumes but when incident on an equilateral prism of
different densities are connected by a string. refractive index 2 . The angle made by the
They are fully immersed in a fluid of density ray inside the prism with base of the prism is
dF. They get arranged into an equilibrium state
1) zero 2) 30o
as shown in figure with some tension in string.
3) 45o 4) 60o
The arrangement is possible only if (density of
A & B are dA & dB) 41. A point source S is placed at the bottom of a
transparent block of height 10 mm and
refractive index 2.72. It is immersed in a lower
refractive index liquid as shown. It is found
that the light emerging from the block to the
liquid surface forms a circular bright spot of
1) dA < dF 2) dB > dF
radius 11.54 mm on the top of the block. The
3) dA > dF 4) dA + dB = 2dF refractive index of liquid (approximately) is
38. The graphs given below depict the dependence
of two impedances X1 and X2 on the
frequency of the alternating emf applied
individually to them. We can say that,

1) 3 2) 1.3
3) 1 4) 3.2
1
1) X1 is an inductor and X2 is a capacitor 42. In YDSE, intensity at a point is of the
4
2) X1 is an resistor and X2 is a capacitor maximum intensity. Angular position of this
3) X1 is an capacitor and X2 is a inductor point is (λ → wavelength of light, d →
4) X1 is an inductor and X2 is a resistor distance between 2 slits)
39. A hollow double concave lens is made of very λ  λ 
1) sin −1   2) sin −1  
thin transparent material. It can be filled with d  2d 
air or either of two liquids L1 or L2 having
 λ   λ 
refractive indices µ1 and µ2 (µ2 > µ1 > 1) the 3) sin −1   4) sin −1  
 3d   4d 
lens will diverge a parallel beam of light it is
filled with
47
43. Which of the following graph shows the 47. The decay constant of a radioactive sample is
dependence of intensity of transmitted light on λ. The half life and mean-life of the sample
the angle between polarizer and analyser? are respectively, given by
ln 2 1 1
1) and 2) ln 2λ and
λ λ λ
1) 2)
λ 1 1
3) and 4) 2 ln λ and
ln 2 λ λ
48. In the given circuit the current through battery
is
3) 4)

44. A conductor carrying constant current ‘i’ is


kept in the x-y plane in a uniform magnetic
field B . If F is the magnitude of the total
magnetic force acting on conductor, then
1) 0.5 A 2) 1 A
3) 1.5 A 4) 2 A
49. A ball of mass 0.2 kg rests on a vertical post
1) If B is along z-axis the F α (L + R) of height 5 m. A bullet of mass 0.01 kg,
2) If B is along x-axis then F α (L + R) travelling with a velocity v m/s in horizontal
direction, hits the centre of the ball. After
3) If B is along z-axis then F = 0
collision, the ball and bullet travel
4) If B is along y-axis then F = 0 independently the ball hits the ground at a
45. The work functions of silver and sodium are distance of 20 m and bullet at a distance of
4.6 and 2.3 eV respectively. The ratio of the 100 m from the foot of the post. The initial
slope of the stopping potential versus velocity of bullet is
frequency plot for silver to that of sodium is
1) 1 2) 1 : 3
1
3) 4) 2
2
46. Suppose the charge of a proton and electron
differ slightly. One of them is –e, the other is
1) 250 m/s 2) 250 2 m/s
(e+∆e). If the net of electrostatic and
3) 400 m/s 4) 500 m/s
gravitational force between 2 hydrogen atoms
placed at a distance d (much greater than 50. For a transistor the parameter β = 99, the value
atomic size) apart is zero, then ∆e is of the of parameter ‘α’ is
order of, (mass of hydrogen = 1.67 x 10-27 kg) 1) 0.9 2) 0.99
1) 10-20C 2) 10-23C 3) 1 4) 9
3) 10-37 C 4) 10-47C
48
CHEMISTRY
SECTION – A (35 questions)
(Answer all the questions)
51. The number of water molecules is maximum in 57. Which of the following conditions will always
1) 1.8 g of H2O lead to a non-spontaneous change?
2) 18 g of H2O 1) ∆H and ∆S both +ve
3) 18 mol of H2O 2) ∆H and –ve and ∆S is +ve
4) 18 molecules of H2O 3) ∆H and ∆S both -ve
52. Which of the following violates Aufbau 4) ∆H is +ve and ∆S is –ve
principle as well as Pauli’s exclusion 58. One mole of N2O4 in a 1L flask decomposes to
principle?
attain the equilibrium N2O4(g) ⇌ 2NO2(g). At
the equilibrium the mole fraction of N2O4 is
1) 1
. Then Kc will be
2
2) 1 1
1) 2)
3 2
3) 2
3) 4) 1
3
59. At 55oC autoprotolysis constant of water is
4) 4 x 10-14. If a given sample of water has a pH
53. The correct increasing order of 2nd ionization of 6.9, then the given water sample is
enthalpy is 1) acidic 2) basic
1) C < O < N < F 2) C < N < F < O 3) neutral 4) can’t be predicted
3) C < N < O < F 4) N < C < F < O 60. Consider the following two reactions
54. Which of the following pairs are isostructural? 2S2 O3−2 + I 2 → S4 O −6 2 + 2I −

1) XeF2, IF 2) NH3, BF3
2
S2 O3−2 + 4Br2 + 5H 2 O → 2SO 4−2 + 8Br − + 10H +
2− 2−
3) CO ,SO
3 3 4) PCl5, ICl5
Which of the following is correct?
55. Which of the following is correct order of 1) Br2 is a weak oxidizing agent than I2
stability of N2, N −2 , N +2 ?
2) S2 O3−2 undergoes reduction
1) N +2 > N 2− > N 2 2) N 2 > N 2− > N 2+ 3) The above reactions are not possible
+ − − +
3) N 2 > N > N
2 2 4) N > N > N 2
2 2
4) I2 is a weak oxidizing agent than Br2
56. Two flasks A and B have equal volumes A is 61. Which are correct statements about properties
maintained at 300K and B at 600K. A contains of hydrides?
H2 gas, B has an equal mass of CO2 gas. Find a) Saline hydrides react with water producing
the ratio of total KE of gases in flask A to that B. dihydrogen gas
1) 1 : 2 2) 11 : 1 b) Hydrides of O and F have lower boiling
3) 33 : 2 4) 55 : 7 points than the hydrides of their subsequent
group members
49
c) HF and H2O are examples for electron rich 66. CH3-CH=CH-Br 
NaNH 2
→ X →
873K
Y
Red hot
hydrides Iron tube

d) In group 6, only chromium forms CrH What is Y in the above reaction?


1) a and b only 2) a, c and d only
3) a, b and c only 4) b, c and d only 1)
62. Correct statement/s about alkali metals is/are
a) Alkali metals are strong oxidizing agents
b) Lithium is the hardest metal of IA group
c) The electropositive character increases down
the group with increase in atomic number 2)
d) Cs and K are used as electrodes in
photoelectric cells 3) CH3-CH2-CH2-NH2
1) a and b only 2) a and c only
3) a, c and d only 4) b, c and d only
63. State the given statements as true or false 4)
[T-true, F-false]
a) CCl4 does not undergo hydrolysis
67. Match the following:
b) Carbon shows maximum catenation
property among 14 group elements. List – I List - II
-
c) Graphite is used as a lubricant (A) NO3 ion in drinking (I) Damage of
d) Pb2+ is less stable than Pb4+ water greater than kidney and
1) a-T, b-T, c-T, d- T 2) a-F, b-F, c-F, d- T 50 ppm causes liver
3) a-T, b-T, c-T, d- F 4) a-T, b-T, c-F, d- F (B) SO42- ion greater than (II) Brown
NaNH 2
CH 3 − CH 2 − Br
500 ppm causes mottling of
64. HC ≡ CH → A 
(1mole)
ether
→B teeth
Tautomerism

dil H 2SO 4 /HgSO 4
→ C ↽ ⇀D (C) F- ion concentration (III) Laxative
(Major)
above 2ppm causes effect
Number of α-hydrogens is in compound D is
(D) > 50 ppb of Pb causes (IV) Blue baby
1) 2 2) 5
syndrome
3) 6 4) 4
65. Match the following correctly The correct match is
Detection/ Respective species (A) (B) (C) (D)
Estimation formed 1) I II III IV
A) Lassaigne’s test of N K) [Fe(CN)5NOS]-4 2) IV III II I
B) Lassaigne’s test of S L) NH3 3) III IV II I
C) Duma’s method M) Fe4[Fe(CN)6]3 4) IV III I II
D) Kjeldahl’s method N) N2 68. The correct statement regarding defects in
1) A-K, B-M, C-L, D-N crystalline solids is
2) A-M, B-K, C-L, D-N 1) Frenkel defect decreases the density of
3) A-K, B-M, C-N, D-L crystalline solids.
4) A-M, B-K, C-N, D-L
50
2) Frenkel defect is a non-stoichiometric 2) based on principle that the impurities are
defect less soluble in the melt than in the solid
3) Schottky defects have no effect on the state of the metal.
density of crystalline solids. 3) based on principle that the impurities are
4) Frenkel defect is a dislocation defect. equally soluble in the solid state as well as
69. For 0.1m aqueous glucose solution, at what in the melt of the metal.
temperature water will boil at 1.013 bar? 4) based on principle that the impurities are
(Boiling point of pure water is 373.15 K at more soluble in the melt than in the solid
1.013 bar ; Kb = 0.52 K kg mol-1) state of the metal.
1) 373.098 K 2) 373.202 K 74.
3) 373.15 K 4) 373.67 K
70. The weight of Mg (At. Wt = 24) displaced by The products of the above reaction are
a quantity of electricity which displaces 11.2L
of O2 at STP will be 1)
1) 1.2 g 2) 12 g
3) 2.4 g 4) 24 g
71. The conversion of molecules X to Y follows 2)
second order kinetics. If the concentration of
X is increased to three times, then rate of 3)
formation of one mole of Y is
1) increases by 3 times
2) decreased by 3 times
4)
3) increased by 9 times
4) decreased by 9 times
72. The following graph is obtained for the
adsorption of an ideal gas at charcoal surface,
75.
obeying Freundlich’s isotherm :
B and C are

1)

2)
x
Find the value of at P = 64 atm.
m
1) 4 2) 32 3)
3) 1 4) 8
73. The zone refining method is
4)
1) based on the principle that metal is
converted into its volatile compound and
then decomposed to give pure metal
51
76. The correct order of reactivity of following
carbonyl compounds towards nucleophilic 3)
addition reaction is
O O 4)
|| ||
H−C−H CH 3 − CH 2 − C − CH 3 79. The first ionization energy of O2 is
(I) (II)
1) very much lesser than first I.E of Xe
O
|| 2) very much higher than first I.E of Xe
CH 3 − CH 2 − C − H 3) Almost identical to first I.E of Xe
(III)
4) Two times of first I.E of Xe
1) IV > II > III > I 80. Choose the correct statements.
2) I > III > II > IV A) The stability of halides of 16th group
3) I > II > IV > III elements decreases in the order of
4) III > II > I > IV F- > Cl- > Br- > I-
77. The correct pKa order of the following B) Among tetrafluorides of 16th group, SF4 is
compounds is a gas, SeF4 is a liquid and TeF4 is a solid.
1) NO2-CH2-COOH < NC-CH2-COOH < C) Reducing property of dioxides of 16th
Cl-CH2-COOH < CH3-COOH
group decreases from SO2 to TeO2
D) Boiling point order of 16th group hydrides is
H2O > H2Te > H2Se > H2S
2)
1) A and B only 2) B, C and D only
3) A, B, C and D 4) A, C and D only
81. Incorrect among the following is
1) Ce(OH)3 > Gd(OH)3 > Lu(OH)3, basic
strength
3)
2) La+3, Ce+4, Yb+2, Lu+3 – Diamagnetic
3) La+3, Ce+3, Gd+3, Lu+3 – Colorless
4) KMnO4, K2Cr2O7, Cu2O – Colorless and
4) All the above
diamagnetic
82. Incorrect match among the following
78. Maximum Total
Complex AgCl ppt ions in
given water
Product B is
1) [Co(NH3)6]Cl3 3 moles 4
1) 2) [Co(NH3)5Cl]Cl2 2 moles 3
3) [Co(NH3)4Cl2]Cl 1 moles 2
2) 4) [Co(NH3)3Cl3] 0 moles 1
52
83. Which of the following reactions of glucose 2) Cross linked addition polymer of phenol
cannot be explained by its open chain and formaldehyde
structure? 3) Cross linked condensation polymer of
1) Glucose reacts with HCN forms phenol and formaldehyde
cyanohydrin 4) Linear condensation polymer of phenol and
2) Pentaacetate of glucose does not react with formaldehyde
hydroxyl amine 85. Polyethylene glycols are used in the
3) Glucose is oxidized by Br2 water to form preparation of which of the following
gluconic acid cleansing agents?
4) On prolonged heating with HI, glucose 1) Non-ionic detergents
forms n-hexane 2) Anionic detergents
84. Novolac is a 3) Cationic detergents
1) Linear condensation polymer of urea and 4) Both (2) and (3)
formaldehyde
SECTION – B (15 questions)
Section – B consists of 15 questions. Candidates are allowed to answer any 10 questions only out of 15
questions given. In case if candidates attempts more than 10 questions, first 10 attempted questions only
considered for marking.
86. Number of degenerate orbitals in 3rd shell of 89. Most stable carbocation among the following
H-atom ⊕
1) CH3 – C H 2 2)
1) 3 2) 8
3) 9 4) 10 O
|| ⊕ ⊕
87. Consider the following redox reaction in basic 3) CH 3 − C − CH 2 4) CH3 – C H – CH3
− −
medium N2H4(l) + ClO3(aq) → NO(g) + Cl(g) 90. The reaction quotient (Q) for the reaction
If X and Y are the number of moles of N2H4 2SO2(g) + O2(g) ⇌ 2SO3(g) is given by

and ClO in the balanced equation. Find X : Y
[SO3 ]
2
3
Q=
1) 2 :1 2) 3 : 4 [SO 2 ] [ O2 ]
2 1

3) 1 : 1 4) 2 : 3
The reaction will proceed from left to right if
88. An unsaturated hydrocarbon ‘X’ on reductive
1) Q = Kc 2) Q < Kc
ozonolysis gives two compounds. These two
3) Q > Kc 4) Q = 0
compounds do not give silver mirror with
91. If ‘a’ is the length of the side of a cube, the
Tollen’s reagent. The unsaturated hydrocarbon
distance between the face-centered atom and
‘X’ is one corner atom in the cube will be
1) CH3 – CH=CH2 2) 3 (a ) 2(a)
1) 2)
2 2
3) 4) a
3) 4) 2 (a )
2
53
92. The relationship between boiling points of 10g 1) Both Statements-I and II are correct
glucose (T1), 10g urea (T2), and 10g sucrose 2) Both Statements-I and II are wrong
(T3) which are dissolved in 250ml of water 3) Statement-I is correct, statement-II is wrong
separately is 4) Statement-I is wrong, statement-II is correct
1) T2 > T1 > T3 2) T2 > T3 > T1 98. Incorrect order among the following is
3) T1 > T2 > T3 4) T3 > T1 > T2
1) Mn < Cr (Second ionization enthalpy)
93. A first order reaction is 50% completed in 2) Sc < Ti < Cr < Mn (Number of variable
1.26 x 1014 s. The time required for 100% oxidation states)
completion of the reaction is
3) V < Cr < Mn < Fe+2 (Paramagnetic
+2 +2 +2
1) 1.26 x 1015 s 2) 2.52 x 1014 s behaviour)
3) 2.52 x 1028 s 4) Infinite +2 +2 +2 +2
4) Ni < Co < Fe < Mn (Ionic size)
94. In the electrochemical cell 99. Match the following.
Zn(s)/ZnSO4(aq)(0.01M)//CuSO4(aq)(0.01M)/Cu(s)
List – I List – II
E ocell = 1.1 V find out Ecell = ? (Complex) (Isomerism)
1) 1.16V 2) 1.10V A) [Ma5b] i) Both geometrical and
3) 1.04V 4) 1.32V optical isomerism
95. Phenol can be distinguished from ethanol by B) [Ma4b2] ii) Optical isomerism only
the reactions with C) [M(AA)3] iii) Geometrical isomerism
A) Br2 water only
B) Na D) [M(AB)3] iv) Neither geometrical nor
C) Neutral FeCl3 optical isomerism
1) A, B only 2) A, C only 1) A-iii, B-ii, C-i, D-iv
3) B, C only 4) A, B and C 2) A-iii, B-iv, C-i, D-ii
3) A-iv, B-iii, C-ii, D-i
96. Heptane-2,6-dione 
NaOH
100° C
→ ? (Major)
4) A-i, B-iii, C-iv, D-ii
O
100. Match the following
CH3 List – I List – II
1) 2) Amino acid Its Specification
A) Histidine I) Secondary amino acid
O B) Asparagine II) Contains –OH group
O
C) Proline III) Basic amino acid
C CH3
D) Serine IV) Contains -CONH2 group
3) 4) 1) A-I, B-II, C-III, D-IV
CH3 2) A-IV, B-III, C-II, D-I
CH3
3) A-III, B-IV, C-I, D-II
97. Statement-I : Phosphinic acid is mono protic
4) A-III, B-II, C-IV, D-I
acid while phosphonic acid is a diprotic acid.
Statement-II : All oxoacids contain atleast one
p=O unit and one p-OH group.
54
BOTANY
SECTION – A (35 questions)
(Answer all the questions)
101. According to ICBN, scientific name of Mango is 107. Hypogynous flowers are seen in
1) Mangifera indica Linn. 1) Sunflower 2) Guava
2) Mangifera Indica L. 3) Mustard 4) Pumpkin
3) Mangofera sativa Linn 108. DNA double helical model is proposed based
4) mangifera indica L. on X-ray diffraction data given by
102. Parbhani kranti is 1) Friedrich Meischer
1) Bhindi variety 2) Watson and Crick
2) Resistant to yellow mosaic virus 3) Wilkins and Franklin
3) Developed by mutation breeding 4) Erwin Chargaff
4) Both (1) and (2) 109. What happened when heat killed S strain
103. How many of the following organisms are bacterial cells along with live R strain cells
belongs to the class ascomycetes? were injected into the mice?
Aspergillus, Albugo, Alternaria, Morels, Toad 1) Mice survived and showed live S strain
stools, Puccinia, Claviceps, Penicillium bacterial cells
1) Three 2) Five 2) Mice died and showed live S strain
3) Four 4) Six bacterial cells
104. Which of the following statements is correct? 3) Mice survived and showed dead R strain
1) Cyanobacteria are heterotrophs bacterial cells
2) All eubacteria possess pili and fimbriae 4) Mice died and showed dead R strain
bacterial cells
3) Gas vacuoles are membranous
110. DNA replication in eukaryotes commences
4) Cholera causing bacteria are vibrios
105. Read the following statements and select the
1) From both ends of a chromosomes
correct one. simultaneously
1) Dmitri Ivanowsky recognized Tobacco 2) From several sites along the DNA of a
Mosaic Virus in 1892 chromosome simultaneously
2) M.W. Beijerinck called the infectious material 3) From centromere to either end
as “Contagium Vivum Fluidum” in 1935 4) From one end of chromosome to other
3) W.M. Stanley crystallized the TMV with 111. Reverse transcriptase is

DNA 1) RNA dependent RNA polymerase


4) Viruses are facultative parasites 2) DNA dependent RNA polymerase
106. Select the correct match from the following. 3) DNA dependent DNA polymerase
A (Plant) B(life cycle) 4) RNA dependent DNA polymerase
112. Which of the following beverages are
1) Chara Haplodiplontic
produced without distillation?
2) Volvox Diplontic
1) Wine and Whisky 2) Beer and Brandy
3) Ectocarpus Haplontic 3) Wine and Beer 4) Beer and Rum
4) Chlamydomonas Haplontic
55
113. The male gamete is “n” and egg is 3n. The 119. Sexual reproduction as compared to asexual
ploidy level in embryo and endosperm will be reproduction is a
1) 4n in embryo and 4n in endosperm 1) Short, complex and slow process
2) 4n in embryo and 7n in endosperm 2) Elaborate, complex and slow process
3) 4n in embryo and 6n in endosperm 3) Elaborate, complex and fast process
4) 6n in embryo and 12n in endosperm 4) Short, complex and fast process
114. What is the ratio of meiotic and mitotic 120. Photorespiration takes place in coordination of
divisions required to produce 8 male gametes 1) Chloroplast, Mitochondria and ER
from pollen mother cell respectively is
2) Mitochondria, Peroxisome and Golgi
1) 1 : 4 2) 2 : 3
3) Chloroplast, Peroxisome and Mitochondria
3) 1 : 8 4) 8 : 1
4) Chloroplast, Lysosome and Mitochondria
115. How many of the following structures are
121. Which of the following statements given
found in a typical monocot seed?
Plumule, Radicle, Root cap, Endosperm, below is incorrect?
Epiblast, Coleoptile 1) The C4 plants have high temperature
1) 6 2) 4 optimum.
3) 5 4) 2 2) Temperate plants have a low temperature
116. Identify A, B and C in the given reaction
optimum
3) C3 plants show high yielding in CO2
Pyruvic acid + Co.A + NAD+ B+C+ enriched conditions
NADH + H+ 4) Water stress causes the enhancement in the
CO2 availability
A B C
122. Statement I: Plasma membrane allows all
1) Acetyl coA PEP CO2 types of solutes and solvents into inner space
2) Pyruvate Acetyl ATP and vice versa.
dehydrogenase coA Statement II: Hydrophobic tails of
3) Pyruvate Acetyl CO2 phospholipids form a barrier to water and
dehydrogenase coA water-soluble ions.
1) Both the statements are correct
4) Acetyl coA Acetyl CO2
2) Both the statements are incorrect
dehydrogenase coA
3) Statement – I is correct, II is incorrect
117. On partial oxidation of glucose molecule, the
4) Statement – I is incorrect, II is correct
end products of alcoholic fermentation are
123. Which among the following processes occurs
1) 1 C2H5OH + 2ATP + 2CO2
against the concentration gradient?
2) 2 C2H5OH + 4 ATP + 2 CO2
1) Osmosis 2) Imbibition
3) 2 C2H5OH + 2 ATP + 2 CO2
3) Plasmolysis 4) Active transport
4) 4 C2H5OH + 4 ATP + 4 CO2
124. Which of the following expression describes
118. Arithmetic growth can be expressed as ____
and gives the _____ curve respectively nitrogen fixation?
1) Lt = Lo + rt, linear 1) N2+ 3H2 → 2NH3
2) w1 = wo + ert, sigmoid 2) 2NH4++ 2O2+ 8e– → N2+ 4H2O
3) Lt = Lo + rt, sigmoid 3) 2NH3 → N2+ 3H2
4) w1 = wo + ert, linear 4) 2N2+ glucose → 2 amino acids
56
125. Identify the odd one w.r.t criteria for IV) Store house molecules of energy
essentiality. 1) I and II 2) I, II and III
1) The element must be absolutely necessary for 3) II, III and IV 4) I, III and IV
supporting normal and reproductive growth 130. Malonate competes with the following to bind
2) The requirement of the element must be to the active site of the enzyme
specific and should not be replaced by any 1) Malate 2) Fumarate
other element 3) Succinate 4) Oxaloacetate
3) Deficiency of any element can be replaced 131. As per general rule of thumb, when
by supplying some other element
temperature is increased by 10°C the rate of
4) The element must be directly involved in reaction increases by
the metabolism
1) ten times 2) two times
126. The special membranous extensions which
3) three times 4) twenty times
contain pigments in cyanobacteria are called
132. Stem tendrils are found in
1) Chloroplasts 2) Mesosomes
1) Cucumber 2) Pumpkins
3) Chromatophores 4) Chromoplasts
3) Grapevines 4) All of these
127. Identify the incorrect statement
133. Which of the following is the green expanded
1) Fimbriae help the bacteria to attach to rocks
part of leaf with vein and veinlets?
in streams
1) Petiole 2) Midrib
2) Pili play a major role in motility
3) Rachis 4) Lamina
3) In prokaryotes reserve material is stored in
134. The venation pattern found in dicots and
the form of inclusion bodies
monocots respectively is
4) In prokaryotes, ribosomes are generally
associated with the plasmamembrane 1) Reticulate and parallel
128. Pigmented region of chloroplast is
2) Parallel and reticulate
1) Stroma 3) Reticulate and perpendicular
2) Inner membrane 4) Obliquely and parallel
135. Identify redifferentiated tissue from the
3) Thylakoid membrane
following.
4) Periplastidial space
1) Cork
129. Which of the following features are common
for starch and cellulose? 2) Shoot apical meristem
I) Homo polysaccharides 3) Phellogen
II) Polymer of glucose 4) Intervascular cambium
III) gives blue colour with iodine
SECTION – B (15 questions)
Section – B consists of 15 questions. Candidates are allowed to answer any 10 questions only out of 15
questions given. In case if candidates attempts more than 10 questions, first 10 attempted questions only
considered for marking.
136. The incorrect features w.r.t angiosperms is 137. The phase of the cell cycle where
1) Presence of tracheary elements chromosomal division occurs is
2) Presence of triple fusion 1) Prophase 2) Metaphase
3) Presence of naked megasporangia 3) Anaphase 4) Telophase
4) Presence of embryosac
57
138. Dissolutionof synaptonemal complex begins at 3) to uptake of DNA through transient pores in
1) Metaphase – I 2) Zygotene the bacterial cell wall
3) Anaphase – I 4) Diplotene 4) to express antibiotic resistance genes
139. Match the columns correctly. 144. Bacillus thuringiensis is widely used in

Column – I Column – II contemporary biology as


1) Indicator of water pollution
A) Elution I) Cloning vector
2) Agent for the production of dairy products
B) Ampicillin II) Restriction
3) Source of industrial enzyme
resistant gene digestion
4) Bio-insecticide and Bio-pesticide
C) Ti plasmid III) Selectable
145. Which one of the following statements is
marker
incorrect w.r.t genetically engineered insulin?
D) Molecular IV) Obtaining 1) E.coli is used for producing humulin
scissor DNA from
2) Chains A and B were produced separately
agarose gel
3) Eli Lilly company prepared it for the first
1) A – III, B – IV, C – I D – II time
2) A – IV, B – III, C – I D – II 4) Genetically engineered insulin has
3) A – II, B – I, C – IV, D – III carbohydrates
4) A – III, B – II, C – I, D – IV 146. The number of chromosomes found in the
140. According to ‘IUBMB’ classification, all endosperm of maize is
nucleases belongs to 1) 20 2) 30
1) Class I 2) Class II 3) 40 4) 15
3) Class III 4) Class IV 147. Asymmetric spindle formation is seen during
141. If recognition sequence of an enzyme has 1) Development of microspore
4 bp, how many recognition sequences could 2) Development of embryosac
you found in a DNA with 20 kb?
3) Development of male gametophyte
1) 78 2) 39
4) Development of female gametophyte
3) 38 4) 48
148. The microbe that helps in the production of
142. Which of the following is not required in the “Swiss cheese” is
construction of rDNA?
1) Aspergillus niger
1) Molecular scissors
2) Propionibacterium sharmanii
2) DNA fragments
3) Saccharomyces cerevisiae
3) Molecular glue
4) Trichoderma polysporum
4) DNase
149. High aspartic acid, low nitrogen and sugar
143. What is the significance of “heat shock” content in maize is resistance to
method in a bacterial transformation
1) Fruit borers 2) Root borers
technique?
3) Stem borers 4) All of the above
1) to facilitate the binding of DNA to the cell
150. Glomus is related to
wall
1) Lichens 2) Mycorrhizae
2) to uptake of RNA through membrane
transport protein only 3) Bacculoviruses 4) Nematode
58

ZOOLOGY
SECTION – A (35 questions)
(Answer all the questions)
151. Observe the given diagrams and select the 154. Match the following columns and choose the
suitable option for them. correct option from the following.
Column - I Column - II
A. Presence of beak i) Reptilia
A) B) B. Crawling mode of
ii) Mammalia
locomotion
C. Can live in aquatic and
iii) Pisces
terrestrial habitats
C) D) D. Young ones are
iv) Amphibians
nourished by milk
E. First jawed vertebrates v) Aves
1) D is amphibian and has eye lids
1) A - iii, B - ii, C - iv, D - v, E - i
2) B is viviparous and homiothermous
2) A - i, B - iv, C - v, D - iii, E - ii
3) A has external ears and is poisonous 3) A - ii, B - iv, C - v, D - iii, E - i
4) C has wings and bisexual 4) A - v, B - i, C - iv, D - ii, E - iii
152. Find out the mammalian characters 155. Assertion (A): Panthera tigris and Panthera

1) Mammary glands, hairy skin, pinnae leo belong to class Mammalia.


Reason (R): They both are predaceous
2) Hairy skin, viviparity, absence of eye lids
animals.
3) Single ventricle, mammary glands, post 1) Both A and R are true and R is the correct
anal tail explanation of A
4) Poikilothermous, gills, operculum 2) Both A and R are true and R is not the
153. Identify A and B based on below given correct explanation of A
characters. 3) A is true but R is false
4) A is false but R is true
A B
156. Assertion (A): When one cardiac muscle cell
i. Ventral nerve cord i. Dorsal nerve cord receives a signal to contract, its neighbour
cells are also stimulated to contract.
ii. Dorsal heart ii. Ventral heart
Reason (R): Communication junctions at
iii. Tracheary system iii. Gills some fusion points of cardiac muscle allow the
1) A - Cockroach, B - Locusta cells to contract as a unit.
1) Both A and R are true and R is the correct
2) A - Limulus, B - Pila
explanation of A
3) A - Petromyzon, B - Butterfly 2) Both A and R are true and R is the not
4) A - Culex, B - Exocoetus correct explanation of A
3) A is true but R is false
4) A is false but R is true
59
157. Assertion (A): Cockroach head is triangular 161. Match the column I with column II.
formed by fusion of six segments. Column - I Column - II
Reason (R): In cockroach, head lies anteriorly
A. Alveoli (O2) i) 40 mm Hg
at right angles to the longitudinal body axis.
1) Both A and R are true and R is the correct B. Deoxygenated blood (O2) ii) 104 mm Hg
explanation of A C. Oxygenated blood (CO2) iii) 95 mm Hg
2) Both A and R are true and R is the not
D. Tissues (CO2) iv) 45 mm Hg
correct explanation of A
3) A is true but R is false 1) A - i, B - ii, C - iii, D - iv
4) A is false but R is true 2) A - iii, B - iv, C - ii, D - iii
158. Assertion (A): Gizzard in cockroach acts as 3) A - i, B - iii, C - iv, D - ii
grinding mill. 4) A - ii, B - i, C - i, D - iv
Reason (R): Proventriculus of cockroach has 162. Identify the A, B, C and D in the given figure.
outer layer of thick circular muscles and thick
inner cuticle forming six highly chitinous
plates called teeth.
1) Both A and R are true and R is the correct
explanation of A
2) Both A and R are true and R is the not
correct explanation of A
3) A is true but R is false
4) A is false but R is true
159. The simple substances of digested food are
absorbed in
1) ileum 2) jejunum
3) colon 4) both 1 and 2
160. Mark the incorrect statement regarding the
1) A - Interventricular septum, B - Bundle of
given diagram.
His, C - Pulmonary veins, D - Chordae
tendinae
2) A - Interventricular septum, B - Bundle of
His, C - Pulmonary veins, D - Apex
3) A - Chordae tendinae, B - Bundle of His, C
- Pulmonary arteries, D - Apex
4) A - Left ventricle, B - Bundle of His,
C - Pulmonary veins, D - Chordae tendinae
163. Statement-I: Lymph is a colourless fluid
containing specialized lymphocytes which are
1) Volume of the thorax decreased responsible for the immune responses of the
2) Ribs and sternum returned to original body.
position Statement - II: Lymphatic system collects the
3) Diaphragm contracted and arched upwards tissue fluid and drains it back to the major
4) Air expelled from lungs veins.
60
1) Both Statements I and II are correct 169. Corpora quadrigemina is a part of
2) Statement I is correct, statement II is 1) cerebrum
incorrect 2) medulla oblongata
3) Statement I is incorrect, statement II is 3) mid brain
correct 4) cerebellum
4) Both statements I and II are incorrect 170. How many of the below functions are related
164. Which of the following are uricotelic? to hormone testosterone?
1) Land snails 2) Reptiles i) Spermatogenesis
3) Birds 4) All the above ii) Development of male sex accessory organs
165. A comparison of the volume of the filtrate iii) Libido
formed per day is iv) Anabolic (synthetic) affects on protein and
1) 180 mL 2) 180 L carbohydrate metabolism
3) 99 L 4) 125 mL 1) three only 2) four only
166. Statement-I: Each myofibril has alternate 3) five only 4) all the above
dark and light bands on it. 171. Mark the incorrect option for complex
Statement - II: Actin and myosin proteins are disorder diabetes mellitus.
arranged as rod like structures, parallel to each 1) Prolonged hypoglycemia could be the
other and also to the longitudinal axis of the reason
myofibrils. 2) Associated with loss of glucose through
1) Both Statements I and II are correct urine and formation of harmful compounds
2) Statement I is correct, statement II is known as ketone bodies
incorrect 3) Diabetic patients are successfully treated
3) Statement I is incorrect, statement II is with insulin therapy
correct 4) Insulin deficiency and/or insulin resistance
4) Both statements I and II are incorrect can result in this disease
167. Name the specialized tissue with slightly 172. Statement-I: Secretions of male accessory
pliable matrix due to chondroitin salts glands constitute the seminal plasma which is
1) ligament 2) bone rich in fructose, calcium and with out
3) adipose tissue 4) cartilage enzymes.
168. Which of the following events that occur
Statement - II: The secretions of
during depolarization phase of an action bulbourethral glands helps in the lubrication of
potential? penis.
1) K+ diffuses into the axon - the inside of the 1) Both Statements I and II are correct
axon become positive 2) Statement I is correct, statement II is
2) K+ diffuses out of the axon - the inside of incorrect
the axon become negative 3) Statement I is incorrect, statement II is
3) Na+ diffuses into the axon - the inside of the correct
axon become positive 4) Both statements I and II are incorrect
4) Na+ diffuses into the axon - the inside of
the axon become negative
61
173. Identifythe odd structure with reference to Haemophilia, Down’s syndrome,
human male reproductive system, from the Phenylketonuria, Klinefelter’s syndrome,
following Myotonic dystrophy, Cystic fibrosis
1) vasa efferentia 2) isthmus 1) six 2) four
3) vas deferens 4) rete testis 3) five 4) all
174. Approximately, how many primary follicles 178. A monosomic (2n - 1) abnormality in human
are left in each ovary at the time of puberty in is
a human ♀? 1) Klinefelter’s syndrome
1) 1,20,000 - 1,60,000 2) 30,000 - 40,000 2) Down’s syndrome
3) 40,000 - 60,000 4) 60,000 - 80,000 3) Turner’s syndrome
175. Match the following columns and choose the 4) Edward’s syndrome
correct option.
179. Large population in a given area in constant
Column - I Column - II genetic equilibrium may be disturbed by
A. Lippes loop i) Hormonal IUD’s 1) absence of gene flow
B. Cu-T ii) Non medicated IUD’s 2) absence of natural selection
C. LNG - 20 iii) Oral contraceptives 3) absence of random mating
4) lack of mutations
D. Pills iv) Cu releasing IUDs
180. Pre historic cave art developed about
1) A - iii, B - ii, C - iv, D - i
1) 1800 ya 2) 18000 ya
2) A - ii, B - iv, C - i, D - iii
3) 1.5 mya 4) 2 mya
3) A - ii, B - i, C - iv, D - iii
181. Which one of the following used hides to
4) A - iv, B - iii, C - i, D - ii protect their body and buried their dead?
176. Statement-I: In grasshopper males have one 1) Homo erectus
allosome but females have a pair of allosomes.
2) Australopithecines
Statement - II: In XX - XY method of sex
3) Neanderthal man
determination both male and female have
4) Homo habilis
equal number of allosomes.
182. Statement-I: Infectious diseases are very
1) Both Statements I and II are correct
common and every one of us suffers from
2) Statement I is correct, statement II is
these at sometime or other.
incorrect
Statement - II: All infectious diseases are
3) Statement I is incorrect, statement II is
lethal.
correct
1) Both Statements I and II are correct
4) Both statements I and II are incorrect
2) Statement I is correct, statement II is
177. How many of the following diseases can be
incorrect
studied with the help of pedigree chart?
3) Statement I is incorrect, statement II is
Turner’s syndrome, Sickle cell anemia, correct
4) Both statements I and II are incorrect
62
183. Fertilization event of plasmodium takes place 185. Assertion (A): Consumption of egg and
in ____ chicken drastically affected in India due to
1) mosquitoes gut 2) human’s gut Bird flu virus.
3) humans liver cells 4) mosquitos blood Reason (R): Bird flu virus created a scare in
184. Assertion(A): Rheumatoid arthritis affects the country.
many people in our society. 1) Both A and R are true and R is the correct
Reason (R): It is an autoimmune disease. explanation of A
2) Both A and R are true and R is the not
1) Both A and R are true and R is the correct
correct explanation of A
explanation of A
3) A is true but R is false
2) Both A and R are true and R is the not
correct explanation of A 4) A is false but R is true
3) A is true but R is false
4) A is false but R is true
SECTION – B (15 questions)
Section – B consists of 15 questions. Candidates are allowed to answer any 10 questions only out of 15
questions given. In case if candidates attempts more than 10 questions, first 10 attempted questions only
considered for marking.
186. Identify the A, B and C in the given figure. 188. Choose the correct combination.
Interaction Species A Species B
1) Amensalism - O
2) Mutualism + -
3) Commensalism + +
4) Competition - +
189. Percent of PAR captured by plants is
1) A - Partial regulators, B - Regulators, 1) 2 - 10 2) 10 - 30
C - Conformers 3) 30 - 50 4) 50 - 70
2) A - Regulators, B - Conformers,
190. Study the following statements w.r.t
C - Partial regulators
ecological succession and choose incorrect
3) A - Conformers, B - Regulators, statements.
C - Partial regulators
i) The gradual and fairly predictable change
4) A - Partial regulators, B - Conformers,
in the species composition of a given area
C - Regulators
is called ecological succession.
187. Which one of the following found in the
ii) The individual transitional communities
deepest waters?
are termed seral stages.
1) Green algae
iii) Secondary succession is slower than
2) Brown algae
primary succession.
3) Red algae
iv) Succession and evolution would have
4) Blue algae
been parallel processes
63
1) i only 2) ii only 196. Which of the following determine to a large
3) iii only 4) iv only extent, the vegetation in any area?
191. Assertion (A): The number of fungi species in 1) Mineral composition 2) Topography
the world is more than combined species 3) Soil composition 4) All the above
richness of all the vertebrates. 197. Which one of the following statement/s is/are
Reason (R): Algae, fungi, bryophytes and not correct?
gymnosperms comprise no more than 22% of i) A type of loose connective tissue that is
total species. specialized for fat storage is areolar tissue
1) Both A and R are true and R is the correct ii) Tendons are responsible for the attachment
explanation of A of one bone to another
2) Both A and R are true and R is the not iii) Ligments are responsible for the
correct explanation of A attachment of muscles with bones.
3) A is true but R is false iv) Dense irregular connective tissue is seen in
4) Both A and R are false the blood.
192. _______ species of plants contribute to the 1) i and ii only 2) ii and iii only
traditional medicines used by native people 3) iii and iv only 4) i, ii, iii and iv
around the world. 198. Hepatitis - B and HIV cannot be transmitted
1) 2500 2) 3000 by
3) 25,000 4) 30,000 1) sharing of injection needles with infected
193. Soil pollution primarily results from persons
1) Industrial chemicals 2) infected mother to foetus
2) Hospital chemicals 3) sharing of utensils
3) Municipal wastes 4) blood transfusions from an infected person
4) Agricultural chemicals (pesticides) 199. Which is not the impact of deforestation?
194. A non endocrine gland hormone which is 1) Soil erosion
released by atrial wall reduces blood pressure 2) Desertification
by causing dilation of blood vessels is 3) Enhanced CO2 concentration in atmosphere
1) histamine 4) Increased ozone concentration in stratosphere
2) atrial natriuretic factor 200. Personwho is AB- cannot receive blood from
3) adrenalin I due to II in the donor.
4) costisol 1) I - AB-, II - absence of Rh antigen
195. All of the following bones are of axial 2) I - O-, II - absence of Rh antigen
skeleton except. 3) I - A-, II - presence of A antigen
1) sternum 2) scapula 4) I - AB+, II - presence of Rh antigen
3) hyoid 4) incus
64

SPP – 4
Max. Marks: 720 Duration : 3 Hrs. 20 Min.
PHYSICS
SECTION – A (35 questions)
(Answer all the questions)

1. A rectangular plate has length (2 ± 0.2) cm and


width (1 ± 0.01)cm. The maximum percentage
error in the measurement of its area is
1) 1 % 2) 11 %
3) 20 % 4) 3 % 1) 200 N 2) 100 N
2. The kilowatt hour is a unit of 3) 170 N 4) 140 N
1) electric power 2) electric charge 6. A sphere is move off the edge of a horizontal
table at a speed of 4 m/s. It hits the ground
3) energy 4) force
after 0.4 sec. Which statement given below is
3. A particle starts from rest, its acceleration true (g = 10 m/s2)
versus time is as shown in the figure. The
1) It hits the ground at a horizontal distance
maximum speed of the particle will be
1.6 m from the edge of the table
2) The speed with which it hits the ground is
4 m/s
3) Height of the table is 1.6 m
4) It hits the ground at an angle of 60o to the
horizontal
1) 5 m/s 2) 20 m/s 7. Two masses P and Q of 20 kg, 10 kg
respectively are connected with a string
3) 25 m/s 4) 50 m/s
passing over a frictional pulley fixed at the
4. A particle is acted upon by a force of constant corner of a table as shown. The coefficient of
magnitude which is always perpendicular to static friction of with table is 0.4. The
the velocity of the particle. The motion of the minimum mass of R that may be placed on to
particle takes place in a plane. It follows that prevent it from moving is
1) velocity is constant
2) acceleration constant
3) kinetic energy constant
4) momentum constant
5. A solid sphere of mass 10 kg is resting inside 1) 10 kg 2) 15 kg
a cube as shown in the figure. The cube is
3) 20 kg 4) 5 kg
moving with a velocity v = 10 tiˆ + 4tjˆ m/s. Here
8. A position dependent force F=(2x + 3x2 +5) N
‘t’ is the time in second. All surfaces are acts on a small body of mass 4 kg and
smooth the sphere is at rest with respect to the displaces from x = 0 to x = 4m.The work done
cube. What is the total force exerted by the in joules is
sphere on the cube (g = 10 m/s2) (nearly)
65
1) 10 J 2) 100 J 13. Gravitational potential at the centre of the
3) 40 J 4) 90 J earth is V, then find the gravitational potential
9. A particle having a charge of 20 µC and a on the surface of the earth is
mass of 1.0 mg is projected perpendicularly 3V
1) V 2)
into a uniform magnetic field of 0.1 T, with a 2
speed of 100 ms-1. Time taken by the particle 2V V
to come back to its original position for the 3) 4)
3 2
first time after being projected is
14. For a perfectly rigid body
1) 2.45 s 2) 3.14 s
1)Young’s modulus is infinity and bulk
3) 1.57 s 4) 1.23 s
modulus is zero
10. A uniform rod of 10 kg is hanging in
2) Young’s modulus and bulk modulus are
horizontal position with the help of two
threads. It is also supports a 50 kg mass as infinity
shown in the figure. Find the tension 3) Young’s modulus and bulk modulus are
developed in each thread. zero
4) Young’s modulus is zero and bulk modulus
is infinity
15. A cube of ice of side length 20 cm is floating
A B in water of density 1000 kg/m3 then pickup the
l/4 correct statement (density of ice = 900 kg/m3)
50 kg 1) 2 cm of ice cube will be in water
2) 18 cm of ice cube will be in water
1) TA = 225 N 2) TA = 375 N
3) 18 cm of ice cube will be out of water
3) TB = 375 N 4) TB = 225 N
4) 1 cm of ice cube will be out of water
11. Find the coordinates of centre of mass of a
16. Three identical rods of same material are
non-uniform rod length L where linear mass
joined to form an equilateral triangle. The
density λ varies as λ = α + βx. Where ‘x’ is
temperature of end A and B is maintained
the distance from the lighter end.
T
L(3α + 2β L) L(3α + 2β L) constant as 4T and T. The ratio of C will be
1) 2) TB
3(2α + β L) 6(2α + β L)
(Assuming no loss of heat from surfaces)
L(2β + 3αL) L(2α + 3β L)
3) 4)
3(2β + αL) 3(2β + αL)
12. Three identical masses 10 m each are placed at
the coordinates (0, 0), (4, 0) and (2, 2 3) ,
then the force on the mass placed at (0, 0) is
 5 1
Gm 2   25Gm 2  1) 2)
1)  3  2) 3  2 2
 4   4 
1 2
 25Gm 2   Gm 2  3) 4)
3)   4) 3  3 3
 4   16 
66
17. Two spheres made of same substance have 22. A simple pendulum of length l is oscillation
diameters in the ratio 3 : 2. Their thermal with a time period T. Match the columns
capacities are in the ratio of Column – 1 Column - 2
1) 1 : 2 2) 1 : 1 A) Time period if the p) More than
3) 9 : 4 4) 27 : 8 pendulum is oscillated T
18. The pressure and volume of a gas are changed inside liquid
as shown in the P-V diagram in the figure. The B) Time period if a q) equal to T
temperature of the gas constant force less
than or equal to
weight of bob is
applied on the bob in
vertically upward
direction
1) increases as it goes from B to C
C) Time period if the r) less than T
2) decreases as it goes from A to B pendulum is
3) decreases as it goes from C to D oscillated in a upward
4) remains constant during these changes moving lift with
19. Heat is supplied at constant pressure to constant velocity
monoatomic gas. The part of this heat which D) Time period if the s) infinite
goes to increase its internal energy will be pendulum is
1) 0.6 2) 0.4 suspended in a
3) 0.5 4) 1 satellite
20. The potential difference between A and B in t) zero
the circuit shown is (assume diode to be ideal) 1) A (p, s) ; B(p, s) ; C(q); D(s)
2) A (p) ; B(s) ; C(q); D(r)
3) A (r) ; B(p, s) ; C(s); D(q)
4) A (p, s) ; B(p, s) ; C(q); D(r)
23. A string passing over a smooth pulley carriers
1) 7.5 V 2) 15 V a stone at one and while its other end is
3) 5 V 4) 10 V attached to a vibrating tunning fork and the
string vibrates forming 8 loops. When the
21. The mass of 10 kg shown in the figure
stone is immersed in water 10 loops are
oscillates in simple harmonic motion with
formed. The specific gravity of the stone is
amplitude 3 cm. The amplitude of the point P
nearly
is
1) 2.78 2) 4.78
3) 1.78 4) 3.78
24. Two infinitely long thin straight wires having
1) 3 cm 2) 2 cm uniform linear charge densities λ and 2λ are
arranged parallel to each other at a distance r
3) 1 cm 4) 4 cm
apart. The intensity of the electric field at a
point midway between them is
67
2λ λ 29. In the steady state, the charge on the capacitor is
1) 2)
πε 0 r πε 0 r
λ 3λ
3) 4)
2πε0 r 2πε0 r
25. In the adjoining figure the potential difference
between x and y is 60 V. The potential
difference between the points M and N will be

Cε 2Cε
1) 2)
5 5
3Cε 4Cε
3) 4)
1) 10 V 2) 15 V 5 5
3) 20 V 4) 30 V 30. For a battery the shown graph is plotted
26. Two thin rings, each having radius 3R are between terminal potential difference V across
placed at distance 4R apart with their axis the battery and the current drawn from it. The
coinciding. The charges on the rings are +Q maximum power that can be produced in the
and –Q. The potential difference between the battery is
 1 
centres of two rings is  K = 
 4πε0 
15 KQ 4 KQ
1) 2)
16 R 15 R
8 KQ 2 KQ
3) 4)
5 R 5 R
27. A cylindrical capacitor has two co-axial
cylinder of length 20 cm and radii 1.8 cm and 1) 6 W 2) 9 W
1.82 cm. The outer cylinder is earthed. The net 3) 12 W 4) 8 W
capacitance of the system is 31. An AC current is given as
1) 1 nF 2) 2 nF i = (2 cos ωt + 3 sin ωt)A. The rms current is
3) 10 nF 4) 20 nF
13
28. Consider the network shown in the figure. The 1) A 2) 13 A
2
current through the 2 Ω resistor is 6A. Then
current ‘i’ is 13 5
3) A 4) A
2 2
32. A magnetic needle is positioned in stable
equilibrium in a uniform magnetic field. An
external agent does W work to rotate it
through 60o. The torque required to maintain
the needle in this position is
1) 2 W 2) W
1) 24 A 2) 28 A 3
3) 12 A 4) 18 A 3) W 3 4) W
2
68
2
33. A circular coil of area 0.2 m , 250 turns and 1) 0.5 Ω 2) 5 Ω
resistance 5 Ω is placed in magnetic field. The 3) 15 Ω 4) 25 Ω
magnetic field is directed perpendicular to the 35. A coaxial circular coil having N turns, radius
plane of coil and it is increased from 0.01 T to R is encircling a long solenoid (cross section
0.06 T during a time interval of 0.25 s, the area S and turns per unit length n). The mutual
average induced current in the coil is inductance of the system is
1) 4 A 2) 8 A
µ0 n 2S
3) 5 A 4) 2 A 1) µ0nNS 2)
N
34. When 0.005 A current flows through a moving
coil galvanometer it gives full scale deflection. µ0 n 2S µ 0 nNS
3) 4)
It is to be converted into a voltmeter of range N+n 2
5V using a resistance 975 Ω. The resistance of
the galvanometer is
SECTION – B (15 questions)
Section – B consists of 15 questions. Candidates are allowed to answer any 10 questions only out of 15 questions
given. In case if candidates attempts more than 10 questions, first 10 attempted questions only considered for
marking.
36. A particle of negative charge –q and mass m 38. A cylinder of radius 5 m rolls on a horizontal
moving with velocity V0 enters a magnetic surface, velocity of its centre is 25 m/s. Find
field B as shown in the figure. The time for velocity of the point A shown in figure.
which the particles remains inside the
magnetic field, is

1) 20 ˆi + 40 ˆj m/s 2) 40 ˆi + 20 ˆj m/s
3) 10 ˆi + 20 ˆj m/s 4) 25 ˆi + 5 ˆj m/s
39. The electric field between the plates of a
parallel plate capacitor having plate area 4 cm2
is changing at the rate of 3 x 1012 NC-1s-1. The
2πm m displacement current is
1) 2) (π + 2θ)
qB qB 1) 2.02 mA 2) 3.0 mA
m πm 3) 15 mA 4) 10.6 mA
3) (π − 2θ) 4) 40. Radius of curvature of concave mirror is
qB qB
40 cm and the size of image is twice as that of
37. A particle having a charge of 10 µC and a object, then the object distance is
mass of 10 mg is projected with a speed of
1) 60 cm 2) 20 cm
5 ms-1 at an angle of 37o with a uniform
3) 40 cm 4) 30 cm
magnetic field of 3.14 T which exists in the
region. The pitch of the helical path followed 41. The refractive index of material of an
by the particle will be equilateral prism is 3 . The angle of
1) 2 m 2) 16 m minimum deviation for this prism will be
3) 32 m 4) 8 m 1) 30o 2) 45o
o
3) 60 4) 37o
69
42. A compound microscope consists of two 46. Light of intensity 1.5 kW/m2 is incident
convex lenses of focal length 1 cm and 2 cm normally on a perfectly absorbing surface. The
and has a tube length of 15 cm. m1 is the pressure exerted by the light on the surface is
magnification produced by microscope when 1) 2 x 10-6 Pa 2) 5 x 10-6 Pa
final image is formed at infinity and m2 is the
3) 1.0 x 10-5 Pa 4) 4 x 10-5 Pa
magnification produced by microscope when
47. A common emitter amplifier has a voltage
final image is formed at near point. The
difference |m2| - |m1| is gain of 80, an input resistance of 50 Ω and

1) 2 2) 187.5 output resistance of 100 Ω. The power gain of


the amplifier is
3) 30 4) 15
1) 800 2) 1600
43. A beam of light of wavelength 6000 Ao falls
on a narrow slit and the diffraction pattern is 3) 2400 4) 3200
observed at a screen 3 m away from the slit. If 48. The P-V diagram for one mole of an ideal gas
the slit width is 0.9 mm, then the position of is shown in figure. Find the volume of the gas
the first order minimum from the central when its temperature is maximum.
maximum is
1) 4 mm 2) 3 mm
3) 1 mm 4) 2 mm
44. Light of wavelength 6000 Ao falls on a single
slit of width 0.1 mm. The angular position of
the second minimum is V0 3V0
1) 2)
1) 0.006 rad 2) 0.012 rad 4 4
3) 0.003 rad 4) 0.004 rad 5V0 7V0
3) 4)
45. For three different metals A, B and C 4 4
photoelectric effect is observed one by one. 49. If in a hypothetical Bohr’s hydrogen atom
The graph which correctly give variation of electron is replaced by a particle of equal
maximum kinetic energy KEmax of
charge and twice mass, then energy E0 of the
photoelectrons with frequency of incident
first orbit will be
radiation is
1) – 27.2 eV 2) – 13.6 eV
3) – 6.8 eV 4) 0 eV
50. The half life of a radioactive isotope is T. The
1) 2)
time after which its activity reduces to 1 % of
its original value is
1) 6.66 T 2) 9.68 T
3) 99 T 4) 49 T
3) 4)
70
CHEMISTRY
SECTION – A (35 questions)
(Answer all the questions)
51. Equal weights of ethane and hydrogen are 57. Which of the following is best method to
mixed in an empty vessel at 25°C. The prepare benzyl-ter-butyl ether?
fraction of the total pressure exerted by 1) (CH3)3C - O-Na+ + C6H5CH2 – Br
hydrogen is 2) C6H5CH2ONa + (CH3)3CBr
1 3) C6H5Br + (CH3)2CHCH2Br
1) 2) 1
2 4) Both (1) and (3)
1 15 58. Highest oxidation state of manganese in
3) 4)
16 16 fluoride is + 4 (MnF4) but highest oxidation
52. Which of the following complex will show state in oxides is + 7 (Mn2O7) because
geometrical as well as optical isomerism? 1) Fluorine is more electronegative than
(en = ethylene diamine) oxygen
1) [Pt(NH3)2Cl2] 2) [Pt(NH3)2Cl4] 2) Fluorine does not possess d – orbitals
4+
3) [Pt(en)3] 4) [Pt(en)2Cl2] 3) Fluorine stabilizes lower oxidation state
53. Maltose is made up of
4) In covalent compounds fluorine can form
1) α - D - glucose single bond only while oxygen forms
2) D - fructose
double bond.
3) α - D - glucose and β - D - glucose
59. Which one of the following has lowest value
4) Glucose and fructose
of magnetic behaviour?
54. Thermal stability of alkaline earth metal
1) [Cr(CN)6]3- 2) [Mn(CN)6]3-
carbonates decreases in the order
3) [Fe(CN)6]3- 4) [Co(CN)6]3-
1) BaCO3 > SrCO3 > CaCO3 > MgCO3
60. Two particles of masses ‘m’ and ‘2m’ have
2) BaCO3 > SrCO3 > MgCO3 > CaCO3
equal kinetic energies. Their de Broglie
3) CaCO3 > SrCO3 > MgCO3 > BaCO3
4) MgCO3 > CaCO3 > SrCO3 > BaCO3 wavelength are in the ratio of
1) 1 : 1 2) 1 : 2
55. Which of the following is heterocyclic
aromatic species? 3) 1 : 2 4) 2 : 1
61. Which of the following compounds is most
1) 2) basic?
1) cyclohexylamine
2) aniline
3) 4) 3) p – methoxyaniline
4) p – nitroaniline
56. In the reaction 62. In the reaction CH3–CH2 –CBr3 
Ag powder/∆
→[p],
the product [p] is
C6H5CH3 
oxidation
→ A 
NaOH
→ B 
Soda lim e

→C
1) propyne
The product 'C' is
2) H3C-C≡C-Ag
1) C6H5OH 2) C6H6
3) 3 – hexyne
3) C6H5COONa 4) C6H5ONa
4) 3 – hexene
71
63. 2 – methyl butane on reacting with Br2 in 70. In which of the following set carbon has
presence of sunlight mainly gives different hybridization states?
1) 1 – bromo – 2- methyl butane 1) graphite, fullerene
2) 2 – bromo – 2 – methyl butane
3) 2 – bromo – 3- methyl butane 2) CO, CO2
4) 1 – bromo – 3- methyl butane 3) CO2, C2O3
64. Which compound on reaction with ethyl 4) CO, graphite
magnesium bromide and water will form
71. Van’t Hoff factors of aqueous solutions of X,
2–methyl–2– butanol?
Y and Z of same molarities are 1.8, 0.8 and
1) C2H5COCH3 2) CH3COOCH3
3) CH3CH2CHO 4) CH3COCH3 2.5 respectively. Then correct among
65. In which of the following silicates, only two following is
corners per tetrahedron are shared, 1) Vapour pressure : Y < X < Z
1) pyro silicate 2) cyclic silicate 2) Boiling point : Y < X < Z
3) 3D – silicate 4) sheet silicate
3) Osmotic pressure : X = Y = Z
66. Which is incorrect among the following?
1) B < C < O < N (IE1) 4) Freezing point : Y < X < Z
2) I < Br < F < Cl(EA) 72. The acid, readily undergo decarboxylation in
presence of soda lime
3) S > O > P > N (EA)
1) butanoic acid
4) O-2 > F- > F > O (radius)
67. Which of the following is incorrect according 2) α - halo butanoic acid
to M.O.T? 3) β - halo butanoic acid
1) B2 has a π - bond 4) γ - halo butanoic acid
2) C2 has 1σ bond and 1π - bond 73. Which of the following is incorrect matching
3) N2 has 1σ bond and 2π - bonds of colloids?
4) O2 has 1σ bond and 1π - bond 1) Smoke = solid in gas
68. Biochemical oxygen demand (BOD) is a 2) Smog = liquid in gas
measure of organic material present in water. 3) Emulsion = solid in liquid
BOD value less than 5 ppm indicates a water 4) Gel = liquid in solid
sample to be
74. Carbon & carbon monoxide can’t be
1) rich in dissolved oxygen
suggested for the reduction of metal oxide, in
2) poor in dissolved oxygen
the metallurgy of
3) highly polluted 1) Zn 2) Cu
4) not suitable for aquatic life 3) Fe 4) Al
69. Which of the following alkane is synthesized
75. Total volume of gases evolved when dilute
from one type of alkyl halide by Wurtz reaction?
H2SO4 is electrolyzed by using 2F electricity
1) 2) at STP is
1) 11.2 L 2) 22.4
3) 4) 3) 33.6 L 4) 44.8 L
72
76. In the face centred cubic lattice, atom A 3) Condensation polymer
occupies the corner positions and atom B 4) Thermosetting polymer
occupies the face centre positions. If one atom 81. Match the column – I with column – II
of B is missing from one of the face centred
Colum – I Colum – II (uses)
points, the formula of the compound is
A) D2O P) Removes the permanent
1) A2B 2) AB2
hardness of water
3) A2B2 4) A2B5
B) Ca(OH)2 Q) Used as antiseptic
77. Addition of inert gas at constant pressure
C) Zeolite R) Removes the temporary
yields more product in the reaction is
hardness of water
1) N2(g) + 3H2(g) ⇌ 2NH3(g)
D) H2O2 S) Moderator in the nuclear
2) 2SO2(g) + O2(g) ⇌ 2SO3(g) reactors
1) A-S, B-Q, C-R, D-P
3) N2(g) + O2(g) ⇌ 2NO(g)
2) A-S, B-R, C-P, D-Q
4) PCl5(g) ⇌ PCl3(g) + Cl2(g) 3) A-R, B-S, C-P, D-Q
78. The incorrect statement of the following 4) A-Q, B-R, C-P, D-S
is/are 82. The correct thermodynamic conditions for the
1) Basicity of all oxoacids of halogens is 1 spontaneous reaction at all temperature is
2) Basicity of all oxoacids of sulphur is 2 1) ∆H < 0 and ∆S > 0
3) Basicity of all oxoacids of phosphorous is 3 2) ∆H < 0 and ∆S < 0
4) All the above 3) ∆H > 0 and ∆S = 0
79. Choose the correct statements. 4) ∆H > 0 and ∆S > 0
A) Glutamine, Serine, Cysteine are Non- 83. Which of the following is a state function as
essential amino acids. well as an extensive property?
B) Amino acids are colourless, crystalline a) Temperature b) Internal energy
solids and water soluble c) Entropy d) Heat capacity
C) Amino acids behave like salts in aqueous 1) a, b, c and d 2) b, c and d
solution rather than simple amines or 3) a, c and d 4) a and b
carboxylic acids.
84. In the reaction SO2 + H2S → 3S + H2O
D) Amino acids show amphoteric behaviour
the substance oxidized is
in zwitter ionic form.
1) S 2) SO2
1) A, B and C only 2) A, B, C and D
3) H2S 4) H2O
3) B, C and D only 4) A, C and D only
85. Correct decreasing order of volatility is
80. Which of the following is not correct
1) H2O > H2S > H2Se
regarding terylene?
2) H2S > H2O > H2Se
1) Step-growth polymer
3) H2Se > H2O > H2S
2) Synthetic fibre
4) H2S > H2Se > H2O
73
SECTION – B (15 questions)
Section – B consists of 15 questions. Candidates are allowed to answer any 10 questions only out of 15
questions given. In case if candidates attempts more than 10 questions, first 10 attempted questions only
considered for marking.
86. Equivalent conductance of 0.05 N solution of 89. A buffer solution is prepared by taking
acetic acid is 7.36 S cm2 eq-1 at 25°C. What is 0.25 mol of acetic acid (pka = 4.761) and
the value of Ka for this acetic acid solution if 0.4 mol of barium acetate in sufficient water to
equivalent conductance of CH3COOH at make 1.40 L of solution. Calculate the pH of
infinite dilution is 390.7 S cm2 eq-1? the solution [log 5 = 0.699]
1) 9.4 x 10-4 2) 7.1 x 10-4 1) 4.25 2) 4.45
3) 1.77 x 10-5 4) 4.7 x 10-5 3) 5.00 4) 5.27
87. Which of the following statement is incorrect 90. The mass and volume of 1 molecule of water
in the context of the B-F bond in BF3? are respectively (density = 1g/cc)
1) All the three B-F bond lengths are equal 1) 1 x 10-23 g, 1 x 10-23 cm3
and each of them is shorter than the sum of 2) 1 x 10-23 g, 3 x 10-23 cm3
the covalent radii of boron and fluorine
3) 3 x 10-23g, 3 x 10-23 cm3
2) The bond energy of the B-F bond is very
high, higher than for any other single bond 4) 3 x 10-23g, 1 x 10-23 cm3
3) The unusual shortness of the B-F bond 91. Match the following with respect when a cell
may be explained by a Pπ - Pπ interaction working as an electrochemical cell
between boron and fluorine atoms List – I List - II
4) The unusual shortness and strength of the A) Sign of cathode P) +ve electrode to
bond may be explained by a Pπ - dπ –ve electrode
interaction between the atoms of boron B) Sign of anode Q) –ve electrode to
and fluorine. +ve electrode
88. End product of the following sequence of C) Electrons flow from R) +ve
reaction is D) Current flow from S) -ve
CO 2 /H 3O +
CH ≡ CH 
→ →
CH 3 MgBr
1) A-S, B-R, C-P, D-Q
→ 

HgSO 4 / H 2SO 4 Ag 2 O
∆ 2) A-S, B-R, C-Q, D-P
O 3) A-R, B-S, C-Q, D-P
||
1) CH 3 − C − COOH 4) A-R, B-S, C-P, D-Q

2) CH2(COOH)2 92. The V-C bond distances in [V(CO)6] and


O [V(CO)6]- are respectively (in pm)
|| 1) 200, 200
3) CH 3 − C − CHO
2) 193, 200
O
|| 3) 200, 193
4) H − C − CH 2COOH 4) 193, 193
74
93. The initial concentration and rate of a first
order reaction is 0.1M and 10-3 mol L-1 s-1. 1) 2)
Determine the half-life of the reaction
1) 0.6935 s 2) 6935 s
3) 69.3 s 4) 6.93 s 3) 4)
94. The correct orders of the following
A) O-O bond length : H2O2 > O3 > O2 98. Aldehydes that do not undergo aldol
-2
B) Bond order : CO > CO2 > CO3 condensation are
C) Molecules does not exist : He2, Be2, Ne2 (a) propanal
D) Order of boiling point (b) methanal
p-nitrophenol > o-nitrophenol (c) 2,2-dimethyl propanal
E) Dipole moment : (d) ethanal
(e) benzaldehyde
1) c and d only
2) b and e only
1) A, B, C, D, E
3) a, b and c only
2) A, B, C, D only
4) b, c and e only
3) B, C, D, E only ∆
99. A 
NH 3
→ B  → C 
KOH /Br2


4) A, C, D, E only
(CH3)2CH−NH2
95. Stable carbocation among the following is
Here ‘A’ is
1) 2)
1) 2)
O OH
3) 4)
3) 4)
96. 1 mole of K2Cr2O7 oxidizes 3 moles of a
100. The following compound is used as
compound X in acidic medium. Then how
many moles of KMnO4 are required to oxidize
1 mole of compound X in acidic medium to
give the same oxidized product.
1) 5/6 2) 5/2
3) 2/5 4) 5/5 1) Antacid
2) Analgesic
3) Hypnotic
4) Antiseptic
97.
75
BOTANY
SECTION – A (35 questions)
(Answer all the questions)
101. Natural plant hormone isolated from corn 109. Matchthe following
kernels and coconut milk is Column – I Column – II
1) Kinetin 2) Zeatin A) Deoxy ribose I) Nitrogen base
3) GA3 4) Ethylene
B) Guanine II) Pentose sugar
102. One hormone helps in ripening of fruits and
other brings about stomatal closure, they C) Adenosine III) Nucleotide
respectively are D) Cytidylic acid IV) Nucleoside
1) ABA, IAA 2) C2H4, IAA 1) A – IV, B – II, C – III, D – I
3) C2H4, Cytokinins 4) C2H4, ABA 2) A – III, B – I, C – IV, D – II
103. Find out the oviparous animals 3) A – II, B – I, C – IV, D – III
1) Mice 2) Birds 4) A – I, B – IV, C – III, D – II
3) Monkey 4) Cockroach 110. During non-cyclic electron transport, the
104. Which are the monocarpic biennial plants in terminal electron acceptor is
following combinations? 1) glucose 2) ATP
1) Rice, Agave, Carrot 3) H2O 4) NADP+
2) Agave, Bambusa, Carrot 111. Plants absorb the element which is required
3) Carrot, Agave for pollen germination in the form of
4) Carrot, Radish 1) B2O32- 2) B4O72-
105. Which of the following cell organelle is 3) B4O33- 4) BO72-
responsible for trapping light energy to form 112. Which among the following elements are
ATP? essential for photolysis of water?
1) Mitochondria 2) Chloroplast 1) Manganese and Zinc
3) Golgi bodies 4) Both (1) and (2) 2) Copper and Boron
106. One extremely short arm and one very long 3) Molybdenum and Chlorine
arm is observed in which type of 4) Manganese and Chlorine
chromosome?
113. If the osmotic pressure of the cytoplasm
1) Metacentric 2) Sub-metacentric balances the osmotic pressure of solution. The
3) Acrocentric 4) Telocentric cell is said to be
107. Find out the odd one w.r.t endomembranous 1) Flaccid 2) Fully turgid
system. 3) Plasmolysed 4) Partially turgid
1) ER 2) Golgibodies 114. The enzymatic step of PCR is
3) Ribosomes 4) Lysosomes 1) Denaturation 2) Primer annealing
108. Transition state of the substrate during an 3) Primer extension 4) Both (2) and (3)
enzymatic reaction is
115. Sickle cell anemia is due to
1) Transient but stable
1) Chromosomal abbreviation
2) Transient and unstable
2) Frame shift mutation
3) Permanent but stable
3) Point mutation
4) Permanent but unstable
4) Both (2) and (3)
76
116. In eukaryotes, gene regulation occurs at IV) Algae synthesize food and fungi absorb
A) Transcriptional level water and mineral nutrients
B) Processing level 1) I, II only 2) II, III only
C) Translational level 3) III, IV only 4) All of the above
D) Translocation level 123. Identify the first archegonial land plants from
1) A, B and C 2) B, C and D the following
3) A, C and D 4) A, B, C and D 1) Bryophytes 2) Pteridophytes
117. Human genome project leads to the 3) Gymnosperms 4) Angiosperms
development of new branch of biology is 124. Find the incorrect combination
called
1) Sphagnum – Protonema
1) Information technology
2) Marchantia – Dioecious
2) Bioinformatics
3) Lycopodium – Prothallus
3) Biophysics
4) Wolffia – Smallest fern
4) Biochemistry
125. AB blood group is an example of
118. Manuals are useful in providing information
for 1) Co-dominance
1) Identification of names of species 2) Incomplete dominance
2) Information on any one taxon 3) Point mutation
3) Books related to botanical titles 4) Pleiotrophic gene
4) Information of plant species in a large 126. The gene which expressed both in
geographic area heterozygous and homozygous condition is
119. During non-cyclic electron transport, the called
reduction of NADP+ into NADPH occurs in 1) Dominant gene 2) Recessive gene
1) the stroma 2) the lumen 3) Pleiotrophic gene 4) Polygene
3) the stroma lamella 4) the cytoplasm 127. Prop roots of banyan tree are meant for
120. High yielding and disease resistant wheat 1) Photosynthesis 2) Climbing
varieties developed in India are 3) Mechanical support 4) Transpiration
1) Sonalika and kalyan sona 128. Cork is formed from
2) Jaya and Ratna 1) Phellogen
3) Jaya and Kalyan sona 2) Vascular cambium
4) Himgiri and Pusa komal
3) Intercalary meristem
121. Generally viruses that infect plants have
4) Apical meristem
1) ssRNA 2) ssDNA
129. Main function of lenticel is
3) dsDNA 4) dsRNA
1) Transpiration 2) Guttation
122. Read the statements and identify the correct
one. 3) Gaseous exchange 4) Both (1) & (3)
130. In which stage of meiosis, the chromosomes
I) Lichens are symbiotic association of algae
and fungi appear thin and long thread-like?
II) Fungal partner is mycobiont 1) Zygotene 2) Leptotene
III) Algal partner is phycobiont 3) Pachytene 4) Metaphase – I
77
131. Identify the asexual reproductive structures of 1) E – Microspore mother cell,
fungi? B – Endothecium
1) Ascospores 2) Basidiospores 2) A – Epidermis, E – Microspore mother cell
3) Conidia 4) Zygospores 3) C – Middle layer, D – Epidermis
132. Select the correct match from the following. 4) D – Microspore mother cell, E – Tapetum
1) Nucellus – Present in microsporangium 134. Sclerenchymatous bundle sheath is the

2) Pollen grain – Developed in characteristic of


megasporangium 1) Dicot stem
3) Embryo sac – Female gametophyte of 2) Monocot stem
Pteridophytes 3) Dicot leaf
4) Generative cell – Produce male gametes 4) Dicot root
133. Identify the diagram and choose the correct 135. Meloidgyne incognitia is a/an
options 1) Insect
2) Nematode
3) Bacteria
4) Fungi

SECTION – B (15 questions)


Section – B consists of 15 questions. Candidates are allowed to answer any 10 questions only out of 15
questions given. In case if candidates attempts more than 10 questions, first 10 attempted questions only
considered for marking.
136. The prokaryotic structural genes are generally 1) 12 2) 14
1) Monocistronic 2) Polycistronic 3) 15 4) 16
3) Split gene 4) All of the above 140. The first substrate level phosphorylation

137. Identify the pair of plants which show reaction occurs in which reaction of aerobic
diplontic life cycle respiration?
1) Volvox and Fucus 1) 1, 3-bis PGA → 2 PGA
2) Fucus and Ectocarpous 2) 1, 3 bis PGA → 3 PGA
3) Pea and Ficus 3) 3 PGA → PEP
4) Ficus and kelps 4) Succinyl coA → Succinate
138. Select the wrongly matched pair 141. Statement – I : Many ribosomes are attached
to single mRNA to form polysome during
1) Double fertilization – Angiosperm
transcription process.
2) Gibberella fujikuroi –Bakane disease Statement – II : Prokaryotic ribosomes are
3) Bryophytes – Vascular plants made up of 50S and 30S subunits.
4) Cycas – Unisexual plant 1) Both the statements are correct
139. If one molecule of pyruvic acid enter into 2) Both the statements are incorrect
aerobic respiration, how many ATP are 3) Statement – I is correct, II is incorrect
produced through ETS? 4) Statement – I is incorrect, II is correct
78
142. Which of the given graphs shows the effect of 146. In lac operon , the regulator gene codes for
substrate concentration on enzyme activity? 1) Aporepressor
2) Co-repressor
1) 2) 3) Inactive repressor
4) Active repressor
147. Which is not correct statement about human
genome?
3) 4)
1) Largest known gene in human genome is
143. Match the following columns dystrophin gene
Column – I Column – II 2) 99.9% nucleotide sequences are identical in
A) Exonuclease I) Synthesis of all people
DNA 3) ‘Snips’ occurs at about 1.4 million locations
B) Endonuclease II) Joining the in humans
DNA 4) Y chromosome has most genes (2968) and
fragments
chromosome I has the fewest (231)
C) DNA III) Cut the DNA
148. Vexillary aestivation is the characteristic
polymerase at 3′ or 5′ ends
feature of
D) Ligase IV) Cut the DNA 1) Brassicaceae
other than
2) Fabaceae
terminal ends
3) Solanaceae
1) A – II, B – IV, C – III, D –I 4) Lilliaceae
2) A – III, B – IV, C – I, D – II 149. Introduction of foreign nucleic acids into cells
3) A – IV, B – III, C – II, D – I by viral vector is done during which of the
4) A – II, B – IV, C – III, D – I following procedures
144. Today, transgenic models have been
1) Gene therapy
developed for many human diseases such as
2) Production of insulin
a) Rheumatoid arthritis
b) Alzhimer’s 3) Bt cotton
c) Cancer 4) All of these
d) Cystic fibrosis 150. Which of the following microbe cannot be
1) a and c only 2) a and d only cultured on artificial medium?
3) b and d only 4) a, b, c and d a) E.coli b) Penicillium
145. The best method and permanent cure of ADA c) TMV d) Retrovirus
deficiency is e) Bacteriophage
1) Bone marrow transplantation 1) a, b, c only
2) Enzyme replacement therapy
b) a, d, e only
3) Giving functional ADA in the form of
injection c) c, d, e only
4) Gene producing ADA is isolated from d) b, c, d only
bone marrow cells and introduced into the
cells of early embryonic stage
79
ZOOLOGY
SECTION – A (35 questions)
(Answer all the questions)
151. Find the total number of mammals from the 1) A - iii, B - iv, C - i, D - ii
following. [Balanoglossus, Macropus, 2) A - iv, B - iii, C - i, D - ii
Psittacula, Ichthyophis, Delphinus,
Balaenoptera, Aptenodytes] 3) A - iii, B - iv, C - ii, D - i
1) 4 2) 6 4) A - i, B - iv, C - ii, D - iii
3) 2 4) 3 157. Statement-I: A cockroach consists of 2000
152. Exclusively marine animals are seen in hexagonal ommatidia.
1) Porifera and Annelida Statement - II: Vision of cockroach is known
2) Arthropoda and Mollusca as mosaic vision with more sensitivity but less
3) Ctenophora and Echinodermata resoluton.
4) Aschelminthes and Platyhelminthes
1) Both Statements I and II are correct
153. Based on classification which set is closely related?
1) Crocodilus, Hemidactylus, Salamandra 2) Statement I is correct, statement II is
2) Panthera, Equus, Alligator incorrect
3) Bufo, Vipera, Rattus 3) Statement I is incorrect, statement II is
4) Aptenodytes, Psittacula, Struthio correct
154. Assertion (A): The characteristic features of 4) Both statements I and II are incorrect
Aves are presence of feathers and wings. 158. Glandular epithelium is made up of
Reason (R): The hind limbs of birds have
1) columnar cells only
scales and are modified for various functions.
2) squamous cells only
1) Both A and R are true and R is the correct
explanation of A 3) cuboidal cells only
2) Both A and R are true and R is the not 4) either columnar or cuboidal cells
correct explanation of A 159. Function of epiglottis is prevention of
3) A is true but R is false 1) entry of air into trachea
4) A is false but R is true 2) back flow of food into pharynx
155. Two chambered heart and poisonous sting are
3) entry of food into larynx
present in
1) Scorpion 2) Trygon 4) entry of food into oesophagus
3) Torpedo 4) Catla 160. Which one of the following statements is
156. Match the following columns w.r.t cockroach incorrect?
reproductive system and choose the correct 1) Total volume of air a person can expire
option. after a normal inspiration is inspiratory
Column - I Column - II capacity
i) Accessory reproductive 2) The presence of non- respiratory air sacs,
A. Titillator
glands in males increases the efficiency of respiration in
B. Collaterial birds
ii) Spermatophores
glands 3) Insects have a network of tubes to transport
C. Mushroom atmospheric air within the body
iii) Left phallomere
shaped gland
4) Amphibians like frogs can respire through
D. Seminal vesicles iv) Female cockroach their skin
80
161. Match the column I with column II. 165. Which one of the following activates the
Column - I Column - II adrenal cortex?
1) Angiotensin - I 2) Aldosterone
A. Carbamino haemoglobin i) 70%
3) ANF 4) Angiotensin - II
B. Bicarbonates ii) 3% 166. A specialized connective tissue with very hard
C. Oxygen in dissolved state iii) 20 - 25% matrix due to calcium salts is the
D. CO2 in dissolved state iv) 7% 1) cartilage 2) bone
1) A - ii, B - i, C - iii, D - iv 3) tendon 4) both 1 and 2
167. Choose the correct matching pair.
2) A - iii, B - i, C - ii, D - iv
3) A - i, B - iii, C - iv, D - ii 1) Flat bone on the ventral midline of thorax -
sternum
4) A - ii, B - i, C - iv, D - iii
2) Along slender bone with two curvatures -
162. Statement-I: The openings of the right and
scapula
the left ventricles into the aorta and the
3) A long slender bone with three curvatures -
pulmonary artery respectively provided with
vertebral column
the semilunar valves.
4) A bone formed by the fusion of three bones
Statement - II: A patch of tissue present in - clavicle
the left upper corner of the right atrium called
168. Which of the following statements are correct
the sino-atrial node.
related to Na+, K+ pump?
1) Both Statements I and II are correct
i) Needs energy to work
2) Statement I is correct, statement II is
ii) Expels 2Na+ for every 3K+ ions imported
incorrect
iii) Works against a concentration gradient
3) Statement I is incorrect, statement II is
iv) Maintains resting potential
correct
1) ii and iv only correct
4) Both statements I and II are incorrect
2) ii, iii and iv only correct
163. Match the following columns and select the
3) i, iii and iv only correct
correct option.
4) all are correct
Column - I Column - II
169. Daylight vision is also called
A. T-wave i) excitation of the atria 1) Scotopic vision and is the function of cones
ii) depolarization of the 2) Scotopic vision and is the function of Rods
B. P-wave
ventricles 3) Photopic vision and is the function of cones
iii) repolarisation of the 4) Photopic vision and is the function of rods
C. QRS complex
ventricles 170. Mark the false statement w.r.t thyroid gland.
1) A - i, B - iii, C - ii 1) Two lobes of thyroid gland are inter
2) A - i, B - ii, C - iii connected by a thin flap of connective
3) A - iii, B - ii, C - i tissue called Isthmus
2) Follicular cells synthesise two hormones,
4) A - iii, B - i, C - ii
thyroxine (T4) and triiodothyronine (T3)
164. In which segment of the nephron conditional
3) Due to cancer / nodular development of
reabsorption of Na+ and water takes place?
thyroid glands increase in the synthesis and
1) DCT 2) PCT secretion of thyroid hormones occur.
3) CD 4) Henle’s loop 4) Thyrocalcitonin of thyroid gland regulates
the blood Na+ levels
81
171. Observe the below anatomical divisions of 175. Which of the following is not an natural /
pituitary gland. Identify the A, B and C traditional methods of contraception?
correctly. 1) Coitus interruptus
2) Periodic abstinence
3) Lactational amenorrhea
4) Tubectomy
176. In vitro fertilisation is a technique that
involves transfer of which one of the
following in to fallopian tube.
1) embryo of 32 cell stage
2) zygote or early embryo upto 8 cell stage
1) A - Neurohypophysis, B - Adenohypophpysis,
3) blastocyst stage
C - Pars distalis
4) ovum unfertilized
2) A - Adenohypophpysis, B - Neurohypophysis,
177. A) Inability to conceive (or) produce children
C - Pars distalis
even after 2 years of unprotected sexual
3) A - Adenohypophpysis, B - Anterior pituitary,
cohabitation is called infertility.
C - Neurohypophysis
4) A - Pars distalis, B - Neurohypophysis, B) Physical, congenital, diseases, drugs,
C - Anterior pituitary immunological or even psychological effects
could be also reason for infertility.
172. Statement-I: FSH hormone helps in the
process of spermatogenesis. C) Improved programmes covering wider
reproduction related areas are currently in
Statement - II: Anterior portion of the sperm
operation under the family planning
is covered by cap-like structure.
programmes
1) Both Statements I and II are correct
D) Programmes called ‘family planning’ were
2) Statement I is correct, statement II is initiated in India in year 1951.
incorrect
How many of the above statements are
3) Statement I is incorrect, statement II is correct?
correct
1) A and B only 2) A, B and D only
4) Both statements I and II are incorrect
3) A, B and C only 4) C and D only
173. The developmental stage at which secondary
178. Statement-I: Multiple alleles can be observed
oocyte secretes zona pellucida around it
in a population but not in individuals.
1) 2) Statement - II: In AB-ve blood group
antibodies are absent.
1) Both Statements I and II are correct
3) 4)
2) Statement I is correct, statement II is
incorrect
174. Identify the cells which line the seminiferous 3) Statement I is incorrect, statement II is
tubules. correct
1) spermatogonia and interstitial cells
4) Both statements I and II are incorrect
2) sertoli cells and interstitial cells
179. Assertion (A): Genotype with 3 dominant
3) male germ cells and sertoli cells alleles and 3 recessive alleles will have an
4) leydig cells and interstitial cells intermediate skin colour.
82
Reason (R): Number of each type of alleles in 4) Life appeared 500 million years after the
the genotype would determine the darkness or formation of earth
lightness of the skin in an individual. 182. Embryological support for evolution was
1) Both A and R are true and R is the correct proposed by
explanation of A 1) Karl Ernst Von Baer 2) S.L. Miller
2) Both A and R are true and R is the not 3) Ernst Heckel 4) Lamarck
correct explanation of A
183. A population will not exist in Hardy Weinberg
3) A is true but R is false
equilibrium if
4) A is false but R is true
1) the population is large
180. A colour blind man marries a woman with
2) there is no gene flow
normal vision but carrier, then the offspings
3) there is no genetic recombination
will be
4) individuals mate selectively
A) 50% daughters are carriers
184. Read the following statements and choose the
B) 50% sons are colour blind
wrong w.r.t malaria.
C) 75% of the generation is normal
1) Malaria, a disease man has been fighting
D) 50% of the daughter are colour blind.
since many years
1) A, B and C are correct
2) The malarial parasite initially multiply
2) B, C and D are correct within the liver cells and then attack the red
3) A, B and D are correct blood cells
4) A, C and D are correct 3) The malarial parasite requires two hosts to
181. Select the incorrect statement. complete its life cycle
1) The first non-cellular forms of life could 4) The malarial parasite reproduce sexually in
have originated three billion years back liver cells
2) The first cellular form of life did not 185. Which type of army of proteins are present in
possibly originate till about 2000 billion colostrum?
years ago 1) Ig - G 2) Ig - E
3) The first cellular form of life did not 3) Ig - D 4) Ig - A
possibly originate till about 2000 mya
SECTION – B (15 questions)
Section – B consists of 15 questions. Candidates are allowed to answer any 10 questions only out of 15
questions given. In case if candidates attempts more than 10 questions, first 10 attempted questions only
considered for marking.
186. Antitoxin
is a 187. Whichof following statements is wrong?
1) preparation containing antibodies to the 1) Human immune system consists of
toxin lymphoid organs, antibodies
2) preparation containing antigens to the toxin 2) Our immune system remembers foreign
antigens
3) preparation containing antibodies to the
3) Our immune system plays an important role
antibodies
in organ transplantation
4) preparation containing antigens to the host 4) Bone marrow, lymph nodes are secondary
and primary lymphoid organs respectively
83
188. Read the following statements and choose the 194. Which animals are under extinction due to
wrong related to the inbreeding. over exploitation?
1) Inbreeding refers to the mating of more 1) Steller’s sea cow 2) Passenger pigeon
closely related individuals within the same 3) Quagga 4) both 1 and 2
breed for 4 - 6 generations 195. Identify the correct statement.
2) A superior cow or buffalo produce more
1) The development of the fertile top soil
milk per lactation
takes centuries
3) All bulls can gives rise to superior progeny
2) Over cultivation do not contribute to soil
4) Inbreeding exposes harmful recessive
genes that are eliminated by selection erosion
189. When does the size of population increases?
3) Increased urbanization do not contribute to
1) Mortality > Natality soil desertification
2) Mortality = Natality 4) Deforestation is the conversion of non-
3) Mortality < Natality forested areas to forested areas
4) Natality < Mortality 196. Which of the following release particulate

190. Identify the equation of Verhulst - Pearl matter?


Logistic growth 1) Smokestacks of thermal power plants
1) dN/dt = rN 2) dN/dt = (b - d) x N 2) Smelters
 K−N 3) Industries
3) dN/dt = rN   4) dN/dt = (b - d) x t
 K  4) all the above
191. Which one of the following employs sexual 197. Which one of the following is a major cause
deceit? for atmospheric pollution atleast in the metro
1) Fig 2) Ophrys cities?
3) Mango tree 4) Mycorrhizae 1) Steel factory 2) Cement factory
192. Amount of biomass or organic matter 3) Automobiles 4) Sugar factory
produced per unit area over a time period by 198. Polyblend is a
plants during photosynthesis is 1) Bioplastic
1) primary production
2) Recycled modified plastic
2) primary productivity
3) A new plastic without hydrocarbons
3) grass primary productivity
4) both 1 and 2
4) net primary productivity
199. Arrange the layers of uterus from inside to
193. According to IUCN red list (2004) 784 species
extincted in the last 500 years. Identify outside
correct one. 1) perimetrium→myometrium →endometrium
1) Invertebrates - 338, vertebrates - 359, 2) perimetrium→endometrium →myometrium
plants - 87 3) myometrium→endometrium→ perimetrium
2) Invertebrates - 359, vertebrates - 338,
4) endometrium→myometrium →perimetrium
plants - 87
200. National forest policy recommended A %
3) Invertebrates - 358, vertebrates - 339,
forest cover for plains and B % for the hills.
plants - 87
4) Invertebrates - 349, vertebrates - 348, 1) A - 21.4, B - 30 2) A - 30, B - 33
plants - 77 3) A - 33, B - 67 4) A - 67, B - 33
84

SPP – 5
Max. Marks: 720 Duration : 3 Hrs. 20 Min.
PHYSICS
SECTION – A (35 questions)
(Answer all the questions)

1. A particle of mass 10 kg is moving in a 6.


straight line. If its displacement, x with time
Section – A Section - B
‘t’ is given by x = (t3 – 2t2 – 10t)m, then the
force acting on it at the end of 4 seconds is a) Incompressible e) density
1) 240 N 2) 200 N liquid constant
3) 300 N 4) 400 N b) Turbulent flow f) stream lines
2. A coast guard ship locates a pirate ship at a c) Tube of flow g) constant
distance 560 m. It fires a cannon with an
initial speed 82 m/s. At what angle from d) Fluid flux rate in h) reynold’s
horizontal the ball must be fired so that it hits laminar flow 2000
the pirate ship 1) a – f, b – e, c – g, d - h
1) 54o 2) 125o 2) a – e, b – h, c – f, d - g
o
3) 27 4) 18o 3) a – g, b – f, c – e, d - h
3. The dimensional formula of magnetic moment is 4) a – h, b – g, c – e, d - f
1) M-2A 2) M2A 7. If 13.6 eV energy is required to ionize the
3) L2A 4) L-2 A hydrogen atom, then the energy required to
4. An object falls from a bridge which is 45m remove an electron from n = 2 is
above the water. It falls directly into a small
1) 10.2 eV 2) 0 eV
row-boat moving with constant velocity that was
12m from the point of impact when the object 3) 3.4 eV 4) 6.8 eV
was released. What was the speed of the boat? 8. An e.m.f of 5 volt is produced by a self
1) 4 ms-1 2) 12 ms-1 inductance, when the current changes at a
-1
3) 16 ms 4) 44 ms-1 steady rate from 3A to 2A in 1 millisecond.
5. A disc of mass and radius '' moves in the xy The value of self inductance is
plane as shown in the figure. The angular 1) zero 2) 5 H
momentum of the disc about the origin O at 3) 5000 H 4) 5 mH
the instant shown is 9. A rubber ball is dropped from a height of 5 m
on a planet where the acceleration due to
gravity is not known. On bouncing, it rises to
1.8 m. The ball loses its velocity on bouncing
by a factor of
16 2
5 7 1) 2)
1) mR 2 ω 2) mR 2 ω 25 5
2 2 3 9
9 3 3) 4)
3) mR 2 ω 4) mR 2 ω 5 25
2 2
85
10. The variation of electric potential with 15. The logic circuit shown below has the input
distance from a fixed point is as shown in the waveforms ‘A’ and ‘B’ as shown. Pick out the
figure. The electric intensity at d = 5m is current output waveform.

1) 2.5 Vm-1 2) -2.5 Vm-1


3) 0.4 Vm-1 4) – 0.4 Vm-1
11. What are the readings of the ammeters A1 and
A2 as shown in the figure. 1)

2)

1) 0, 1 2) 2, 4 3)
3) 2, 0 4) 3, 6
12. A radioactive decay chain starts from 93Np237
and produces 90Th229 by successive emissions. 4)
The emitted particles can be
1) Two α-particles and one β-particle 16. The current from the battery in circuit diagram
2) Three β+ particles shown is
3) One α particle and two β+ particles
4) One α particle and two β- particles
13. If the de-Broglie wavelengths for a proton and
for a α - particle are equal, then the ratio of
their velocities will be
1) 1 A 2) 2 A
1) 4 : 1 2) 2 : 1
3) 1.5 A 4) 3 A
3) 1 : 2 4) 1 : 4
17. If ε0 and µ0 are, respectively, the electric
14. When a metal surface is illuminated by light permittivity and magnetic permeability of free
of wavelengths 400 nm and 250 nm, the space, ε and µ the corresponding quantities in
maximum velocities of the photoelectrons a medium, the index of refraction of the
ejected are ‘v’ and ‘2v’ respectively. The work medium in terms of the above parameters is
function of the metal is (h = Planck’s constant, µε µε
c = velocity of light in air) 1) 2 2)
µ0 ε0 µ0 ε0
1) 2 hc x 106 J 2) 1.5 hc x 106 J
3) hc x 106 J 4) 0.5 hc x 106 J 1 µε
3) 4) none of these
2 µ0 ε0
86
18. In a microscope, the focal lengths of the 23. A wire of length 50 cm and weighing 10 gm is
objective and eye lenses are 1.6 cm and 2.5 cm attached to a spring at one end and to a fixed wall
respectively. The distance between the two at the other end. The spring has a spring constant
lenses is 21.7 cm. If the final image is formed of 50 N/m and is stretched by 1 cm. If a wave
at infinity, the distance between the object and pulse is produced on the string near the wall, then
the objective lens is how much time, will it take to reach the spring?
1) 1.65 cm 2) 0.7 cm 1) 0.1 s 2) 0.2 s
3) 1.75 cm 4) 1.8 cm 3) 0.3 s 4) 0.4 s
19. One end of a uniform wire of length L and of 24. A concave mirror for face viewing has a focal
weight W is attached rigidly to a point in the
length of 0.4 m The distance at which you hold
roof and a weight W1 is suspended from its
the mirror from your face in order to see your
lower end. If S is the area of cross-section of
image upright with a magnification of 5 is
3L
the wire, the stress in the wire at a height 1) 1.60 m 2) 0.16 m
4
from its lower end is 3) 0.24 m 4) 0.32 m
W1  W 25. Molecules of an ideal gas are known to have
1) 2)  W1 +  S three translational degrees of freedom and two
S  4 
rotational degrees of freedom. The gas is
 3W  maintained at a temperature of T. The total
 W1 +  (W1 + W)
3) 
4  internal energy, U of a mole of this gas, and the
4)
S S  C 
20. Two particles A and B initially at rest, move value of γ  = P  are given, respectively, by
towards each other, under the mutual force of  CV 
attraction. At an instance when the speed of A 5 6
1) U = RT and γ =
is ‘v’ and speed of B is ‘2v’, the speed of the 2 5
centre of mass (C.M) is 7
1) zero 2) v 2) U = 5RT and γ =
5
3) 2.5 v 4) 4 v 6
21. A body executing S.H.M. along a straight line 3) U = 5 RT and γ =
5
has a velocity of 3 ms-1 when it is at a distance
5 7
of 4 m from its mean position and 4 ms-1 when 4) U = RT and γ =
it is at a distance of 3 m from its mean position. 2 5
Its angular frequency and amplitude are 26. In the figure shown, the current in the 10 V
1) 2 rad s-1 & 5m 2) 1 rad s-1 & 10 m battery is close to
3) 2 rad s-1 & 10 m 4) 1 rad s-1 & 5 m
22. The correct value of dip angle at a place is 45o.
If the dip circle is rotated by 45o out of the
meridian, then the tangent of the angle of
apparent dip at the place is
1 1) 0.42 A from positive to negative terminal
1) 1 2)
2 2) 0.71 A from positive to negative terminal
1 3) 0.36 A from negative to positive terminal
3) 4) 2
2 4) 0.21 A from positive to negative terminal
87
2
27. If on the surface of the earth is 9.8 m/s , its 1) 7.27 x 10-4 Cm-1 2) 7.98 x 10-4 Cm-1
value at a height of 6400 km is (Radius of the 3) 7.11 x 10-4 Cm-1 4) 7.04 x 10-4 Cm-1
earth = 6400 km) 32. The ratio of radiant energies radiated per unit
1) 4.9 ms-2 2) 9.8 ms-2 surface area by two bodies is 16 : 1, the
3) 2.45 ms-2 4) 19.6 ms-2 temperature of hotter body is 1000 K, then the
28. The mass and radius of a planet are double temperature of colder body will be
that of the earth. If the time period of a simple 1) 250 K 2) 500 K
pendulum on the earth is T, the time period on 3) 1000 K 4) 62.5 K
the planet is 33. In a Young's experiment, two coherent sources
1 are placed 0.90 mm apart and the fringes are
1) 2 T 2)
2T observed one metre away. If it produces the
second dark fringe at a distance of 1 mm from
1
3) 3T 4) the central fringe, the wavelength of
3T monochromatic light used would be
29. A Carnot's engine working between 27oC and 1) 60 x 10-4 cm 2) 10 x 10-4 cm
127oC takes up of 800 J heat from the resevoir 3) 10 x 10-5 cm 4) 6 x 10-5 cm
in one cycle. What is the work done by the
34. An insulated sphere of radius R has charge
engine
density ρ. The electric field at a distance ‘r’
1) 100 J 2) 200 J
from the centre of the sphere (r < R)
3) 300 J 4) 400 J
ρr ρR
30. A particle is released from a height h, At a 1) 2)
3ε0 3ε0
certain height; its KE is two times its potential
energy. Height and speed of the particle at that ρr ρR
3) 4)
instant are ε0 ε0
h 2gh h gh 35. Voltage V and current ‘i’ in AC circuit are given
1) , 2) ,2
3 3 3 3  π
by V = 50 sin (50 t) volt i = 50 sin  50t + 
2h 2gh h  3
3) , 4) , 2gh mA. The power dissipated in circuit is
3 3 3
1) 5.0 W 2) 2.5 W
31. An infinite line charge produce a field of
7.182 x 108 NC-1 at a distance of 2 cm. The 3) 1.25 W 4) zero
linear charge density is

SECTION – B (15 questions)


Section – B consists of 15 questions. Candidates are allowed to answer any 10 questions only out of 15 questions
given. In case if candidates attempts more than 10 questions, first 10 attempted questions only considered for
marking.
36. A light rod of length ‘l’ has two masses m1 m1m 2 2
1) m1m 2 l 2 2) l
and m2 attached to its two ends. The moment m1 + m 2
of inertia of the system about an axis
m1 + m 2 2
perpendicular to the rod and passing through 3) l 4) (m1 + m2)
m1m 2
the centre of mass is
88
37. A body of mass 10 kg lies on a rough  π   π   3π 1 
horizontal surface. When a horizontal force of 2)  − + 1 :  + 1 :  + 
 2   2   4 2
F newton acts on it, it gets an acceleration of
5 ms-2 and when the horizontal force is π π π
3) − : :3
doubled, it gets an acceleration of 18 ms-2, 2 2 4
then the coefficient of friction between the  π   π 1   3π 1 
body and the horizontal surface is (Assume 4)  − − 1 : −  : + 
 2   2 4  4 2
g = 10 ms-2)
41. A disc is at rest at the top of a rough inclined
1) 0.2 2) 0.8
plane. It rolls without slipping. At the bottom
3) 0.4 4) 0.6 of the inclined plane there is a vertical groove
38. A train moves towards a stationary observer of radius ‘R’. In order to loop the groove. The
with speed 68 m/s. The train sounds a whistle minimum height of incline required is
and its frequency registered by observer is f1. 15 R 9R
If the trains speed is reduced to 34 m/s, the 1) 2)
frequency registered is f2. If the speed of 4 4
f 5R 7R
sound is 340 m/s, then the ratio 1 is 3) 4)
f2 2 5
42. A mixture of light, consisting of wavelength 590
18 1 nm and an unknown wavelength, illuminates
1) 2)
19 2 Young’s double-slit and gives rise to two
9 overlapping interference patterns on the screen.
3) 2 4)
8 The central maximum of both lights coincide.
39. A ray of light passes normally through a slab Further, it is observed that the third bright fringe
(µ = 1.5) of thickness ‘t’. If the speed of light of known light coincides with the 4th bright
in vaccum be ‘C’, then time taken by the ray fringe of the unknown light. From this data, the
to go across the slab will be wavelength of the unknown light is
t 3t 1) 885.0 nm 2) 442.5 nm
1) 2) 3) 776.8 nm 4) 393.4 nm
C 2C
43. The figure shows a square loop L of side 5 cm
2t 4t
3) 4) which is connected to a network of
3C 9C resistances. The whole setup is moving
40. The magnetic field at the centre of a circular towards the right with a constant speed of
coil of radius r is π times that due to a long 1 cms-1. At some instant, a part of L is in a
straight wire at a distance r from it, for equal uniform magnetic field of 1 T, perpendicular
currents. Figure here shows three cases : in all to the plane of the loop. If the resistance of L
cases the circular part has radius r and straight is 1.7Ω, the current in the loop at that instant
ones are infinitely long. For same current the will be close to.
B field at the centre P in cases 1, 2, 3 have the
ratio

1) 170 µA 2) 150 µA
 π   π   3π 1 
1)  −  :   :  −  3) 115 µA 4) 60 µA
 2 2  4 2
89
44. In the circuit shown in the figure, when the input 47. An electric dipole consists of two opposite
voltage of the base resistance is 10 V, VBE is zero charges of magnitude q = 1 x 10-6C separated
and VCE is also zero. Then current amplification by 2.0 cm. The dipole is placed in an external
factor of transistor is: field of 1 x 105 NC-1. What maximum torque
does the field exert on the dipole? How much
work must an external agent do to turn the
dipole end for end, starting from position of
alignment (θ = 0o)
1) 4.4 x 106 N-m, 3.2 x 10-4 J
2) – 2 x 103 N-m, -4 x 103 J
3) 4 x 103 N-m, 2 x 10-3 J
1) 103 2) 83
4) 2 x 10-3 N –m, 4 x 10-3 J
3) 133 4) 93
1
45. The specific heat of water = 4200 J – kg-1K-1 48. The mass of the moon is of earth's mass
81
and the latent heat of ice = 3.4 x 105 J – kg-1.
1 th
100 grams of ice at 0oC is placed in 200 g of and its radius that of the earth. If the
4
water at 25oC. The amount of ice that will
escape velocity from the earth's surface is 11.2
melt as the temperature of water reaches 0oC
kms-1, its value for the moon will be
is close to (in grams)
1) 0.15 kms-1
1) 61.7 2) 69.3
2) 5 kms-1
3) 64.6 4) 63.8
3) 2.5 kms-1
 7R 
46. Three moles of an ideal gas  C P =  at 4) 0.5 kms-1
 2 
49. A 10 µF capacitor is charged to a potential
pressure PA and temperature TA are
difference of 50 V and is connected to another
isothermally expanded to twice the original
uncharged capacitor in parallel. Now the
volume. The gas is then compressed at
common potential difference becomes 20 volt.
constant pressure to its original volume.
The capacitance of second capacitor is
Finally the gas is heated at constant volume to
its original pressure PA. Which of the graphs 1) 10 µF 2) 20 µF
in this figure represents the P – V diagram for 3) 30 µF 4) 15 µF
the complete process? 50. A rod of ferromagnetic material with
dimensions 10 cm x 0.5 cm x 0.2 cm is placed
in a magnetic field of strength 0.5 x 104 A-m-1
1) 2) as a result of which of which a magnetic
moment of 0.5 A-m2 is produced in rod. The
value of magnetic induction will be
1) 0.54 T 2) 6.28 T
3) 4) 3) 0.358 T 4) 2.591 T
90
CHEMISTRY
SECTION – A (35 questions)
(Answer all the questions)
51. In which set, bond energy decreases from left 2) The dihedral angle in gas phase is 111.5°
to right? and solid phase is 90.2°
i) N2, O2, H2, He2 3) The reason for the difference is due to the
ii) F2, Cl2, N2, H2 difference in the intermolecular forces
iii) O2, N2, F2, Cl2 4) Both (2) and (3)
1) i and ii only 2) i only 57. The correct IUPAC name of following
3) i and iii only 4) ii and iii only compound is
52. Molecular plane of C2H4 does not contain
1) C – C - σ bond
2) C – H - σ bonds
3) All the atoms of C2H4 1) 2,2-Dibromo-1-chloro-4-methylcyclohexane
4) π - electrons 2) 3,3-Dibromo-4-chloro-1-methylcyclohexane
53. Which of the following molecules are 3) 1,1-Dibromo-2-chloro-5-methylcyclohexane
hypovalent?
4) 1,1-Dibromo-6-chloro-3-methylcyclohexane
i) BCl3 ii) BeCl2
58. Statement – I : o-nitro phenol is purified by
iii) AlCl3 iv) PCl5 steam distillation
1) i and iv only 2) ii and iii only Statement – II : o-nitro phenol is highly
3) iii and iv only 4) i, ii and iii only soluble in water due to intra molecular
o
54. Calculate ∆G for the following reaction hydrogen bond
CO(g) + ½O2(g) → CO2(g) ∆Ho = -282.84 kJ Choose the most appropriate answer about
Given, SoCO = 213.8J / K mol, SCO
o
= 197.9 J / K mol above statements from the options
2
1) Both statement – I and II are true
So
O2 = 205.0J / K mol
2) Statement – I is true and II is false
1) -157.033 kJ 2) -257.033 kJ 3) Both statement – I and II are false
3) -15.70 kJ 4) -25.703 kJ 4) Statement – I is false and II is true
55. Which is a redox reaction? 59. The correct order of relative stability of the
1) Decomposition of CaCO3 following contributing structures are
2) HBr formation from H2 and Br2 O O−
3) Neutralization of HCl with NaOH || |
CH 3 − C − O − CH 3 ↔ CH 3 − C− O − CH 3 ↔
4) Both (1) and (3) +
(I) (II)
56. The change in dihedral angle of H2O2 differs
between gas phase and solid phase. Which are O−
correct statements about H2O2 structure? | +
CH 3 − C = O − CH 3
1) The dihedral angle in gas phase is 90.2° ••
(III)
and solid phase is 120°
1) I > II > III 2) I > III > II
3) II > III > I 4) III > II > I
91
60. Which of the following has smallest heat of 66. The chemical reaction 2O3(g) → 3O2(g)
hydrogenation? proceeds as follows
1) 2) O3 ⇌ O2 + O (fast) ; O3 + O → 2O2 (slow)
The rate law expression is
3) 4)
1) rate = K[O3]2
61. The correct order of decreasing stability of 2) rate = K[O3]2 [O2]-1
conformers of n-butane
3) rate = K[O3] [O2]
1) Anti > Fully eclipsed > Partial eclipsed >
4) rate = K[O3]-1 [O2]2
Gauche
2) Anti > Gauche > Partial eclipsed > Fully 67. Match List – I with List – II
eclipsed List – I List – II
3) Gauche > Partial eclipsed > Anti > Fully a) Concentration of i) 98% Cu
eclipsed Ag ore
4) Fully eclipsed > Gauche > Partial eclipsed b) Blast furnace ii) Pig iron
> Anti c) Blister copper iii) leaching with
62. 1-Butene and 1-Butyne can be distinguished by NaCN
1) conc.H2SO4 d) Froth floatation iv) CuFeS2
2) cold alkaline KMnO4 method
3) Br2 in CCl4
Choose the correct answer from the option
4) Ammonical AgNO3 given below
63. A crystal in a ccp lattice with a cell edge of 1) a-iii, b-ii, c-i, d-iv 2) a-iii, b-iv, c-i, d-ii
0.4518 nm has been revealed by employing X-
ray diffraction studies. The density of a copper 3) a-iv, b-i, c-iii, d-ii 4) a-iv, b-iii, c-ii, d-i
complex is found to be 7.62 g cm-3. The molar 68. Number of P-O-P bonds in P4O6 and P4O10
mass of crystal is ______ g mol-1 respectively
1) 4.24 2) 53 1) 6, 10 2) 10, 6
3) 212 4) 106 3) 6, 6 4) 10, 10
64. AB2 is 50% dissociated in water to A+2 and B-. 69. Highest oxidation state of Mn in its fluoride is
The freezing point of a one molal aqueous 1) +7 2) +2
solution of AB2 is ___ oC (Kf = 0.5 K kg mol-1) 3) +4 4) +3
1) 274 2) -1 70. Which of the following is/are correct?
3) 1 4) 272
A) Shaving soaps contain glycerol to prevent
65. Select the incorrect order for the given rapid drying
properties
B) Anionic detergents are used in tooth pastes
1) LiH > NaH > KH > RbH (Thermal stability)
2) CdCO3 > CaCO3 > K2CO3 (Covalent C) Sucralose appearance and taste are like
character) sugar and it is not stable at cooking
temperature
3) BeCO3 > MgCO3 > CaCO3 (Ease of oxide
formation on heating at same temperature) D) Cetyl trimethyl ammonium bromide is an
4) MF2 > MCl2 (general order for solubility of anionic detergent
fluorides and chlorides of alkaline earth 1) A, B, C, D 2) A, B, C
metals) 3) B, D 4) A, B
92
71. Mechanism of Finkelstein and Swarts O O
reactions are respectively || ||
3) CH 3 − C − O − C − CH 3
1) SN1 and SN2 2) SN1 and SN1
4) CH3COOC2H5
3) SN2 and SN2 4) SN2 and SN1 −

72. Which of the following reagents cannot be 78. PhCHO + HCHO →
i)OH
ii) H O +
products?
3

used to distinguish phenols from alcohols? 1) PhCOOH, HCOOH


1) Neutral FeCl3 2) Phenophthalein 2) PhCH2OH, HCOOH
3) Libermann’s test 4) NaHCO3 3) PhCOOH, CH3OH
73. The increasing order of basicity of the 4) PhCH2OH, CH3OH
following compounds is
79. The property of halogen acids, that indicated
incorrect is
1) HF > HCl > HBr > HI…. acidic strength
2) Hl > HBr > HCl > HF…. reducing strength
3) Hl > HBr > HCl > HF…. bond length
4) HF > HCl > HBr > Hl…. thermal stability
1) A < B < C 2) C < B < A 80. Photochemical smog is caused by
3) C < A < B 4) A < C < B 1) CO 2) CO2
74. Acrolein test is positive for 3) O3 4) NO2
1) polysaccharides 2) proteins 81. The standard enthalpy of combustion of H2,
3) oils and fats 4) reducing sugars C6H10 and cyclohexane (C6H12) are -241, -
75. Arrange the following polymers in decreasing 3800, -3920 kJ/mole at 25oC respectively
order of their intermolecular forces. calculate the heat of hydrogenation of
I) Nylon – 6, 6 II) Neoprene III) Polystyrene cyclohexene
1) I > II > III 2) III > II > I 1) -242 kJ 2) -121 kJ
3) I > III > II 4) II > III > I 3) +121 kJ 4) +242 kJ
82. The statement that is not true about SF4
1) It is fluorinating agent for organic compound
76.
2) It has sp3 hybridization of S and tetrahedral
shape
3) The hybridization in SF4 is sp3d and sea-
saw shape
Above conversion can be achieved by
4) SF4 is a Lewis base
1) Wolf Kishner Reduction
83. XeO4 contains
2) Clemmensen Reduction
1) 4π bonds and the remaining 4 electron pairs
3) Both 1 & 2
form a tetrahedron
4) LiAlH4
2) 3π bonds and the remaining 5 electron pairs
77. Which one of the following compound give
form a trigonal bipyramidal
carboxylic acid with HNO2?
3) 4π bonds and the remaining 4 electron pairs
O
|| from a square planar
1) C6 H 5 − C − Cl 4) 4π bonds and the remaining 4 electron pairs
2) C6H5CONH2 form pyramid
93
84. Among the TiF62− , CoF63− , Cu 2 Cl 2 & NiCl42 − the 85. A transition metal exists in its highest
colourless species are oxidation state. It is expected to behave as
1) Chelating agent
1) CoF63− and NiCl 42−
2) An oxidizing agent
2) TiF62 − and CoF63− 3) A reducing agent
3) Cu2Cl2 and NiCl4 4) Both (1) and (3)
4) TiF62− and Cu2Cl2
SECTION – B (15 questions)
Section – B consists of 15 questions. Candidates are allowed to answer any 10 questions only out of 15
questions given. In case if candidates attempts more than 10 questions, first 10 attempted questions only
considered for marking.
86. 2VO + 3Fe2O3 → 6FeO + V2O5
If we start with 2g of VO and 5.75 g of Fe2O3
which is limiting reagent 3) 4)
[V = 47.87 g, Fe = 55.85 g]
1) VO 2) Fe2O3
90. Ksp of PbBr2 is 8 x 10-5. If the salt is 80%
3) FeO 4) V2O5 dissociated in solution, calculate the solubility
87. The angular momentum of an electron in a of salt in g/L approximately (Given molar
Bohr’s orbit of He+ is 3.1652 x 10-34 kg mass of PbBr2 = 367 g/mol)
m2/sec. What is the wave number in terms of 1) 6.25 2) 12.50
Rydberg constant (R) of the spectral line 3) 1.562 4) 3.125
emitted when an electron falls from this level 91. In the following equilibrium
to first excited state. [Use h = 6.626 x 10-34 Js]
N2O4 ⇌ 2NO2
5R
1) 3R 2) When 1 mol of each N2O4 and NO2 is taken
9
and Temperature is kept at 298K and the total
3R 8R
3) 4) pressure was found to be 20 bars
4 9
Given ∆ f G oN 2O4 =100kJ and ∆ f G oNO2 = 50kJ
88. Aqueous solutions of two compounds M-O-H
and M′-O-H have been prepared in two What would be the ∆G of the reaction at 298K
different beakers. If the electronegativity of and direction of the reaction?
M = 3.0, M′=1.72, O = 3.5 and H = 2.1, Then 1) 5.705 kJ, Backward
the solutions respectively are 2) -5.705 kJ, Forward
1) Acidic, acidic 2) acidic, basic 3) -3.205 kJ, Forward
3) basic, basic, 4) basic, acidic 4) +3.205 kJ, Backward
89. Which is incorrect curve for Boyle’s law? 92. Theoretical efficiency of a hypothetical cell is
about 84% which involves the following
reaction
135o
log P +2
A(s) + B(aq) 2+
→ A (aq) + B(s), ∆H = - 285 kJ then,
1) 2)
the standard EMF of the cell is
log V 1) 1.10V 2) 1.24V
3) 2.48V 4) 2.20V
94
96. Match the following.
93.
List – I List – II
A) Sheet silicates i) SiO44-
B) Ortho silicates ii) (SiO3)n2n-
Here Y is
C) Cyclic silicates iii) Si2O76-
1) 2) D) Three dimensional iv) (Si2O5)n2n-
silicates
3) 4) v) (SiO2)n
1) A-i, B-ii, C-iii, D-iv
2) A-iv, B-i, C-ii, D-v
3) A-iv, B-ii, C-iii, D-v
94. Major product is 4) A-ii, B-iv, C-i, D-iii
97. Hydrolysis of sucrose with dilute aqueous
sulphuric acid yields.
1) 1 : 1D-(+)-glucose; D-(-)-fructose
1) 2) 2) 1 : 2D-(+)-glucose; D-(-)-fructose
3) 1 : 1D-(-)-glucose; D-(+)-fructose
4) 1 : 2D-(-)-glucose; D-(+)-fructose
3) 4) 98. If COD of a water sample is 3.2ppm, what is
the weight of acidified K2Cr2O7 required for
95. Coupling of benzene diazonium chloride with 1 litre water sample for the oxidation of
1-napthol in alkaline medium will give dissolved organic matter?
[M.W of K2Cr2O7 = 294]
1) 0.98 mg 2) 19.6 g
1)
3) 0.98 g 4) 19.6 mg
99. The crystal field stabilization energy (CFSE)
of [CoF3(H2O)3] (∆0 < P)
1) -0.8 ∆0
2) 2) -0.4 ∆0 + P
3) -0.4 ∆0
4) -0.8 ∆0 + 2P
3) 100. For the coagulation of 50ml of Fe(OH)3 sol.,
10ml of 0.5 M KCl is required. What is the
coagulation value of KCl?
1) 5 2) 10
4) 3) 100 4) 50
95
BOTANY
SECTION – A (35 questions)
(Answer all the questions)
101. Mixotrophic nutrition is found in 109. Zygote divides by meiosis in the organisms
1) Chrysophytes 2) Dinoflagellates that exhibit
3) Euglenoids 4) Protozoans 1) Diplontic life cycle
102. The parallel venation in monocot leaves is 2) Haplontic life cycle
reflected in 3) Haplo diplontic life cycle
1) Size of vascular bundles 4) Diplo haplontic life cycle
2) Undifferentiated mesophyll 110. Which of the following statements is
3) Turgidity of parenchyma cells incorrect?
4) Radial arrangement of vascular bundles 1) Secondary medullary rays are the products
103. Brassicaceae and Solanaceae members can be of redifferentiation
similar in having 2) Generally pteridophytes lack vessels in
1) Perigynous flowers their xylem
2) Bicarpellary syncarpous ovary 3) Complex tissues are a part of ground tissue
3) Epipetalous stamens system
4) Tetramerous flowers 4) Size of vascular bundles depend on the size
104. The end products of tricarboxylic acid cycle are of the veins
1) CO2 + Citric acid 111. Osmoregulation is the main function of

2) H2O + Citric acid 1) Plastids 2) Lysosomes


3) CO2 + Oxaloacetic acid 3) Microbodies 4) Vacuoles
4) CO2 + H2O 112. Match column – I with column – II

105. Which phytohormone helps the plants to Column – I Column –II


overcome apical dominance A) Alkaloid I) Codeine
1) Auxin 2) ABA B) Toxin II) Vinblastin
3) Cytokinin 4) Gibberellins
C) Pigment III) Anthocyanin
106. Which condition in plants can prevent both
autogamy and geitonogamy D) Drug IV) Abrin
1) Dioecy 2) Monoecy A B C D
3) Polygamy 4) All the above 1) I II III IV
107. Which of the following is not a substrate for 2) I IV III II
transcription? 3) III IV I II
1) ATP 2) GTP 4) II III IV I
3) TTP 4) CTP 113. Find out the mis-match from the following.
108. Which technique is employed to check the 1) Muscidae – Family
progression of restriction enzyme digestion of
2) Diptera – Order
DNA?
3) Poales – Class
1) Centrifugation 2) Chromatography
4) Chordata – Phylum
3) Gel electrophoresis 4) Autoradiography
96
114. Activator of alcohol dehydrogenase is 122. Basalbodies of cilia and flagella consists of
2+
1) Zn 2) Fe2+ 1) a pair of central microtubules with 9
3) Mg2+ 4) Mn2+ peripheral doublets
115. Cyclosporin-A is obtained from 2) A pair of central microtubules with 9
1) Monascus purpureus Peripheral triplets
2) Trichoderma polysporum 3) 9 Peripheral triplets with no central
3) Saccharomyces cerevisiae microtubules
4) Aspergillus niger 4) 9 peripheral doublets with no central
116. Cotton bollworms can be controlled by the microtubules
proteins encoded by the genes 123. Polycistronic genes can be seen in
1) cryIAb and cryIIAb 1) Puccinia 2) Polytrichum
2) cryIAc and cryIAb 3) Pseudomonas 4) Pteris
3) cryIAc and cryIIAb 124. Incorrect property of genetic code is
4) cryIAb only 1) code is nearly universal
117. The first action spectrum of photosynthesis
2) code is degenerate
was described by
3) code is unambiguous
1) Julius Von Sachs
4) code is overlapping
2) Cornelius Van Niel
125. The first clinical gene therapy was given to
3) Jan Ingenhouz treat the deficiency of
4) T.W. Engelmann 1) Adenosine deaminase
118. Centrioles duplicate in the cytoplasm of
2) Adenosine dehydrogenase
animal cells during
3) Methyl transferase
1) G1 phase 2) S phase
4) β-galactosidase
3) G2 phase 4) M phase
126. Find out the correct match
119. What is not true about apoplastic movement of
1) Pusa Swarnim – Cow pea
water in plants?
2) Karanrai – Brassica
1) Movement depends on gradient
3) Atlas 66 – Maize
2) Does not have any barrier
4) Parbhani kranti – Potato
3) Water movement is through mass flow
127. Swiss cheese is produced by the following
4) It is aided by cytoplasmic streaming
bacterium
120. The common reserved food material in both
1) Propionibacterium sharmanii
fungi and animals is
2) Lactobacillus bulgaricus
1) Starch 2) Glycogen
3) Acetobacter aceti
3) Mannitol 4) Inulin
4) Clostridium butylicum
121. A plant with alternate phyllotaxy and
128. Correct statement among the following is
actinomorphic flowers is
1) Pea 2) Canna 1) Methanogens are anerobic microbes
3) Mustard 4) Alstonia 2) Mycoplasmas are wall less protists
3) All protozoans are autotrophs
4) Lichens are acellular parasites
97
129. Coralloid roots that are associated with 1) Both the statements are correct
nitrogen fixing cyanobacteria are seen in 2) Both the statements are incorrect
1) Cedrus 2) Cycas 3) Statement – I is correct, statement – II is
3) Pinus 4) Gnetum incorrect
130. What is false w.r.t nature of enzyme action? 4) Statement – I is incorrect, statement – II is
1) Substrate binds to active site of the enzyme correct
2) Enzyme induces the substrate to alter its 134. Match list – I with list – II w.r.t placentation.
shape for fitting more tightly List – I List – II
3) Enzyme-product complex is formed after A) Marginal i) Sunflower
expected bond making or breaking in the
B) Parietal ii) Tomato
substrate
4) Enzyme release the product and free C) Axile iii) Pea
enzyme is ready to bind with another D) Basal iv) Argemone
molecule of substrate A B C D
131. The homologous chromosomes separate while 1) iii iv i ii
sister chromatids remain associated during 2) iii iv ii i
1) Anaphase II 2) Metaphase II 3) iv iii ii i
3) Anaphase I 4) Prophase I 4) i ii iv iii
132. The main amino donar in transamination
135. Number of genotyptes and phenotypes
process in plants is possible in human ABO blood types
1) Asparagine 2) Glutamine respectively
3) Glycine 4) Glutamic acid 1) 2 and 4 2) 4 and 4
133. Statement – I : Grana lamellae have both 3) 6 and 4 4) 4 and 6
PS – I and PS – II.
Statement – II : Chemiosmosis involves
diffusion of protons across the membrane
SECTION – B (15 questions)
Section – B consists of 15 questions. Candidates are allowed to answer any 10 questions only out of 15
questions given. In case if candidates attempts more than 10 questions, first 10 attempted questions only
considered for marking.
136. Pollen grains are well preserved as fossils 138. In prokaryotes, the predominal site for control
because of the presence of of gene expression is
1) sporopollenin in intine 1) transcriptional level
2) cellulose in exine 2) processing level
3) sporopollenin in exine 3) transport of mRNA from nucleus to
cytoplasm
4) pectin in intine
4) translational level to cytoplasm
137. The critical research areas of biotechnology are
139. What is false about mycorrhizal association?
1) Genetic engineering 1) Plants get resistance to root born pathogen
2) Bioprocess engineering 2) Plants get tolerance to salinity
3) Downstream processing 3) Fungi get food from plants
4) All of the above 4) Fungi can fix atmospheric nitrogen
98
140. Who among the following popularized the use 146. Match
the following w.r.t ETS
of embryological characters in taxonomy Column – I Column – II
1) James Watson A) Complex I I) NADH dehydrogenase
2) Panchanan Maheshwari B) Complex II) Cytochrome C oxidase
3) Ramachandran II
4) Rene Descartes C) Complex iii) Succinic dehydrogenase
141. Important site for formation of glycoproteins III
and glycolipids is D) Complex iv) Cytochrome c reductase
1) Lysosomes IV
2) Golgi complex 1) A – i, B – iii, C – iv, D – ii
3) Vacuoles 2) A – i, B – iv, C – ii, D – iii
4) Plastids 3) A – iii, B – i, C – ii, D – iv
142. Incorrect statement among the following is 4) A – ii, B – iii, C – i, D – iv
1) Lower the taxa, more are the characteristics 147. Find out the odd one w.r.t their recognition
that the members share within the taxon sequence in pBR322
2) Manuals provide information for 1) EcoRI 2) Cla I
identification of names of species 3) Hind III 4) Ori
3) Halophiles are eubacteria that are salt 148. What is incorrect about mendel’s
resistants experiments?
4) Vegetative characters are more easily 1) He conducted hybridization experiments on
affected by environment in plants garden pea plant
143. Correct match among the following is 2) He selected 14 true breeding pea plant
1) Rhizome – Ginger varieties as contrasting pairs
2) Turmeric – Stilt roots 3) He studied 14 pairs of contrasting
characters in pea plant
3) Pistia – Runner
4) Mendel called genes as factors
4) Jasmine – Bulb
149. Which of the following is not a part of
144. Where a plant adjust the quantity and types of
transcription?
solutes that reach the xylem
1) Promoter 2) Structural gene
1) Epidermis 3) DNA polymerase 4) Terminator
2) Exodermis 150. Identify the plant belongs to Lycopsida from
3) Endodermis the following.
4) Pericycle
145. Photosynthetic, anaerobic, free living, nitrogen
fixing bacteria among the following is
1) Rhizobium
2) Rhodospirillum
3) Frankia
A B C
4) Thiobacillus
1) A 2) B
3) C 4) Both A & B
99

ZOOLOGY
SECTION – A (35 questions)
(Answer all the questions)
151. Reproduction takes place only by sexual 1) A - common oviduct, B - vagina,
means in C - spermatheca, D - gonapophysis
1) sponges 2) A - ovary, B - vagina, C - spermatheca,
2) coelenterates D - gonapophysis
3) ctenophores 3) A - oviduct, B - vagina, C - spermatheca,
4) ctenophores and cnidarians
D - gonapophysis
152. What is common in Hydra and planaria?
1) Diploblastic condition 4) A - vagina, B - common oviduct,
2) Tentacles C - spermatheca, D - gonapophysis
3) Blind sac body plan 157. Which of the following are not sensory organs

4) Coelenteron of cockroach?
153. A. Mouth is terminal 1) Maxillary palps 2) Labial palps
B. External fertilisation 3) Anal styles 4) Anal cerci
C. Presence of air bladder 158. Junctions that facilitate the cells to
D. Four pairs of gills with operculum communicate with each other by connecting
The above characters are seen in the cytoplasm of adjoining cells are
1) Scoliodon 2) Pterophyllum 1) tight junctions
3) Trygon 4) Pristis 2) adhering junctions
154. Which of the following sets has the two 3) gap junctions
members of the same phylum? 4) both 2 and 3
1) Taenia and Ascaris 159. How many teeth appear twice during the life
2) Physalia and Ctenoplana span of a man?
3) Meandrina and Pennatula
1) 12 2) 20
4) Gorgonia and Wuchereria
155. Which of the following features is not
3) 28 4) 8
applicable to all sponges? 160. Which of the following gas is quite
1) Water canal system insignificant for the regulation of respiration
2) Radial symmetry rhythm?
3) Intra cellular digestion 1) SO2 2) N2
4) Asexual reproduction 3) CO 4) O2
156. Identify the option with correct labeling in the 161. Statement-I: The first step in respiration is
given diagram. breathing by which alveolar air is taken in.
Statement - II: Cellular respiration is the
utilization of CO2 by the cells.
1) Both Statements I and II are correct
2) Statement I is correct, statement II is
incorrect
3) Statement I is incorrect, statement II is
correct
4) Both statements I and II are incorrect
100
162. Select the incorrect statement w.r.t the ECG. 166. During muscle contraction Ca++ ions binds to
1) Electrocardiogram is a machine a sub unit of
2) ECG is a graphical representation of the 1) tropomyosin 2) troponin
electrical activity of the heart during a 3) myosin 4) meromyosin
cardiac cycle 167. A bone has a slightly elevated ridge called
3) By counting the number of QRS complexes spine which projects as a flat, expanded
that occur in a given time period, one can process called the
determine the heart beat rate of an
1) acromion 2) clavicle
individual
3) scapula 4) sacrum
4) The end of the T-wave marks the end of
systole 168. Identify the correct statement/s.

163. Match the following columns and select the A) Light induces association of retinal and
correct option. opsin
Column - I Column - II B) Action potential is generated in ganglion
cells through the bipolar cells.
A. Hypertension i) Congestion of the lungs
C) Optic nerve transmits action potentials to
B. Heart failure ii) Acute chest pain the visual cortex area of brain.
iii) Deposits of calcium, D) Visual activity is greatest at fovea.
C. Atherosclerosis fat, cholesterol and fibrous 1) A and B only 2) A and C only
tissue
3) B, C and D 4) A only
iv) Affects vital organs
D. Angina pectoris 169. Macula of internal ear refers to
like brain and kidney
1) yellow spot
E. Cardiac arrest v) heart stops beating
2) ridge present in otolith organ
1) A - v, B - ii, C - iii, D - i, E - iv 3) ridge present in basilar membrane
2) A - iv, B - i, C - iii, D - ii, E - v
4) Crista ampullaris of semicircular canal
3) A - v, B - ii, C - i, D - iii, E - iv
170. Match the following and choose the correct
4) A - iv, B - v, C - iii, D - ii, E - i option.
164. Statement-I : A rise in GFR can activate the
JG cells to release renin. Column - I Column - II
Statement - II: Reabsorption of water occurs A. Zona reticularis i) Androgenic steroids
passively in the initial segments of the B. Zona fasciculata ii) Mineralo corticoids
nephron.
C. Zona glomerulosa iii) Catecholamines
1) Both Statements I and II are correct
D. Adrenal medulla iv) Glucocorticoids
2) Statement I is correct, statement II is
incorrect 1) A - iv, B - iii, C - ii, D - i
3) Statement I is incorrect, statement II is 2) A - iii, B - ii, C - i, D - iv
correct 3) A - i, B - iv, C - ii, D - iii
4) Both statements I and II are incorrect 4) A - ii, B - i, C - iv, D - iii
165. In which of the following the reabsorption is 171. Which is not a function of a peptide hormone
minimum? produced by β-cells of islets of Langerhans?
1) PCT 2) DCT 1) Gluconeogenesis 2) Glycogenesis
3) CD 4) Henle’s loop 3) Glycogenolysis 4) both 1 and 3
101
172. Pituitary gland is located in a bony cavity 178. In GIFT, an ovum collected from a donor and
called transfer into fallopian tube of another female who
1) sella tursica of pelvic bone 1) can produce ovum
2) sella tursica of cranial bone 2) can’t produce ovum and can’t provide
3) sella tursica of facial bone suitable environment for fertilization and
development
4) sella tursica of pectoral bone
3) can’t produce ovum and can provide
173. The foetal ejection reflex in human triggers
suitable environment for fertilization and
the release of development
1) oxytocin from foetal pituitary 4) undergone hysterectomy
2) progesterone from placenta 179. A child born to parents having ‘A’ and AB
3) human chorionic gonadotrophin from the group cannot possess
placenta 1) A and AB group 2) B and AB group
4) oxytocin from matermal pituitary 3) A and B group 4) O group
174. The ovary in the female reproductive system is 180. There is no colour blindness in grandparents
called as primary sex organ, because and parents but in the third generation one of
1) it produces the female gametes for the children is colur blind the reason for this
reproduction and the sex of the child affected can be
2) it secretes female sex hormones 1) it is a crisscross inheritance and the affected
3) it nourishes the developing embryo child is female
2) grandfather is carrier of colour blindness
4) both 1 and 2
and affected child is female
175. Find the mis-matched from the following.
3) grandmother and mother are carriers for
Column - I Column - II colour blindness and the child is a male
A. Perimetrium i) Thick membranous 4) grandfather is affected mother is a carrier
and child is male
B. Myometrium ii) Thick smooth muscle
181. Which of the following disease is an
C. Endometrium iii) Glandular layer autosomal recessive?
iv) Weak contractions 1) Thalassemia 2) Haemophilia
D. Myometrium
during child birth 3) Colour blindness 4) All of these
1) B, C 2) A, D 182. Match the following columns and choose the
3) A, C, D 4) A, B, C correct option.
176. Emergency contraceptive should be taken after Column - I Column - II
coitus with in A. Potato i) Root modification
1) 73 hrs 2) 72 hrs B. Sweet potato ii) Convergent evolution
3) 74 hrs 4) 75 hrs
C. Homology iii) Stem modification
177. MTP is an technique by which pregnancy
D. Analogy iv) Divergent evolution
terminated at
1) after full term maturity of foetus 1) A - iv, B - ii, C - i, D - iii
2) before full term maturity of foetus 2) A - ii, B - iii, C - iv, D - i
3) A - iii, B - iv, C - i, D - ii
3) the time of fertilization
4) A - iii, B - i, C - iv, D - ii
4) time of ovulation
102
183. Matchthe following columns and choose the 3) A - iv, B - i, C - ii, D - iii
correct option. 4) A - iii, B - i, C - ii, D - iv
Column - I Column - II 184. The homologous organs among different
st
A. 1 non cellular forms i) 4 - billion years back organisms of same groups show similarity in
1) size 2) appearance
ii) 2000 million years
B. Life appeared almost 3) functions 4) origin & structure
ago
185. Which lymphocytes produced army proteins in
iii) 320 million years
C. 1st cellular forms response to pathogens into our blood to fight
ago
with them?
D. Sea weeds existed 1) T - lymphocytes
iv) 3 million years ago
around
2) B - lymphocytes
1) A - iv, B - ii, C - i, D - iii 3) both B & T lymphocytes
2) A - ii, B - iii, C - iv, D - i 4) First T- lymphocytes then B - lymphocytes
SECTION – B (15 questions)
Section – B consists of 15 questions. Candidates are allowed to answer any 10 questions only out of 15
questions given. In case if candidates attempts more than 10 questions, first 10 attempted questions only
considered for marking.
186. Which immune response is responsible for the Reason (R): It exposes harmful recessive
graft rejection? genes that are eliminated by selection.
1) Humoral immune response 1) Both A and R are true and R is the correct
2) Innate immunity explanation of A
3) Cell mediated immune response 2) Both A and R are true and R is the not
4) both 1 and 3 correct explanation of A
187. Which of the following is not a drug to reduce 3) A is true but R is false
allergy? 4) A is false but R is true
1) Anti-histamine 2) Adrenalin 191. Statement-I: The productivity and distribution

3) Steroids 4) Animal dander of plants is also heavily dependent on water.


188. Which type of immunity developed if ready- Statement - II: Many plants are also
made antibodies are directly given to protect dependent on sunlight to meet their
the body against pathogen? photoperiodic requirement for flowering.
1) Humoral immunity 1) Both Statements I and II are correct
2) Cell mediated immunity 2) Statement I is correct, statement II is
3) Passive immunity incorrect
4) Active immunity 3) Statement I is incorrect, statement II is
correct
189. A widely diagnostic test for AIDS is
1) ELISA 2) WIDAL 4) Both statements I and II are incorrect
192. Statement-I: 99% of animals and nearly all
3) Western blot 4) both 1 and 2
plants cannot maintain a constant internal
190. Assertion (A): Due to continued close
environment.
inbreeding, reduces fertility and even
productivity. Statement - II: Small animals have a larger
surface area relative to their volume.
103
1) Both Statements I and II are correct 1) A - i, B - iii, C - iv, D - ii
2) Statement I is correct, statement II is 2) A - iv, B - iii, C - ii, D - i
incorrect 3) A - i, B - ii, C - iii, D - iv
3) Statement I is incorrect, statement II is 4) A - iv, B - ii, C - i, D - iii
correct 197. Ecologists have discovered that the value of
4) Both statements I and II are incorrect Z-lies in the range of _____ regardless of the
193. Which one of the following is not the special taxonomic group of the region.
adaptation of the parasites? 1) 0.2 to 0.3 2) 0.3 to 0.4
1) Loss of unnecessary sense organs 3) 0.001 to 0.02 4) 0.1 to 0.2
2) Presence of suckers 198. Statement-I: In troposphere UV-rays act on
3) Low reproductive capacity CFC and releases CI ions.
4) Loss of digestive system Statement - II: Ozone gas is continuously
194. Primary production is expressed in terms of formed by the action of UV rays on molecular
energy by oxygen.
1) Kcal m-2 2) Kcal m-2/ yr-1 1) Both Statements I and II are correct
3) gm-2 4) gm-2/yr-1 2) Statement I is correct, statement II is
195. Statement-I: Different steps in decomposition incorrect
like fragmentation, leaching and catabolism 3) Statement I is incorrect, statement II is
operate simultaneously on the detritus. correct
Statement - II: Humification and 4) Both statements I and II are incorrect
minerlinsation occur during decomposition in 199. Identify the wrong one.
the soil. 1) Water bodies are our lifeline as well as that
1) Both Statements I and II are correct of all other living organisms
2) Statement I is correct, statement II is 2) The government of India has passed the
incorrect water Act, 1974 to safeguard our water
3) Statement I is incorrect, statement II is resources
correct 3) Colloidal material in polluted water are
4) Both statements I and II are incorrect faecal matter bacteria, cloth and paper
196. Match the following. fibers
4) Suspended solids in the water are nitrates,
Column - I Column - II
ammonia and phosphates
A. Mega diversity countries i) 45,000
200. Recycling of E-wastes generates all the
B. India’s land area in the following metals except.
ii) 8.1
world 1) Nickel 2) Gold
C. India’s global species 3) Iron 4) Sodium
iii) 2.4
diversity
D. Total number of plant
iv) 12
species reported from India
104

E- mail : gpccnamakkal@gmail.com Website : www.gpccnamakkal.com

SPP – 1 to 5 (Answer Key)

SPP – 1 (Answer Key & Solutions)

PHYSICS
-27
1. 2) -7 ×10 J x 1.22 λ
R.L = =
Solution: D a
Potential energy U = -PE cos θ 1.22 λ 1.22 x 5500 x10−10
= -10-29 x 103 x cos 45o x= xD= x 4 x105 x103
a 5
= -0.707 x 10-29 J = 53.68 m ≈ 60 m
= -7 x 10-27 J 6. 3) R1+ G
2. 1) 0.02 V/cm Solution:
Solution: V = Ig (G + R1) ---- (1)
potential drep 1 = Ig (G + R1)
Potential gradient =
balancing length V′ = Ig (G + R1 + R2) ---- (2)
1.02 1.02 2 = Ig (G + R1 + R2)
= = = 0.02 V / cm
100 − 49 51 (1) 1 G + R1
⇒ =
(2) 2 G + R1 + R 2
3. 4) E = − 9sin[200π(y+ct)]kɵ v/m
G + R1 + R2 = 2G + 2R1
Solution:
R2 = G + R1
B0 = 3 x 10-8
7. 2) 7.8 ×1014
E0 = CB0 = 3 x 108 x 3 x 10-8 = 9 V/m
Solution:
Direction of wave propagation 3 3
(E x B)||C n3 T n  2 8
T α 2 ⇒ 2 = 2  =  =
Z T1  n1   1  1
B = ˆi C = − ˆj E = − kˆ
4. 4) 28 T2 = 8T1
1 1 1
Solution: n2 = = =
n1λ1 = n2λ2 T2 8T1 8 x1.6 x10 −16
700 x16 = 7.8 x 1014 Hz
n2 = = 28
400 8. 3) λe > λp >λHe++
5. 1) 60 m Solution:
Solution: h h 1
λ= = ⇒λα
p 2mKE m
++
mHe > mP > me
λHe++ < λP < λe
or λe > λp > λHe++
105
9. 4) Graph does not change R
14. 4)
Solution: 3
hc hc φ Solution:
ev 0 = − φ0 ⇒ v 0 = − 0
λ λe e 1 2
P = = (µ − 1) ---- (1)
hc f R
slope =
e 1 1
1.5P = = (µ − 1) ---- (2)
Increasing the frequency of incident radiation f' R'
has no effect on work function. So graph will (2) 1 R
not change. ⇒ 1.5 = x
(1) R' 2
10. 4) increases initially and saturate finally
R R
Solution: R '= =
2 x1.5 3
I – V characteristic of photodiode as follow
15. 1) 6.24 × 106 Pa
Solution:
F 50 x 9.8
P= ⇒ P=
A 3.14 x 25 x10−6
= 6.24 x 106 Pa
first photocurrent increase then attains 16. 2) 25 W
saturation.
Solution:
11. 4) According to first law of thermodynamics
du = dQ – dW = 100 – 75
Solution: = 25 J/s = 25 W
A logic gate is reversible if we can recover 17. 1) Material A
input data from output. Hence NOT gate. Solution:
12. 1) 10 V Strength of a material is determined by the
Solution: amount of stress (load) required to cause
breaking or fracture of the material
corresponding the breaking point.
Material A is stronger than B as it can with
stand more load without breaking than the
material B corresponding to point D.
30 18. 4) 256
i= = 2 mA Solution:
(10 + 5) x103
P = σ AT4 = P α T4
i
VAB = x10 kΩ = 1 x 10-3 x 10 x 103= 10 V 1
2 According to Wein’s law T α
λm
13. 1) 2
4
Solution: 4 4  
 1  P2  λm  λ 
1 L 1 80 x10 −3 ⇒ P α  ⇒ = 1  = 0 
Q= =  λm
R C 100 2 x10−6  λm  P1  2   λ0 
 4 
1 200 np 256
= 4 x104 = =2 ⇒ = ⇒ n = 256
100 100 p 1
106
19. 1) elastic Acceleration of the tip of the clock second’s
Solution: hand a = ω2R = (0.105)2 (0.1)
Linear momentum is always conserved = 0.0011= 1.1 x 10-3 m/s2
whenever the collision is elastic or inelastic. 25. 3) 3) 15 m/s
As the sped of the molecule remains the same Solution:
before and after the collision. So kinetic
energy of the molecule is also conserved.  v+u 
n '= n  
Hence collision is elastic.  v−u 
20. 2) 5.3 m/s  345 + u 
Solution: 480 = 440  
 345 − u 
KE at B = 95% of PE at A
12 345 + u
1
mv 2 =
95
mgh =
2 100 11 345 − u
12 x 345 – 12u = 11 x 345 + 11u
95 95
v= 2x xgxh = 2x x 9.8 x1.5 12 x 345 – 11 x 345 = 23 u
100 100
345 345
= 5.3 ms-1 u= m/s= ⇒ 15 m / s
23 23
21. 3) 6.5 J
26. 4) 90 J
Solution:
Solution:
We know area under F – x graph gives the
work done by the body. Q − Q2 W
η= 1 =
1 Q1 Q1
W = x (3 + 2) x (3 − 2) + (2 x 2) = 6.5J
2 1 W
= Q1 = W x 10 = 10 x 10 = 100 J
Using work energy theorem 10 Q1
∆KE = work done = 6.5 J Q2 = Q1 – W = 100 – 10 = 90 J
v 9
22. 4) 27. 1)
16 16
Solution: Solution:
After the final collision, all blocks are moving 2
v A r 2  4.8  9
together. Let v′ final velocity using A1v1 = A2v2 ⇒ 1 = 2 = 22 =   =
momentum conservation. v 2 A1 r1  6.4  16
v 28. 4) X-rays, IR rays, Radio waves
mv =16 mv ' ⇒ v ' = Solution:
16
23. 4) FA V-1 0 X-rays, IR rays, Radio waves
Solution: 29. 3) 3
F Solution:
stress
Y= = A = FA −1v 0 A = 30 ; i1 = 60o ; d = 30
strain ∆l
l i1 + i2 = A + d
-3
24. 1) 10 60 + i2 = 30 + 30
Solution: i2 = 0 ⇒ r2 = 0
2 π 2π r1 + r2 = A ⇒ r1 = A = 30o
ω= = = 0.105 rad / s
T 60 sin i1 sin 60o 3 2 3
= = o
= x =
sin r1 sin 30 2 1 1
107
a1 L di
30. 1) =2 But VA = VB ⇒ =− 4
a2 dt
Solution: di di
10 x10−3 = − 4 ⇒ = 400 A / s
Force is same in magnitude for both dt dt
a m 1 2 35. 4) It is I – V characteristics of photodiode in
m1a1 = m2a2 ⇒ 1 = 2 = = reverse bias
a 2 m1 0.5 1
Solution:
1 I – V characteristic of photodiode lies in III
31. 4)
4 Quadrant.
Solution: Q2
h h m c 36. 3)
λe = λv ⇒ = ⇒ e= µπE 0 R
me v m v c mv v Solution:
1 KQ1 KQ 2
2
Ee 2 me v 1 c  v2  v VA = +
= = x x = R 4R
E v m v c 2 2 v  c2  2c KQ1 KQ 2
VB = +
1.5 x108 1 4R 4R
= = 3KQ1 3Q1
2 x 3 x108 4 ∆V = VA − VB = =
4R 16 πε 0 R
32. 4) 7 : 108
Solution: 37. 2) 590µC
Solution:
1  1 1  3R
=R  2 − 2 = q2 20 V
λ1 1 2  4 15 F
1  1 1  7R
=R  2 − 2 = q3
λ2  3 4  144 750 C
V1
8 F1 20 V
λ1 7R 4 7
= x =
λ 2 144 3R 108 q2 + q3 = 750 µC V2 = V3 = 20 V
AB + AC q3 = C3V3 = 20 x 8 = 160µC
33. 2)
3 q2 = 750 µC – 160 µC = 590 µC
Solution: 38. 1) 40 cm
m1 = m2 = m3 = 1 kg Solution:
R l1 25 1
= = = ---- (1)
S 100 − l1 75 3
l
ρ
ρl x 2
m1x1 + m 2 x 2 + m3 x 3 R '= 2 = = 2R
x cm = A A
m1 + m 2 + m3
4
Where x1 = 0, x2 = AB, x3 = AC R' l2 2R l2
34. 1) 400 A/s = ⇒ = ---- (2)
S 100 − l2 5 100 − l2
Solution:
2 l2
VA −1(2) + 6 +
Ldi
= VB (1) ln (2), =
dt 3 100 − l2
l2 = 40 cm
108
39. 4) 0.75 A/m L = Iω = 3 ml2ω
Solution:
According to curie law
1
χα χ1T1 = χ2T2
T
I I I
But χ = ⇒ 1 T1 = 2 T2
B B1 B2
6 I 0.3
⇒ x 4 = 2 x 24 ⇒ I 2 = = 0.75
0.4 0.3 0.4
40. 4) 6 : 5 44. 1) 2200 Hz
Solution: Solution:
µi v=2n(l 2 – l1)
B= 0 ⊙
4πR v 330
n= =
3π 2( l2 − l1 ) 2(24.5 − 17) x10−2
B1 i1 θ1 R 2 2 2 4 6
= x x = x x = 330
x100 = 2200 Hz
B2 i 2 θ 2 R1 3 5π 2 5 =
2 x 7.5
3
41. 3) 0.32 m 45. 4) The PE is equal to KE of the oscillation at t
T
Solution:
f 0.4 2
m= ⇒ 5= Solution:
f −u 0.4 − u
From graph equation of SHM is x = A cos ωt
0.4
0.4 − u = = 0.08 3T
5 1) At particle at mean position so
u = 0.4 – 0.08 = 0.32 m 4
acceleration = 0, Force = 0
42. 2) 4 J
Solution: 2) At T particle again at extreme position so
acceleration is maximum
If ω be the angular velocity of CM of loop
T
then ω= =
v 0.20
= 0.10 rad s −1 3) At t = particle is at mean position so
r 2 4
velocity is maximum, acceleration = 0
I = mr2 = 100 x (2)2 = 400 kgm2
Total KE of ring = Rotational KE + 4) When KE = PE
Translational KE 1 1
k(A 2 − x 2 ) = kx 2
1 1 2 2
= Iω2 + mv 2
2 2 A
A2 = 2x2 ⇒ x = ±
1 1 2
= x 400 x (0.1) 2 + x100 x (0.2) 2 = 4 J
2 2 A T A
2 = A cos ωt ⇒ t = and x = ±
43. 3) 3 ml W 2 2 2
Solution: T
Angular momentum L = Iω But t = , x = -A which is not possible
2 2
2
 l   l 
( )
2
I = m(0)2 + m   + m  +m 2l
 2  2
= 3 ml2
109
P 25G
x y 49. 2) −
46. 1) r
z Solution:
V
If P is a point of zero intensity then
Solution: r
x= (from 4 kg)
Process x y → isobaric expansion 9
+1
Process yz → isochoric (pressure decrease) 4
Process zx → isothermal compression
47. 1) 1.06 m/s2
Solution:
r 2r
x= =
3 5
+1
2
∴ Net potential at point P is = V1 + V2
−G(4) −G(9)
∴ V1 = ; V2 =
F cos 30 − f F cos 30o −µN x (r − x)
a= =
m m −G4 −G9 25G
V= + =−
Fcos 30 −µ (mg + Fsin 30o )
o  2r   2r  r
=   r − 
m 5  5
20 cos 30o − 0.2 (50 + 10) 50. 2) A
= Solution:
5
Mean life is less for A.
10 3 −12 17.32 −12 5.32
= = = = 1.06  dN 1
5 5 5 ∵ = − λN & τ = 
 dt λ
R 2W2
48. 1) dN dN
2g = − λN ⇒ (t) = − λNt
dt dt
Solution: 1
Wequation = Wh Area = -λNt ⇒ Area ∝ x Nt
τ
mg0 = mgh (θ = 0o) As time is same for A and B mean life is
 2h  inversely proportional to area under the curve
g − Rω2 cos 2 0o = g  1 − 
 R  and time axis.
2hg R 2 ω2
Rω2 = ⇒ h=
R 2g
CHEMISTRY
51. 2) MnO2 3
52. 1)
Solution : 8
Solution :
a
x 3b
Pc Vc 2 3
M Zc = = 27b =
E= RTc 8a 8
2 Rx
27bR
110
1 1
1) Mg2C3 + 2H2O →2Mg(OH)2 +H3C –C ≡ CH
53. 4) Rate = K 3 K eq2 [CHCl3 ] [ Cl2 ] 2
C3-4 contains [C – C ≡ C]-4 = 2σ and 2π bonds
Solution : 2) ‘Mg’ is more dense than ‘Ca’
R = K3 [CHCl3] [Cl•] 3) Fact
2 1

K1 Cl  K 2 1 1 4) Larger anions will be stabilized by larger
= ⇒  Cl•  =  1 [ Cl2 ] = K eq2 [ Cl2 ] 2 cations
K2 [ Cl2 ]  K2 
1 1
59. 4) Availability of 4f electrons result in the
R = K3K 2
eq [CHCl3 ] [ Cl2 ] 2 formation of compound in +4 state for all
the members of the series.
54. 2) Adsorption is multi layered
Solution :
Solution :
(1) and (3) due to lanthanoid contraction
Chemisorption is single layered.
(2) ionization potential for the formation of
55. 4) a, b, c and d
La+3 is comparatively low, hence ‘+3’ state
Solution :
(4) +4 state formation requires very high
Bromine water can be used to differentiate energy hence incorrect
aldose from a ketose.
60. 4) TFFT
All monosaacharides, whether aldoses or
Solution :
ketoses, are reducing sugars.
Boiling point SbH3 > NH3 > ASH3 > PH3
Sucrose is reducing sugar
Lewis basic strength : NH3 > PH3 > ASH3 >
A pair of diastereomeric aldoses which differ in SbH3 > BiH3
configuration about C – 2 termed as pair of
epimers. 61. 3) KO2
56. 4) All are correct Solution :
3
Solution : 2KO2 + CO2 → K2CO3 + O2
Total electron in C2 = 12 2
62. 2) Secondary
M.O configuration σ1s2 σ*1s2 σ2s2 σ*2s2π2px2
π2py2 Solution :
Nb = 8, Na = 4 α-helix and β-pleated sheet structure are
related to secondary structure of proteins
8−4
B.O = =2
2 63. 3)
It is diamagnetic,
Solution :
Bond order = 2, but both bonds are π bonds
−CF3 is a meta directing group (it is a
[No sigma bond is present] deactivating group) towards electrophilic
57. 3) Element R aromatic substitution reaction.
Solution : 64. 3) NH2NH2.HCl
After losing three electrons in element R, there Solution :
is sudden huge increase in I.E from 2751 to To give the result with Lassaigne’s test
11584 kJ/mole. It means that after three compound must have 'C' atom along with 'N'.
electrons some stable configuration is
65. 1) -920.47 kJ
achieved. This is applicable to group 13
elements with outer configuration of ns2np1 Solution :
58. 2) Density of Mg is less than ‘Ca’ 4NH3(g) + 5O2(g) → 4NO(g) + 6H2O(l)
Solution : ∆n = 4 – (4 + 5) = - 5
111
∆H = ∆U + ∆n RT 71. 4) Different with 1, 0 and 2 lone pair of
8.314 electron respectively
= -908 + (-5) x 300 Solution :
1000
••
= -908 – 12.471 S F4 : 1lp, sp3d, sea-saw
= -920.471 CF4 : 0lp, sp3, Tetrahedral
66. 2) 50 amp
: 2lp, sp3d2 square planar.
Solution :
72. 4) 100ppm
w x 96500 10.8 x 96500
i= = Solution:
Ext 108 x193
n x100
= 50 amp DOH = x106 ppm
vol.of water (ml)
67. 1) H2O < H2S < H2Se < H2Te – acidic character
10−3 x100 x106
Solution: = = 100 ppm
1000
Correct orders of given properties
73. 3) III > I > II > IV
1) H2O < H2S < H2Se < H2Te – acidic character
Solution:
2) H2O > H2S > H2Se > H2Te – thermal stability
3) H2O<H2S<H2Se<H2Te – reducing character
4) H2O > H2S > H2Se > H2Te – boiling point
68. 2) 20 days
Solution:
t 1
= 50% =
t 75% 2 74. 1) CF2Cl2
10 1 Solution:
= = Chlorofluoro carbons responsible for depletion
t 75% 2
of ozone layer
t75% = 20 days • •

69. 4) Phenol-aniline, +ve deviation CF2Cl2 → C F2 Cl + C l
Solution : • •
Cl + O3 → C lO + O 2
Phenol-aniline, -ve deviation due to strong
1
interactions. 75. 4) LiNO3 ∆
→ LiNO2 + O2
70. 1) 2 : 1 2
Solution : Solution :
From De-Broglie’s equation ∆
1) NH4NO3  → N2 O
− H 2O
h
λ= 2) NH4NO2

 → N2
mv − H 2O
λA mBvB
= 3) (NH4)2Cr2O7 ∆ → N2
λB mA vA Here cation is oxidized by anion
VB 0.02 2 3
mB = 5mA ⇒ mB/mA = 5 ; = = 4) 2LiNO3 ∆ → Li2O + NO2 + O2
VA 0.05 5 2
λA 2 All other ammonium salts like NH4Cl,
= 5 x ⇒ 2 :1 (NH4)2CO3, (NH4)2SO4 on thermal
λB 5
decomposition give ammonia.
112
76. 3) Ofloxacin 84. 4) Orlon, Styrene ⇒ condensation & fibres
Solution : Solution :
1) Pencilin G is narrow spectrum and Orlon & Styrene ⇒ addition polymers
bactericidal Orlon is a fibre, Styrene is plastic
2) Chloramphenicol is bacterestatic and broad
spectrum
3) Ofloxacin is both bactericidal and broad 85. 3)
spectrum antibiotic
77. 4) Sn, Pb are refined by distillation
Solution : Solution :
Low melting metals like Sn, Pb are refined by
liquation
Low boiling metals like Zn, Hg are refined by
distillation
78. 4) Inflammable and does not conduct electricity
Solution :
Though inert gases are inflammable but
conduct electricity and show fluorescence
79. 4) M(ab)3
Solution :
Ma3b3 and Ma4b2 shows only GI but not OI
M(aa)3 shows only OI but not GI
M(ab)3 shows both GI and OI

80. 4)
Solution :
86. 1) 30%
4 is more stable with 12 ∝-hydrogens Solution :
81. 4) All are equal at STP 1mole ______ 22400 mL
Solution :  x 10 − x 
 +  ______ 4480 mL
All are equally acidic in water due to leveling  30 70 
effect x 10 − x 4480 1
82. 4) CH3COOH + = =
30 70 22400 5
Solution : ⇒ x = 3g, % C2H6 by mass = 30%
CH3COOH does not give idoform test 87. 4) 750 K
83. 3) IV < I < III< II Solution :
1 3 
∑S − ∑ SR = 50 −  x 60 + x 40 
0
Solution : ∆S0 = 0
P
1  2 2 
λ ∝ = 50 – (30 + 60) = -40 J K-1 mol-1
(Absorption ) Strength of ligand
∆H 0
Ligand strength order : Reaction will be in Equilibrium if T =
∆S0
Cl- < H2O < NH3 < CN- −30 x 103
T =
−40
113
88. 2) 0.433  i)O3
→ Ph-CHO +
Ph HC ii) Zn + H 2 O
Solution :
M normal 94
i= = = 0.783
M Obs 120 
dil.NaOH


1 
1 +  − 1 α = 0.783
n 
Aldol condensation
1 
1 +  − 1 α = 0.783 94. 1) 2.23 V
2 
Solution :
α = 0.433 +2
Mg (s) + Sn (aq) → Mg +(aq)
2
+ Sn (s)
Nernest equation
89. 3)
0.0591 [Mg +2 ] 
Ecell = ( E oRHS − E LHS
o
)− log  +2 
Solution : n  [Sn ] 
0.0591  10−2 
Ecell = [-0.14 (-2.34)] - log  −1 
2  10 
90. 1) b < a < c Ecell = 2.20 + 0.03 = 2.23V
Solution :

95. 3)
Rate of ESR

91. 4) Violet Solution :


Solution :
1
∆o ∝ chelation ∝ frequency ∝
wave length
∴ ∆o of [Ni(en)3]+2 > [Ni(H2O)2en2]+2
[Ni(en)3]+2 absorb more frequency radiation.
92. 3) Both II & III
Solution :
I) in pyro silicates : (1 corner O-atom per
tetrahedron is shared) 96. 4) 2.408 × 1020
II) In cyclic silicates : (2 corner O-atoms per Solution:
tetrahedron is shared) 1mol CaCl2  NaCl
→ 1mol cation vacancy
III) Single chain silicate : (2 corner O-atoms 4 x10−2
are shared) mol of CaCl2 → ?
100
IV) 3D silicate (4 corner O-atoms are shared)
Number of cation vacancies
93. 2) = 4 x 10-4 moles of cation vacancy
= 4 x 10-4 x 6 x 1023
= 2.4 x 1020 cation vacancies
Solution:
114
99. 2) 0.266
Solution :
97. 3) Number of moles of PCl3 dissociated at
equilibrium = 2 x 40/100 = 0.8
PCl5 ↽ ⇀ PCl3 + Cl2
Solution : (initial) 2 0 0
(At equilibrium) (2-0.8) mol 0.8 mol 0.8 mol
1.2
[PCl5] = = 0.6 ML-1
2
0.8
[PCl3] = [Cl2] = = 0.4 ML-1
2

∴ KC =
[ PCl3 ][Cl2 ] = 0.4 x 0.4
[ PCl5 ] 0.6
= 0.267 mol/dm3
100. 3) I and III
1
 K eq  x+y Solution :
98. 3) α =  x + y −1 x y 
C . x .y 
Solution :
A x B y ↽ ⇀ xA + y + yB x −
C 0 0
C - Cα xCα yCα
( xCα ) ( yCα )
x y

K eq =
C (1 − α )
x x .y y C x + y . α x + y
K eq =
C Both I and III hence exhibit tautomerism.
K eq
αx +y =
C x + y −1 . x x . y y
1
 K eq 
α = x+ y − 1 x y 
x+y

C .x .y 

BOTANY
101. 1) Unicellular fungi – Fragmentation 108. 3) cry I Ab
102. 1) Ascomycetes 109. 2) UAA, UAG and UGA
103. 4) All of the above 110. 3) iii only
104. 3) formation of endosperm 111. 1) Lt = Lo + rt
105. 1) X-ray crystallography 112. 2) i and iv
106. 1) Hershey and Chase 113. 2) Phosphorylation
107. 3) Helicase 114. 4) nitrate
115
115. 3) Magnesium 133. 4) Pod length
116. 1) Simple diffusion – Transport saturation 134. 2) 50%
117. 3) Cytoskeleton 135. 1) Methylophilus methylotrophus
118. 4) Lysosomes – DNA synthesis 136. 1) Marchantia
119. 2) polymer of aminoacids 137. 1) 2 3 4 1
120. 4) The right end of glycogen is called non- 138. 3)Hydrophily occurs in water hyacinth and
reducing end water lily
121. 3) Valvate 139. 4) being a diploid tissue
122. 2) Monocarpellary and superior ovary 140. 4) Inner membrane Inter membrane space
123. 3) Metaphase 141. 2) III IV I II
124. 2) Non-sister chromatids of homologous 142. 4) Five
chromosomes.
143. 3)
125. 2) RrYy

144. 3) Dicot stem


126. 3)
145. 4) A, B, C and D
127. 3) all transformed cells contain vector
146. 3) iv, i, ii, iii, vi, v
128. 2) These are small, circular, DNA molecules
147. 1) human gene have intrans
with their own ori site
148. 4) all of the above
129. 1) Vaccine safety testing
149. 4) Statement – I is incorrect, statement - II is
130. 1) Nucleopolyhedrovirus (NPV)
correct
131. 3) Double fertilization
150. 1) Infectious agent that causes madcow
132. 3) n:1 disease

ZOOLOGY
151. 2) Both A and R are true and R is the not 161. 4) Eosinophils are the type of granulocytes
correct explanation of A which can resist infections
152. 4) presence of skull and vertebral column 162. 4) Both the A and R are false
153. 4) cnidarians 163. 3) A - iv, B - iii, C - i, D - ii
154. 2) Development is indirect 164. 3) 200 mL/min
155. 4) All of these 165. 2) H - zone
156. 1) Both Statements I and II are correct 166. 2) 8th - 19th vertebrae
157. 4) i and iv only 167. 3) Malleus, Stapes
158. 2) ileo-caecal valve 168. 1) Crista
159. 1) 5 169. 1) Both Statements I and II are correct
160. 2) Branchial respiration 170. 4) iodothyronines
116
171. 1) morula 187. 4) A - ii, B - iii, C - iv, D - i
172. 1) A - ii, B - iv, C - i, D - iii 188. 1) It will decline
173. 4) 32 189. 1) Brood parasitism
174. 2) A - ii, B - iii, C - iv, D - i 190. 1) Both A and R are true and R is the correct
175. 3) first trimester explanation of A
176. 2) A - i, B - iii, C - ii, D - iv, E - v 191. 3) Inverted pyramid
rd
177. 2) 3 child 192. 2) animals
178. 2) Rh antibodies 193. 1) Habitat loss and fragmentation
179. 3) multiple phenotypes and single genotype 194. 1) A - i, B - ii, C - iii, D - iv
180. 2) adaptive radiation 195. 4) both 1 and 2
181. 3) Homology 196. 3) DDT causes thinning of egg shell and
182. 3) natural selection premature breaking of eggs in mammals
183. 3) Statement I is incorrect, statement II is 197. 3) Statement I is incorrect, statement II is
correct correct
184. 4) Intestinal perforation 198. 1) Both Statements I and II are correct
185. 3) Ringworm 199. 1) Amrita Devi
186. 1) Both A and R are true and R is the correct 200. 2) Statement I is correct, statement II is
explanation of A incorrect

SPP – 2 (Answer Key & Solutions)

PHYSICS
1. 1) [M-1L-3T4A2] Zero correction = -0.07 cm
Solution: Length of cylinder
We know that, force between two charges q1 = MSR + VC x LC ± correction
and q2 placed at distance ‘r’ is given as = 3.1 + 7 x 0.01
1 q1q 2 qq = 3.07 cm
F= ⇒ ε0 = 1 2 2
4πε 0 r 2
4πFr 3. 3) average velocity is zero
(AT) (AT) Solution:
⇒ ε0 = = M-1L-3T4A2
[MLT −2 ][L2 ] According to given displacement time graph,
in a certain time interval ∆t, displacement of
2. 3) 3.07 cm
the object becomes zero.
Solution:
Hence,
L.C = 1 MSD - 1 VSD
Total displacement
9 1 1 Average velocity =
= 1 MSD - MSD = MSD = mm Total time int erval
10 10 10
= 0.1 mm ⇒ L.C = 0.01 cm =0
∴ Zero error = +7 x 0.01 = 0.07 cm
117
o
4. 2) 40 20
7. 4) 0 m, m
Solution: 11
u 2 sin 2θ Solution:
Range R = xcm = 0
g
A y + A 2 y 2 0 + 16 x 5
For same projection velocity, y cm = 1 1 =
A1 + A 2 28 + 16
R ∝ sin 2θ
80 20
i.e., higher value of sin 2θ, range will also be ⇒ y cm = =
44 11
greater.
8. 2) Mass of linear motion
Among the given values of project angle, the
Solution:
value of sin 2θ is greater for θ = 40o.
Mass is measure of inertia in translatory
So, range will be highest for projected angle motion. Moment of inertia is a measure of
θ = 40o rotational inertia. Therefore, the moment of
5. 3) 1 : 5 inertia in rotational motion is equivalent to
Solution: mass of linear motion.
3Gm 2
9. 3)
2l
Solution:
T1 = 3a --- (1) Work done, W = change in potential energy
= U f - Ui
 3Gm 2   3Gm 2 
= − −− 
 2l   l 
T2 – T1 = 12 a
3Gm 2
⇒T2 = 15 a (∴ From (1)) --- (2) =
2l
T 3a 1
∴ 1= = 10. 1) 0.9 Mpa
T2 15a 5
Solution:
6. 1) 2 A A
sin 25 = ⇒A'=
Solution: A' sin 25
F
v = u + at = at = t
m
1 1 F2 F2 t 2
⇒ K.E = mv 2 = m 2 t 2 =
2 2 m 2m
Since KE and F are same
t2 Fsin 25
Normal stress across a section =

m
= constant
( A sin 25)
⇒t∝ m
Fsin 2 25
=
tA 20 A
= =2
tB 5 Given, cos-10.6428 = 50
⇒ cos 50 = 0.6428
118
2
We can write, 1 – 2 sin 25 = cos 50 As argon is a monoatomic gas, its molecules
⇒ 2 sin225 = 1 – 0.6428 = 0.352 possess only translational kinetic energy, that
⇒ sin225 = 0.1756 is share of translational and rotational energies
are 100 % and 0 % respectively.
Fsin 2 25
∴ Normal stress across a section = 15. 1) A, C, E, G
A Solution:
= 5 x 106 x 0.1756 At the extreme position of SHM, the velocity
= 0.893 x 106 of the particle becomes zero momentarily.
= 0.9 MPa 16. 1) 1 : 4
π Solution:
11. 1) less than
2 Fundamental frequency in closed pipe is
Solution: v 1 γRT
A liquid wet the solid when angle of contact of ν0 = =
4l 4l M
π
given pair of solid and liquid less than that . Since ν0 and T are same.
2
12. 1) 0.130 l M = cons tan t
Solution: l1 M2 2 1
= = =
Let m be the mass of steam condensed. l2 M1 32 4
1 2q
17. 3)
4πε 0 3 3R 2
Heat lost by steam
Solution:
= m x 540 x 4200 + m x 4200 x 20
Electric field at a point on the axis of ring is
= m x 4200 x 560 J
1 qx dE
Heat gained by calorimeter and water E= ; = 0 for maximum E
4πε 0 (R + x )
2 2 3/ 2
dx
= (0.02 x 103 + 1.1 x 103 x 1) 65
= 1.12 x 103 x 65 cal R
At x = ± , E is maximum
= 1.12 x 65 x 4.2 x 103J 2
From the principle of calorimeter,  R 
q 
Heat lost by steam = Heat gained by 1  2
calorimeter and water. Emax =
4πε0  2 R 2 3/ 2
m x 4200 x 560 = 1.12 x 65 x 4200 R + 
 2 
1.12 x 65
⇒ m= = 0.13kg 1 2q
560 =
13. 3) 102 πJ 4πε0 3 3R 2
Solution: 18. 2) 5
In cyclic process, dU = 0 Solution:
dQ = dW E = − ∇V
= Area bounded by cycle  ∂v ∂v ∂v 
= π x 10 x 103 x 10 x 10-3 = −  ˆi + ˆj + kˆ
 ∂x ∂y ∂z
( ˆ ˆ
 = − 4i + 3j − 0

)
= 100 π J = 102 πJ
14. 4) 100 % and 0 % | E | = 16 + 9 = 5 N/C
Solution:
119
19. 3) 30 Ω 23. 3) 65 Wb
Solution: Solution:
10 10 10 φ = B.A = (5iˆ + 10ˆj + 6k).(3i
ˆ ˆ + 2ˆj + 5k)
ˆ
I I2
A C = 15 + 20 + 30 = 65 Wb
I1 I2
24. 3) 0.01 H
10 10 Solution:
B D L = L1 + L2 + 2M
10 I1 10 I 10 ⇒ 0.06 = 0.01 + 0.03 + 2M
We know that, current always flows from high ⇒ 2M = 0.02 ⇒ M = 0.01 H
potential to low potential. 25. 1) increases
20
Req = 10 + + 10 = 30 Ω Solution:
2 As average power of an AC circuit,
20. 3) Heat produced in a conductor is
R
proportional to charge P = vrms Irms cos φ = vrms I rms  
Z
Solution:
The heat produced is given by, In CR series circuit with an AC source,
H = i2Rt
2
--- (1)  1 
Net reactive impedance, Z = R 2 +  
and current is given by i =
q
--- (2)  ωC 
t So, increasing the value of ω, then impedance
From equation (1) & (2), we get will decreases. Hence, power consumption
q2R across CR series circuit will increases.
H= 26. 2) 10-6 F
t
21. 3) four times of its first value Solution:
Solution: dQ dV
id = =C
µi dt dt
For first coil, B = 0 , l = 2πr
2r i 10−1 x10−3
C= d = = 10-6 F
Same length bent in the form of two loops  dV  100
 
l = 2(2πr′)  dt 
r 27. 1) 15 cm, concave
⇒ 2πr = 4πr′ ⇒ r ' =
2 Solution:
µ Ni µ Ni µi 1  1 1   1  1 1 
B' = 0 = 0 = 4 0 = 4 B = ( g µ a − 1)  − =  − 1  + 
2r  
r 2r f  R 1 R 2   1.5   10 10 
2 
 
2
0.5  2  1
22. 4) 5 rad/µA = −  =− ∴ f = -15 cm
1.5  10  15
Solution:
28. 3) 25o
θ BAN
Current sensitivity, = Solution:
I C
β f0 β 100
−4
5 x10 x100 = ⇒ =
= = 5 x 106 rad/A α fe 0.5 2
10−8
⇒ β = 25o
θ
(or) = 5 rad/µA
I
120
o
29. 4) 90 After 2 half life period 25 %
Solution: After 3 half life period 12.5 %
When light incident on the surface with After 4 half life period 6.25 %
polarizing angle, the reflected and refracted After 5 half life period 3.125 %
rays are perpendicular to each other. It is 5 x 12.6 years + 7 years
30. 2) 75 % ⇒ Total 70 years only
Solution: 34. 1) 0.53 A
Let E1 = E, then E2 = 16 E Solution:
h From figure D1 forward bias and D2 reverse
λ=
2mE bias. So current passing through 15 Ω only.
λ2 E1 E 1 V 8
⇒ = = = ∴ i= = = 0.53 A
λ1 E2 16 E 4 R 15
35. 3) 1250
λ 1
⇒ 2 −1= −1 Solution:
λ1 4
Voltage gain = β x Impedance gain
λ 2 − λ1 −3
⇒ x100 = x100 = -75 % ⇒ 50 = β x
200
⇒ β = 25
λ1 4 100
ν ∴ Power gain = β2 x Impedance gain
31. 2)
3 200
Solution: = (25)2 x = 1250
100
From photo electric equation, 36. 2) 0.98 m/s2
eV0 = hν - hν0 --- (1) Solution:
eV0 hν Static friction force between 10 kg and 40 kg
= − hν 0
4 2 block is fs = µsN
⇒ eV0 = 2hν - 4hν0 --- (2) = 0.6 x 10 x 9.8 = 58.8 N
From equations (1) & (2), we get Here, we see that the applied force (F = 100
hν - hν0 = 2hν - 4hν0 N) is greater than friction force, hence 10 kg
block will start motion due to application of
⇒ 3hν0 = hν
100 N force.
ν
⇒ ν0 =
3
32. 4) 122.4
Solution: Due to motion, kinetic friction force
13.6 Z2 fk = µkN = 0.4 x 10 x 9.8
En = − eV = 39.2 N
n2
40 kg body experiences a force of
13.6 x (3)2
⇒ E0 = − = - 122.4 eV f 39.2
(1)2 fk = 39.2 N ⇒ a = k = = 0.98 m/s2
40 40
So, ionization energy = 122.4 eV
33. 4) 70 years back 37. 3) 50 m/s
Solution: Solution:
After one half life period, the activity of Velocity at any point P,
tritium becomes 50 % v p = v12 − 2gr(1 − cos θ)
121
F 40
⇒ α= = = 25 rad/s2
MR 4 x 40 x10−2
39. 3) decreases by 4%
Solution:
Gm 2
F1 =
Tension at any point P, r2
mv2P If 20 % of mass is transferred from one body
TP = + mg cos θ
r to another, then their masses are
Tension is maximum at lowest point A and 80 120
minimum at highest point B. m1 = m, m2 = m
100 100
mv12 4 6
Tmax = + mg (∵ At A, θ = 0) = m = m
r 5 5
mv22  4  6 
Tmin = -mg + G  m m
r
Final force, F2 = 
5  5  24
m = F1
= -mg + ( v12 − 4gr ) (∵ At B, θ = 180o)
2
r 25
r F −F 1
mv 2 mv12 ⇒ 2 1 x100 = − x100 = - 4 %
= − mg + 1 − 4mg = -5mg + F1 25
r r
2
40. 3) 1 : 3 : 5
mv1
Tmax + mg Solution:
= 4⇒ r =4 ρL 2 2
Tmin mv12 ∆t = (y 2 − y1 )
−5mg + 2kθ
r
2 ∆t1 : ∆t2 : ∆t3 =(y2 – 02) :(4y2 – y2) :(9y2 – 4y2)
v 4v 2
⇒ 1 + g = − 20g + 1 = y2 : 3y2 : 5y2
r r
2 =1:3:5
3v
⇒ 1 = 21g ⇒ v1 = 7gr 41. 3) Change in internal energy of gas is 157 cal
r
∴ Speed of stone at highest point, Solution:
CP – CV = R
v 2 = v12 − 4gr = 7gr − 4gr = 3gr
⇒ CP = R + CV = 1.98 + 5.03
⇒ v2 = 3 x10 x
5
= 50 m/s = 7.01 cal mol-1K-1
3 Heat supplied = nCPdT
38. 3) 25 rad/s2 10
Solution: = x 7.01 x 100 = 219 cal
32
Torque (τ) = Iα Work done by gas = PdV = nRdT
⇒ FR = MR2α
10
= x 1.98 x 100
32
= 61.9 cal = 62 cal
Change in internal energy,
dU = dQ – dW = 219 – 62 = 157 cal
122
8L0
42. 2)
g
Solution:
Mass of element, dm = µ0y dy
Mass of the string upto the length y is F2 2kλ1λ 2
y ⇒ =
µ0 y 2
l R
m = ∫ dm = ∫ µ 0 ydy ⇒ m =
2 F F F 2kλ1λ 2
0
Also 1 = 2 = =
Tension at the position of element, l l l R
44. 3) 8 kV
µ y2
T = mg = 0 g Solution:
2 As q = CV,
(Q1)max = 10-6 x 6 x 103 = 6 mC
(Q2)max = 3 x 10-6 x 4 x 103 = 12 mC
However in series charge is same so maximum
charge on C2 will also be 6 mC (And not 12
mC) and potential difference across it
T 6mC
Speed of the pulse at this location, v = v2 = = 2 kV
µ 3µF
and as in series V = V1 + V2
µ0 y2 So, Vmax = 6 kV + 2kV = 8 kV
g
dy 2 g
⇒ = = y 45. 3) 0.6 L
dt µ0 y 2
Solution:
1 g Let R be the resistance in primary circuit is
⇒ dy = dt 2.5
y 2 I=
10 + R
L0 t
g
∫ y dy = 2 ∫0
−1/2
⇒ dt V R
Potential gradient   = I  
0
L L
L0
 y1/2  2.5  10  25 V
 = g (t)0t =  = ⇒ E =  l
⇒ 10 + R  L  (10 + R)L L
 1  2
 2 0 25 L
1= x ⇒ 25 = 20 + 2R
g 8L0 (10 + R)L 2
⇒ 2 L0 = t ⇒ t=
2 g ⇒ R = 2.5 Ω
2λ1λ 2 Now resistance doubled,
43. 2) k R′ = 2 x 2 .5 = 5 Ω
R
New potential gradient (x)
Solution:
25 25 5
Force on l length of the wire (2) is = = =
(10 + 5)L 15 L 3L
2kλ1
F2 = QE1 = (λ2l) 1 3L
R E = xl2 ⇒ l2 = = = 0.6 L
( 53 L) 5
123
+ +2
46. 2) He and O will be deflected equally 48. 1) 1 cm/s in leftward direction
Solution: Solution:
2mE m 1 1 1
r= ⇒rα From mirror formula, = +
Bq q f v u
1 q 1 1 1 1 2
Deflection, y α ⇒ y α ⇒ = − ⇒ = ⇒ v = 5 cm
r m 10 v 10 v 10
e e 2e  dv  v2
yH+ : yHe+ : yO+2 = : : Image speed   = − 2 (object sped)
m 4m 16 m  dt  u
25
1 1 = − x 4 = -1 cm/s
= 1: : = 2 : 1 : 1 100
2 2
49. 2) 6000 Ao
47. 3) 2 cms-1
Solution:
Solution:
λD
As bridge is balanced, Req = 3Ω β=
d
Total resistance = 3 + 1 = 4Ω
λ (D1 − D 2 ) d(β1 − β2 )
⇒ β1 − β2 = ⇒ λ=
d D1 − D 2
10−3 x 3 x10−5
⇒ λ= = 0.6 x 10-6
5 x10−2
⇒ λ = 6000 Ao
ε Blv 50. 1) 79.0
i= = Solution:
R R
The ionisation energy of helium atom
2 x10 x10 −2 x v
⇒ =1x10 −3 = 13.6 Z2 = 13.6 x (2)2 = 54.4 eV
4 So, total energy needed to remove both the
⇒ v = 2 x 10-2 m/s = 2 cm/s electrons from helium atom = 24.6 + 54.4
= 79 eV

CHEMISTRY
51. 1) C6H8N2 52. 2) n = 3, l = 2, m = –3, s = – ½
Solution : Solution :
C H N m = −l to +l through zero, for l = 2, values of
9 1 3.5 m will be -2, -1, 0, +1, +2
Moles m cannot have the value −3
12 1 14
53. 4) O−(g) + e− → O2−(g)
3 1 Solution :
: 1 :
4 4 O− ion repel the incoming electron thus energy
3 : 4 : 1 is required to add incoming electron.
108 54. 1) NO → NO+
n= =2 Solution :
54
MF = (EF)n = (C3H4N)2 = C6H8N2 For NO (e−s = 15) BO = 2.5; Paramagnetic
For NO+ (e−s = 14) BO = 3
Diamagnetic
124
55. 3) 32 J 61. 2) 3-Ethyl-4-methylhexane
Solution: Solution :
q = 40 J
w = -8 J (Work done by the system)
∆E = q + w = 40 - 8 = 32 J
56. 2) −, +, −
Solution :
For combustion reaction ∆H = −ve 62. 1) Identical
∆n = (16 + 18) − (25 + 2) = +7 Solution :
So, ∆S = +ve Identical
∴ ∆G = −ve 63. 3) Iodoform test
57. 3) F2 and ClO4− Solution :
Solution : H+ / Hg2+
CH3CH2COCH3
F2 being most electronegative element cannot (Ketone with -COCH3)
exhibit any positive oxidation state. In ClO4− CH3CH2C CH
chlorine is present in its highest oxidation state
i.e., +7. It doesn’t show disproportionation BH3
CH3CH2CH2CHO
H2O2/OH-
reaction.
58. 1) Mg2+, Ca2+ 64. 2) B
Solution : Solution :
As Na+ ions in sodium zeolite are replaced by
Ca2+, Mg2+ ions present in hard water, these
two ions will not be present.
59. 4) Ba(OH)2 < Sr(OH)2 < Ca(OH)2 < Mg(OH)2 65. 3) Hg(OCOCH3)2, H2O / NaBH4, NaOH
Solution : Solution :
The stability of alkaline earth metal OMDM reaction addition of H2O is according
hydroxides decreases with decrease in lattice to Markovnikoff’s rule.
enthalpy as the size of alkali earth metal 66. 3) A and B
cations increases down the group.
Solution :
60. 3) RSiCl3 Catalytic converters prevent the release of
Solution : nitrogen oxide and hydrocarbons. Plants like
pinus, pyrus can metabolise nitrogen oxide.
nRSiCl3 →
3n H 2 O
−3n HCl
67. 2) Rock salt type
Solution :
r 146
Radius ratio + = = 0.675
O O r− 216
R Si O Si R It lies between 0.414 – 0.732
It exhibits rock salt type structure.
→ O O
68. 2) [Pt(NH3)4Cl2]Cl2
R Si O Si R Solution :
O O ∆Tf = iKfm
0.0054 = i x 1.8 x 0.001
i = 3 ; So it is [Pt(NH3)4Cl2]Cl2
125
69. 3) +3 75. 2) [Cu(NH3)4]2+
Solution : Solution :
E.wt × Q CuSO4 + 4NH3 → [Cu(NH3)4]SO4
m=
96500 Blue complex due to [Cu(NH3)4]2+
m × 96500 22.2 × 96,500 76. 3) S2
E= = = 60.3
Q 2 × 5 × 60 × 60 Solution :
At.wt 177 S2 contains two unpaired electrons in the
Oxidation state = = =3
Eq.wt 60.3 antibonding π* orbital.
70. 3) 1.25 x 10-2 77. 3) Cr2O72− - acidic medium
Solution : Solution :
Order w.r.t A = 1 CrO42−(aq) + 4H2O + 3e− → Cr(OH)3 + 5OH−
Order w.r.t B = 1 E° = –0.13V
1 d ( AB ) −
Cr2O7 (aq) + 14H (aq) + 6e → 2Cr3+ + 7H2O
2− +
Rate = = K[A][B]
2 dt E° = 1.33V
Initial rate of appearance of A 78. 3) (a), (b) and (c)
d(AB) Solution :
= 2.5 x 10-4
dt
1
2
( 2.5 ×10 4 ) = K ( 0.1)( 0.1)

K = 1.25 x 10-2
71. 2) II = IV < III < I
Solution : Non-super imposable, optically active, can
Ferric hydroxide sol is positively charged and rotate ppl, non-planar
hence anions would be effective in causing 79. 1) [Mn(CO)6]+
coagulation. Greater the valence of the
effective ion, more will be its coagulating Solution :
power. As +ve charge on the CMA increases, the less
72. 4) Zincite − ZnS readily the metal can donate electron density
into the π* orbitals of CO ligand (donation of
Solution :
electron density into π* orbitals of CO result in
Zincite − ZnO
weakening of CO bond)
73. 2) PCl5, POCl3, H3PO4
80. 2) Ln (III) compounds are generally colourless
Solution :
Solution :
PCl5 + H2O → POCl3 + 2HCl
Most of the Ln3+ compounds are coloured due
(A) (B)
to f-f transition.
POCl3 + 3H2O → H3PO4 + 3HCl
81. 3) C > A > B
(B) (C)
Solution :
74. 3) P4O10 is acidic and NH3 is basic
The acid character follows the order :
Solution :
CH3COOH > C6H5OH > H2O
P4O10 + NH3  H2O
→ (NH4)3PO4
The basic character will follow the order :
Acidic Basic Salt
CH3COO− < C6H5O− < OH−
126

82. 3)

Solution :
CH2 CH CH2 CH2 CH CH3
OH
SOCl2 Pyridine 86. 1) 78.4
Solution :
CH2 CH CH2 CH2 CH CH3 Meq. Mohr’s salt = Meq. KMnO4
w
Cl ×1×1000 = 0.1 x 50
392
O3 / Zn (H2O) w = 1.96
CHO CH2 CH2 CH CH3 1.96
% purity = × 100 = 78.4%
2.5
Cl 87. 2) 0.05
NaBH4 Solution :
µ=qxd
HO CH2 CH2 CH2 CH CH3 0.38 x 10-18 = q x 1.61 x 10-8
Cl q = 2.36 x 10-11 esu
83. 3) CH3CHO 2.36 × 10−11
Fractional charge = = 0.049
Solution : 4.802 ×10 −10
Only those aldehydes which donot have α-H ≈ 0.05
88. 3) 1 mole NH3 at 300K and 1 atm occupies
atom undergo Cannizaro’s reaction. Hence volume 22.4L
CH3CHO will not undergo Cannzizzaro’s Solution :
reaction as it has 3αH-atoms. For ideal gas PV = nRT
Volume of 1 mole gas at 1 atm pressure and
273 K is 22.4L.
84. 2)
89. 2) –2035 kJ/mol
Solution :
For the equation,
B2H6(g) + 3O2(g) → B2O3(g) + 3H2O(g)
Solution : Equations, (a) + 3(b) + 3(c) – (d)
∆H = −1273 + 3(−286) + 3(44) − 36
= −2035 kJ/mol
A) B) 90. 2) < 7.92 atm
Solution :
The efflorescent salts lose water to atmosphere
Kp = PH2 2O = 1.086 x 10-4
85. 1) PH2O = 1.042 x 10-2 atm < 7.92 atm
If H2O pressure at 25°C is less than 7.92 atm,
Solution : the reaction
CuSO4.5H2O(s) → CuSO4.3H2O(s) + 2H2O(g)
127
91. 1) 0.0064g Ionic molar conductivity of H+ is very high
Solution : and NH4OH is a weak electrolyte.
CaC2O4 ⇌ Ca2+ + C2O42− 1
97. 3) C(s) + O2(g) → CO(g)
2
Ksp = S2 ; S = K sp = ( 2.5 ×10 )
−9
Solution :
= 5 x 10-5 mol/lit 1
C(s) + O2(g) → CO(g)
S (in grams) = 5 x 10-5 x 128 = 0.0064g 2
92. 4) a = 1, b = 4, c = 6 and x = 2, y = 6, z = 3 ∆S increases
Solution : Hence as the temperature increases, T∆S
K2Cr2O7 + 4KCl + 6H2SO4 → 2CrO2Cl2 + increases and hence ∆G (∆H - T∆S) decreases.
6KHSO4 + 3H2O 98. 2) Y is an oxyacid of xenon
93. 3) Trigonal bipyramidal Solution :
Solution :

Y is not an oxyacid of Xe.

94. 4) 99. 3)

Solution :

Solution :

95. 4) 528 g
Solution :
Moles of CO2 dissolved 100. 1) C>A>B
= K × PCO2 × Vbottle Solution :
= 3 × 10−2 × 4 × 100 (C) contains the most reactive methylene
group followed by (A) then (B).
Mass of CO2 = No. of moles × Mol. wt.
96. 2) I : HCl, II : NaCl, III : NH4OH
Solution :
128
BOTANY
101. 2) Key 126. 3) Schleiden – Proposed omins cellula – e –
102. 3) Cholesterol content cellula
103. 3) A – ii, B – iii, C – iv, D – i 127. 3) Centrioles
104. 1) Mycorrhizae 128. 4) It can replicate by depending on the main
105. 1) Agaricus genome
106. 4) Both 2 and 3 129. 4) 15 and 19

107. 3) Absence of sieve tubes 130. 3) Mice

108. 4) Thymine = Sugar + Thymidine 131. 3) Glucose + Galactose

109. 3) DNA is negatively charged and histones are 132. 2) A – II; B – I; C – IV; D – III
positively charged 133. 3) (i), (iii) and (iv)
110. 4) 6 and 4 134. 2) Thorns

111. 4) It takes place in 3′ → 5′ direction 135. 4) Intercalary meristems

112. 2) Anaphase 136. 1) Collenchyma

113. 2) Zygotene 137. 3) Crossing over

114. 1) Aspergillus niger – Citric acid 138. 3) 4 : 1

115. 2) Synergids 139. 4) Four and Two

116. 2) 12, 36, 24 140. 3) Transformation

117. 4) ATP and H2O 141. 1) Both the statements are correct

118. 3) A and B – antibiotic resistance genes 142. 2) GEAC

119. 3) Buttercup 143. 2) Bell shaped curve

120. 2) Fertilization 144. 2) unfertilized female gamete

121. 1) Oxygen binds with RuBisCO 145. 2) dsRNA

122. 3) Pressure flow hypothesis 146. 1) Creating disulphide bonds

123. 2) Imbibition 147. 2) Sporopollenin in exine

124. 3) the relative amounts required by the plants 148. 2) 50%

125. 2) Nutrient solution 149. 1) Autogamy and geitonogamy


150. 3) cytoplasm of mesophyll cells

ZOOLOGY
151. 1) Lamprey inner pleural membrane is in contact with
152. 1) Ascaris the alveolar surface
153. 4) Carcharodon 161. 1) A - Systemic arteries, B - Pulmonary vein,
154. 4) have milk producing glands C - Systemic veins, D - Pulmonary artery
155. 4) A is false but R is true 162. 4) Eosinophils resist infections
156. 3) A - iv, B - ii, C - i, D - iii 163. 1) Both A and R are true and R is the correct
157. 4) A - ii, B - iv, C - i, D - iii explanation of A
158. 3) Dense regular connective tissue of tendon 164. 2) diabetes mellitus
159. 2) Starch  → Maltose 165. 3) In hemodialysis, blood drained from a
Salivaryamylase
pH 6.8
convenient vein is pumped into dialyzing
160. 3) The outer pleural membrane is in close
unit after adding an anticoagulant like heparin
contact with the thoracic lining whereas
166. 1) elastic fibre
129
167. 4) c and d 184. 4) Common cold
168. 2) a, b, d 185. 1) Both A and R are true and R is the correct
169. 3) C and D only explanation of A
170. 1) Except statement v all are correct 186. 1) 50
171. 4) all the above 187. 1) cirrhosis
172. 4) both 1 and 2 188. 3) all except iii
173. 3) Immediately after implantation, inner cell 189. 1) A - ii, B - i, C - iii
mass differentiates into germ layers 190. 1) Both Statements I and II are correct
174. 1) A - iii, B - iv, C - ii, D - i 191. 4) Pug marks and faecal pellets
175. 3) Statement I is incorrect, statement II is 192. 4) Aquarium
correct 193. 2) inverted
176. 3) A - i, B - ii, C - iii, D - iv 194. 1) Both Statements I and II are correct
177. 3) Statement I is incorrect, statement II is 195. 3) 2000 species of native birds
correct 196. 3) One of the major effects of deforestation is
178. 3) A only reduced CO2 concentration in the atmosphere
179. 3) A and C 197. 3) Clouds and gases reflect about one fifth of
180. 2) A - ii, B - i, C - iii, D - iv the incoming solar radiation
181. 2) Both A and R are true and R is the not 198. 4) Gold
correct explanation of A 199. 3) Biomagnification
182. 2) Neanderthal man 200. 2) green revolution
183. 1) P. falciparum

SPP – 3 (Answer Key & Solutions)

PHYSICS
1. 1) 4 2. 1) (i) → b, c ; (ii) → a, e ; (iii) → d
Solution : Solution :
E = A2e-αt q
C = ; F = Bqv ;
ln E = 2 ln A - αt v
dE 2dA F
= − αdt B=
E A qv
dE dA
x100 = 2 x100 − αdt x100
E A
∆t 3. 4)
x100 = 1.5
t
dE dA
x100 = 2 x100 + 1.5 αt Solution :
E A R α U2
∆t x 100 = 1.5 t 4. 2) 55 m/s
dE Solution :
x100 = 2 x 1.25 + 0.2 x 1.5 x 5 = 4
E 1
∆V = at = x 10 x 11 = 55 m/s
2
130
5. 1) 9.8 N 12. 2) 2 v
Solution : Solution :
mg sin θ < µN
g gR 2
1 =
∴ f = mg sin θ = 2 x 9.8 x = 9.8 N 4 (R + h)2
2
R + h = 2R R=h
6. 3) 45o
Solution : Rk 2
we know that h =
v2 100 1− k2
tan θ = tan θ = ⇒ θ = 45o
rg 10 x10 Rk 2
R= ⇒ 1 – k2 = k2
7. 3) v3 1− k2
Solution : 1
k=
dm d 2
F= v = v ρ(vol) = vρ(Av) = v2Aρ
dt dt 13. 1) increases
P = FV = Aρv3 Solution :
8. 1) 0.4 m/s GM 1
g= 2 gα 2
Solution : r r
1 1
mv 2 = µmg x + kx 2 14. 1) 2 x 1011 N/m
2 2 Solution :
1 Wl ∆l 3 x10−4
x 0.18 v2 = (0.1 x 0.18 x 10 x 0.06) Y= ⇒ = slope =
2 A∆l W 60
1 60 x1
+ ( x 2 x (0.06)2) Y=
2 3 x10−4 x10−6
2
v = 0.16 ⇒ v = 0.4 m/s = 20 x 1010 = 2 x 1011 N/m
9. 4) 0, 1 15. 2) Blv
Solution : Solution :
Use truth table of AND and NAND gates. Motional emf
10. 2) remains constant 5
Solution : 16. 4)
7
Solution :
dU 1 5
= =
dQ r 7

L = mvr = const.
11. 3) 2 3 m/s 17. 1)
Solution :
Velocity along the length of the rod is same as Solution :
length is constant. ∆P ∆V
2 cos 30 = vB cos 60 PV = C ⇒ =−
P V
2 3 vB ∆V
⇒ = −
2 2 ∆P = 1
∴ v B = 2 3 m/s V P
βP=C
131
18. 2) 0.69 4πε 0 Fd 2
Solution : 24. 4)
e2
T1 λ m2 350
= = = 0.686 Solution :
T2 λ m1 510 1 n 2e2
F=
19. 1) H2 4πε0 d 2
Solution :
Fd 2
3RT n2 = 4πε0
v= 4πe 2
M
4πε 0 Fd 2
3
3 x 8.3 x10 x 300 n=
1930 = e2
M 25. 4) 122.4
M=2 Solution :
k m1g z2
20. 1) , E = − 13.6 2
m2 k n
Solution : E = 122.4 eV
m1g = kA 4q
26. 4)
k 3
w=
m2 Solution :
q1 ' q 2 '
m1g q11 + q21 = -q =
A= R 2R
k
3) 600 Hz q2 '
21. + q2 ' = − q
Solution : 2
γα T 3q 2 ' 2q
=−q q2 ' = −
T = mg 2 3
2q 2q q
γ1 mg q1 ' − = − q ⇒ q ' = − q = −
= 3 3 3
γ2 4mg
27. 2) 400 V
γ1 1 Solution :
=
γ2 2
∴ γ2 = 2γ1 = 600 Hz
22. 1) 6 beats/sec
Solution :
342 + 5 x 200 347
n '= = x 200 28. 2) 80 MW
342 − 5 337 Solution :
= 205.93 Hz qv 4 x 4 x106
1) volume P= = = 80 MW
23.
2t 2 x100 x10−3
Solution :
29. 1) IR = IG
∆V ∆r ∆A ∆r
=3 and =2 Solution :
V r A r
Balanced Wheatstone bridge circuit.
132
30. 2) 5 sec 37. 4) dA + dB = 2dF
Solution : Solution :

τ = 4 sec ; v = v0(1 – e-t/τ)


4 = 10 (1 – e-t/4)
6 5
= e − t /4 ⇒ = e t / 4
10 3
t dFvg = T + dAvg
ln 5 – ln 3 = & t = 4 x 0.5 = 2 sec
4 dBvg = dFvg + T
31. 3) Independent of L dFvg = dBvg – dFvg + dAvg
Solution :
dF = dB – d F + d A
ρL ρ
R= = d + dB
Lt t ⇒ dF = A
2
32. 1) 2 : 1 38. 3) X1 is an capacitor and X2 is a inductor
Solution : Solution :
h 1 XL = ωL
λ= ; λα
2mkE KE 1
XC =
λ1 KE 2 2E 0 ωC
= = = 2
λ2 KE1 E0 39. 4) L2 and immersed in L1
33. 4) west Solution :
Solution : If refractive index of lens is more than
Right hand thumb rule. refractive index of surroundings then its nature
remains same.
r2
34. 1) 40. 1) zero
(r 2 + x 2 )3/ 2 Solution :
Solution : 60 + 30
µ ix 2 n sin
B = 2 0 22 3/2 µ= 2 = 1 x2= 2
2(r + x ) 60 2 1
sin
35. 4) 30o 2
Solution : 41. 2) 1.3
BH = 3 BV Solution :
h
BV 1 r=
= = tan θ
L µg − 1
2
BH 3
36. 1) 0.1 V Lµg=2
Solution : 2.72
2= µL = 1.36
di 5 µL
e = L = 2 x10 −3 x = 100 x 10-3 = 0.1 V
dt 0.1
133
 λ  6.67 x10−11 x (1.67 x10−27 )2
42. 3) sin −1   ∆e2 = = 2 x 10-74
 3d  9 x10 9

Solution : ∆e = 1.414 x 10-37C


φ
I0 = 4I0 'cos 2 1 ln 2
2 47. 1) and
λ λ
2π λ 2π λ Solution :
φ= pd = =
3 2π 3 3 1 ln 2
τ = and T1/ 2 =
λ  λ  λ λ
d sin θ = θ = sin −1  
3  3d  48. 3) 1.5 A
Solution :
43. 2) 10 x 20 20
R eff = = Ω
30 3
10
i = x 3 = 1.5 A
Solution : 20
I = I0 cos2θ for θ = 0o I = I0 49. 4) 500 m/s
for θ = 90o I=0 Solution :
44. 1) If B is along z-axis the F α (L + R) 2h
R =u
Solution : g
F = Bil =Bi (2L + 2R) F α (L + R)
2h
45. 1) 1 For ball R B = u B
Solution : g
hν = hν0 + eV0 2h
h hν For bullet, R b = u b
⇒ v0 = ν − 0 g
e e
m1u1 + m2u2 = m1v1 + m2v2
h
Slope = ⇒ub = 500 m/s
e 50. 2) 0.99
-37
46. 3) 10 C Solution :
Solution : β
1 q1q 2 mm α=
=G 12 2 1+ β
4πε 0 d 2
d
99
9 x 109 (∆e)2 = 6.67 x 10-11 x (1.67 x 10-2) α= = 0.99
100

CHEMISTRY
51. 3) 18 mol of H2O Option (4) violates Aufbau principle since 2p
Solution : orbitals are filled without filling 2s orbital and
Number of water molecules = 18 x N0 it also violates Pauli’s exclusion principle
since two electrons in 2p have same spin.
53. 2) C < N < F < O
52. 4)
Solution :
Solution : Second ionization enthalpy order is
C<N<F<O
134
54. 1) XeF2, IF2− 2
2

 
Kc =  
Solution : 3 2
∴ Kc =
XeF2 sp3d ⇒ linear ; IF2− sp3d ⇒ linear 2 3
∴ XeF2, IF2- are isostructural 3
55. 3) N 2 > N +2 > N −2 59. 2) basic
Solution :
Solution :
1
Bond order N2 = 3, N +2 = 2.5, N −2 = 2.5 For neutral sample pH = (pKw)
2
Both N +2 and N −2 have same bond order, but 1 1
= (-log 4 x 10-14) = [14 – log 4]
N −2 has more anti bonding orbital electrons so 2 2
it is less stable. = 7 – log 2 = 6.7
56. 2) 11 : 1 But given pH = 6.9 > 6.7 (neutral pH)
Solution : ∴ The sample is basic
3 3w 4) I2 is a weak oxidizing agent than Br2
KE = nRT = RT 60.
2 2M Solution :
KE1 T1M 2 300 44 11
= = x = Br2 is a strong oxidizing agent than I2
KE 2 T2 M1 2 600 1 61. 2) a, c and d only
57. 4) ∆H is +ve and ∆S is -ve Solution :
Solution : Statements (a), (c) and (d) are correct.
∆G = ∆H - T∆S = +ve for Non-spontaneous Hydrides of O and F have higher boiling point
∆H = +ve ; ∆S = -ve ; ∆G = +ve than hydrides of their subsequent group
2 members.
58. 3) 62. 4) b, c and d only
3
Solution : Solution :
Alkali metals are reducing agents.
N2O4(g) ⇌ 2NO2(g) 63. 3) a-T, b-T, c-T, d- F
Initially 1 mol 0 Solution :
at equilibrium (1-x) mol 2x mol (d) only false statement, Pb2+ is more stable
∴ At equilibrium nN2O4 = 1 – x & nNO2 = 2x than Pb4+, due to inert pair effect.
64. 2) 5
(1 − x) 1− x
XN2O4 = = Solution :
(1 − x) + 2x 1 + x NaNH 2

1 HC ≡ CH 
(1mole)
ether
→ HC ≡ CNa
But given XN2O4 = (A)
2 CH 3 − CH 2 − Br

→ HC ≡ C − CH 2 − CH 3
1− x 1 1 (B)
∴ = ⇒ 2 – 2x = 1 + x ⇒x=
1+ x 2 3 OH
2x |
[NO2] = (∵ V = 1 lit)  → CH = C − CH CH
dil H 2SO 4 /HgSO 4

2 2 3
1 (Major) (C)
2 O
=
3 Tautomerism
⇀ ||
↽ CH 3 − C − CH 2 − CH 3
[ NO2 ]
2
1 2 (D)
[N2O4] = 1 - = ; Kc =
3 3 [ N 2O 4 ]
Number of α-hydrogens = 5
135
65. 4) A-M, B-K, C-N, D-L 70. 4) 24 g
Solution: Solution :
A) Lassaigne’s test of N ⇒ Fe4[Fe(CN)6]3 According to Faraday’s 2nd law
B) Lassaigne’s test of S ⇒ [Fe(CN)5NOS]-4 WMg VO2
=
C) Duma’s method ⇒ N2 GEW of Mg GEV of O 2
D) Kjeldahl’s method ⇒ NH3 WMg 11.2
= ; WMg = 24
12 5.6
71. 3) increased by 9 times
66. 4) Solution :
r1 = k [X]2
r2 = k [3X]2
Solution :
r2 = 9 k [X]2 = 9r1
CH3-CH=CH-Br 
NaNH 2
→ CH3-C≡CH
The rate of reaction increases 9 times.
72. 2) 32
Solution:
→
873K
Red hot
x
= KP l/ n
Iron tube
m
x 1
67.2) IV III II I log = log K + log P
m n
Solution :
1
(A) NO3- ion in drinking Blue baby = 0.5 ⇒ n = 2
water greater than syndrome n
50 ppm causes log K = 0.6020 ⇒ K = 4
1
(B) SO42- ion greater than Laxative effect x
= 4(26 ) 2 = 32
500 ppm causes m
(C) F- ion concentration Brown mottling 73. 4) based on principle that the impurities are
above 2ppm causes of teeth more soluble in the melt than in the solid state
of the metal.
(D) > 50 ppb of Pb causes Damage of
Solution :
kidney and liver
Zone refining method is based on principle
68. 4) Frenkel defect is a dislocation defect. that the impurities are more soluble in the melt
Solution: than in the solid state of the metal.
Frenkel defect is a dislocation defect as some
cations are dislocated from lattice sites and 74. 2)
occupy interstitial sites.
69. 2) 373.202 K Solution :
Solution : HI
O I + CH CH O H
2
∆Tb = i Kbm = 1 x 0.52 (0.1)
Tautomerisation
Tbs − Tbo = 0.052 O
s
T = 0.052 + 373.150
b CH3 C H

= 373.202 K
136
B) Among tetrafluorides, SF4 is a gas, SeF4 is
75. 3) a liquid and TeF4 is a solid.
C) Dioxides of 16th group are reducing agents
Solution : and their reducing property decreases
from SO2 to TeO2
D) BP order of 16th group hydrides is

81. 4) KMnO4, K2Cr2O7, Cu2O – Colorless and


76. 2) I > III > II > IV diamagnetic
Solution : Solution :
Aldehydes are more reactive than ketones KMnO4, K2Cr2O7, Cu2O in which there are no
number alkyl groups increases around unpaired electrons hence diamagnetic but they
carbonyl group (or) size of alkyl group around are colored compounds due to charge transfer
carbonyl group increases, reactivity decreases. phenomenon.
77. 4) All the above 82. 4) [Co(NH3)3Cl3] 0 moles 1
Solution : Solution :
Acidic strength of carboxylic ∝ EWG ∝ 1 [Co(NH3)3Cl3] is non-ionizable
acids and phenols EDG 83. 2) Pentaacetate of glucose does not react with
hydroxyl amine
Acidic strength of carboxylic ∝ K ∝ 1
acids and phenols a Solution :
pK a
Due to cyclic structure free –CHO group is
78. 1) absent in penta acetate of glucose, so it does
not react with hydroxyl amine.
Solution : 84. 4) Linear condensation polymer of phenol and
O O formaldehyde
KOH Br CH2Cl Solution :
N H N K

O O
O
+
N CH2 Br H3O Br CH2NH2

79. 3) Almost identical to first I.E of Xe


Solution :
First ionisation energy of O2 and Xe are
1175 kJ mol-1 and 1170 kJ mol-1 respectively.
80. 3) A, B, C and D 85. 1) Non-ionic detergents
Solution : Solution :
A) The stability of halides of 16th group Polyethylene glycols are used to prepare non-
halides decreases in the order of ionic detergents which are used as liquid
F- > Cl- > Br- > I- dishwashing detergents.
137
86. 3) 9
is more stable carbocation
Solution :
For hydrogen atom because stabilized by strong +R-effect of the

–OCH3 group
Total degenerate orbital in 3rd shell = 9 90. 2) Q < Kc
87. 2) 3 : 4 Solution:
Solution : 2SO2(g) + O2(g) ⇌ 2SO3(g)

[SO3 ]
2

Kc = ; ∆n(g) = 2 – 3 = −1
[SO 2 ] [ O2 ]
2 1

Thus, the reaction will shift in backward


direction when Q < Kc
− −
a
3N2H4(l) + 4ClO3(aq) → 6NO(g) + 4Cl(g) 91. 3)
2
Left side Right side Solution:
12 ‘H’ atoms No ‘H’ atoms
12 ‘O’ atoms 6 ‘O’ atoms
Balance ‘H’ & ‘O’ atoms by adding 6H2O to
the right side
− −
3N2H4(l) + 4ClO3(aq) → 6NO(g) + 4Cl(g) The distance between the face centred atom
+ 6H2O(l) 2 (a )
and one corner atom is i.e half of the
∴ The equation is balanced & hence nN2H4 = x = 3 2
face diagonal.
n ClO− = y = 4
3 92. 1) T2 > T1 > T3
∴x:y=3:4 Solution :
1
Boiling point (T) ∝ (For the same
88. 4) GMW
mass of solute and solvent)
Solution : 93. 4) Infinite
Solution :
0.693
k= , for 50% completion
t1/2
0.693
k=
1.26 x1014
for 100% completion,
The formed products both are ketones and do 2.303  a 
not give silver mirror with Tollen’s reagent. t= log  
k a−x
89. 2) 2.303 x1.26 x1014 100
t= log
0.693 0
Solution :
t =∞
138
94. 2) 1.10V Phosphinic acid − Hypo phosphorous acid
Solution : Phosphonic acid − Ortho phosphorous acid
0.059  Zn +2  98. 3) V
+2
< Cr+2 < Mn+2 < Fe+2, paramagnetic
E=E o
− log
cell
n Cu +2  behaviour
Solution :
0.059  0.01 
E = 1.1 − log   = 1.1 Correct paramagnetic behaviour (theoretically):
2  0.01 
V+2 < Cr+2 = Fe+2 < Mn+2
95. 2) A, C
Solution : (n=3) (n=4) (n=4) (n=5)
C2H5OH does not react with Br2 water. 99. 3) A-iv, B-iii, C-ii, D-i
C2H5OH does not react with neutral FeCl3 test. Solution:
Phenol and ethanol are react with Na metal. E) [Ma5b] - Neither geometrical nor
optical isomerism
F) [Ma4b2] - Geometrical isomerism only
96. 1) G) [M(AA)3] - Optical isomerism only
H) [M(AB)3] - Both optical and geometrical
isomerism
Solution:
100. 3) A-III, B-IV, C-I, D-II
Solution :
A) Histidine - Basic amino acid
B) Asparagine - Contains -CONH2 group
C) Proline - Secondary amino acid
97. 1) Both Statements-I and II are correct D) Serine - Contains –OH group
Solution:

BOTANY
101. 1) Mangifera indica Linn. 117. 3) 2 C2H5OH + 2 ATP + 2 CO2
102. 4) Both (1) and (2) 118. 1) Lt = Lo + rt, linear
103. 3) Four 119. 2) Elaborate, complex and slow process
104. 4) Cholera causing bacteria are vibrios 120. 3) Chloroplast, Peroxisome and Mitochondria
105. 1) Dmitri Ivanowsky recognized Tobacco 121. 4) Water stress causes the enhancement in the
Mosaic Virus in 1892 CO2 availability
106. 4) Chlamydomonas Haplontic 122. 4) Statement – I is incorrect, statement – II is
107. 3) Mustard correct
108. 3) Wilkins and Franklin 123. 4) Active transport
109. 2) Mice died and showed live S strain 124. 1) N2+ 3H2 → 2NH3
bacterial cells 125. 3) Deficiency of any element can be replaced
110. 2) From several sites along the DNA of a by supplying some other element
chromosome simultaneously 126. 3) Chromatophores
111. 4) RNA dependent DNA polymerase 127. 2) Pili play a major role in motility
112. 3) Wine and Beer 128. 3) Thylakoid membrane
113. 2) 4n in embryo and 7n in endosperm 129. 1) I and II
114. 3) 1 : 8 130. 3) Succinate
115. 1) 6 131. 2) two times
116. 3) Pyruvate dehydrogenase Acetyl coA CO2 132. 4) All of these
139
133. 4) Lamina 143. 3) to uptake of DNA through transient pores in
134. 1) Reticulate and parallel the bacterial cell wall
135. 1) Cork 144. 4) Bio-insecticide and Bio-pesticide
136. 3) Presence of naked megasporangia 145. 4) Genetically engineered insulin has
137. 3) Anaphase carbohydrates
138. 4) Diplotene 146. 2) 30
139. 2) A – IV, B – III, C – I D – II 147. 3) Development of male gametophyte
140. 3) Class III 148. 2) Propionibacterium sharmanii
141. 1) 78 149. 3) Stem borers
142. 4) DNase 150. 2) Mycorrhizae

ZOOLOGY
151. 2) B is viviparous and homiothermous 174. 4) 60,000 - 80,000
152. 1) Mammary glands, hairy skin, pinnae 175. 2) A - ii, B - iv, C - i, D - iii
153. 4) A - Culex, B - Exocoetus 176. 1) Both Statements I and II are correct
154. 4) A - v, B - i, C - iv, D - ii, E - iii 177. 3) five
155. 2) Both A and R are true and R is not the 178. 3) Turner’s syndrome
correct explanation of A 179. 3) absence of random mating
156. 1) Both A and R are true and R is the correct 180. 2) 18000 ya
explanation of A 181. 3) Neanderthal man
157. 2) Both A and R are true and R is the not 182. 2) Statement I is correct, statement II is
correct explanation of A incorrect
158. 1) Both A and R are true and R is the correct 183. 1) mosquitoes gut
explanation of A 184. 2) Both A and R are true and R is the not
159. 4) both 1 and 2 correct explanation of A
160. 3) Diaphragm contracted and arched upwards 185. 2) Both A and R are true and R is the not
161. 4) A - ii, B - i, C - i, D - iv correct explanation of A
162. 1) A - Interventricular septum, B - Bundle of 186. 1) A - Partial regulators, B - Regulators,
His, C - Pulmonary veins, D - Chordae C - Conformers
tendinae 187. 3) Red algae
163. 1) Both Statements I and II are correct 188.
164. 4) All the above 1) Amensalism - O
165. 2) 180 L 189. 1) 2 - 10
166. 1) Both Statements I and II are correct 190. 3) iii only
167. 4) cartilage 191. 4) Both A and R are false
+
168. 3) Na diffuses into the axon - the inside of the 192. 3) 25,000
axon become positive 193. 4) Agricultural chemicals (pesticides)
169. 3) mid brain 194. 2) atrial natriuretic factor
170. 4) all the above 195. 2) scapula
171. 1) Prolonged hypoglycemia could be the 196. 4) All the above
reason 197. 4) i, ii, iii and iv
172. 3) Statement I is incorrect, statement II is 198. 3) sharing of utensils
correct 199. 4) Increased ozone concentration in stratosphere
+
173. 2) isthmus 200. 4) I - AB , II - presence of Rh antigen
140
SPP – 4 (Answer Key & Solutions)

PHYSICS
1. 2) 11 % 6. 1) It hits the ground at a horizontal distance
Solution : 1.6 m from the edge of the table
∆A ∆l ∆b Solution :
= + Vertical component of velocity
A l b
∆A vy = uy + ay = 0 + 10 x 0.4
 ∆l   ∆b 
x100 =  x100  +  x100  vy = 4 m/s
A  l   b 
Horizontal component of velocity vx = 4 m/s
 0.2   0.01 
= x100  +  x100  Speed with which it hits the ground= 4 2 m/s
 2   1 
v
20 1 tan θ = y = 1 ⇒ θ = 45o with horizontal.
= + = 10 + 1 = 11 % vx
2 1
3) energy 1 1
2. Height of the table h = gt 2 = x 10 x (0.4)2
Solution : 2 2
Kilowatt hour is a unit of energy = 0.8 m
1 kwH = 3.6 x 106 J Horizontal distance travelled from the edge of
3. 3) 25 m/s the table.
Solution : R = ut = 4 x 0.4 = 1.6 m
Area under a – t graph gives change in 7. 4) 5 kg
velocity. Solution :
1 mQ
∆ V = x 10 x 5 = 25 For limiting condition µ =
2 (m P + m R )
V – 0 = 25 ⇒ V = 25 m/s
10
4. 3) kinetic energy constant 0.4 =
(20 + m R )
Solution :
In the given condition, the particle undergoes 100
20 + mR = = 25
uniform circular motion and for uniform 4
circular motion the velocity and acceleration mR = 5 kg
vector changes continuously but kinetic 8. 2) 100 J
energy is constant.
Solution :
5. 3) 170 N 4 4

Solution : W = ∫ Fdx = ∫ (2x + 3x 2 + 5)dx


v = 10 tiˆ + 4tjˆ 0 0
4 4
dv  x2   x3 
a = =10 ˆi + 4ˆj W = 2   + 3   + 5(x)04
dt  2 0  3 0
Net force F = ma x ˆi + m(g + a y )ˆj = (4)2 + (4)3 + 5(4)
= 16 + 64 + 20 = 100 J
| F | = m a 2x + (g + a y )2
9. 2) 3.14 s
= 10 (10) + (14) =10 100 + 196 = 10 296
2 2 Solution :
2πm 2π x10−6
= 10 (17) = 170 N T= = = πs = 3.14 s
qB 20 x10−6 x10−1
141
10. 2) TA = 375 N  25Gm 2 
Solution : 12. 2) 3 
 4 
For translational equilibrium
TA + TB = 100 + 500 = 600 Solution :
G(10m)(10m)
∴ TA + TB = 600 --- (1) F1 = F2 =
16
For rotational equilibrium, about A
100 Gm 2 25Gm 2
l l F1 = F2 = =
TA(0) + 500   + 100   - TB l = 0 16 4
 4 2
l 100 l
500 + = TB l
4 2
900 l
= TB l
4
∴ TB = 225 N
From eqn (1), TA= 375 N Net force at origin
L(3α + 2β L)  60o   25Gm 2   3 
11. 1)
3(2α + β L) Fnet = 2F1 cos   = 2   
 2   4  2 
Solution :
The rod lies along the x-axis with its lighter  25Gm 2 
∴ Fnet = 3  
and on the origin to make mass distribution  4 
equation consistent with coordinate system.
2V
13. 3)
3
Solution :
Gravitational potential at the centre of the
3 GM
L earth is VC = V = − --- (1)
∫ xdm ∫ xλdx 2 R
Gm
x cm = = 0L On the surface of the earth VS = −
R
∫ dm ∫ λdx Gm 2V 2V
0
From equation (1) − = ∴ VS =
L
 L2   L3  R 3 3
∫ x(α + βx)dxα   +β 
2 3
14. 2) Young’s modulus and bulk modulus are
x cm = 0L =   2  infinity
L  Solution :
∫0 (α + βx)dx α(L) + β  2  For a perfectly rigid body both youngs
modulus and bulk modulus are infinite.
αL β L2
+ 15. 2) 18 cm of ice cube will be in water
= 2 3 Solution :
βL Fg = FB
α+
2 ρicea3g = ρωa2x g
L
( 3α + 2βL ) L(3α + 2βL)
= 6 =
 2α + β L  3(2α + βL)
 
 2  900 x 20 x a2g = 1000 x a2 x xg
18000 = 1000 x ∴ x = 18 cm
142
5 5
16. 1) for monoatomic gas γ =
2 3
Solution : du 1 3
Heat from A to C = Heat from C to B ∴ = = = 0.6
dQ 5 5
3
20. 3) 5 V
Solution :
The diode is in forward bias

k A (4T − x) k A (x − T)
=
l l
4T – x = x – T
2x = 4T + T
5T x 5 T 5
x= ⇒ = ∴ C=
2 T 2 TB 2
17. 4) 27 : 8
Solution :
Thermal capacity = S m
H = 5m 15 15 x 2 5
i= = = 2 ; V2 = x 2 = 5 V
Both are same material so S1 = S2 5 15 2
5+
H m 2
∴ 1= 1
H2 m2 21. 2) 2 cm
Solution :
H1 (3)3 27 Internal forces in the spring are same
= =
H 2 (2)3 8 k1x1 = k2x2 --- (i)
18. 3) decreases as it goes from C to D and x1 + x2 = A --- (ii)
Solution : k A 10 x 3 10 x 3
∴ x1 = 2 = = = 2 cm
Draw the rectangular cycle against the k1 + k 2 5 + 10 15
background of various dotted isothermal lines. 22. 1) A (p, s) ; B(p, s) ; C(q); D(s)
Here T5 > T4 > T3 > T2 > T1 Solution :
l
Time period is a liquid T = 2π
 ρ 
g 1 − l 
 ρb 
A constant vertical upward force
So in the process D to A the temperature l
increases. T = 2π
F
In A to B it further increases, in B to C it g 
m
decreases and in C to D it decreases so the
correct response is 3. l
Time period in a lift T = 2π
19. 1) 0.6 g±a
Solution :
l
du nC V dT 1 Time period in a satellite T = 2π =∝
= = g−g
dQ nCP dT γ
143
23. 1) 2.78 4 KQ
Solution : 26. 2)
15 R
Ν T Solution :
f=
2l µ
8 T1
∴ f=
2l µ
10 T2
f=
2l µ KQ KQ 2KQ
VC1 = − =
∴ 8 T1 =10 T2 3R 5R 15R
 ρ   ρl  KQ KQ 2KQ
VC2 = − + =−
mg  1 − l  1 −  3R 5R 15R
64 T2  ρb  16  ρb 
= = ⇒ = 4KQ
100 T1 mg 25 1 =
15R
ρl 16 ρ 9 1) 1 nF
1− = ⇒ l= 27.
ρb 25 ρb 25 Solution :
ρb 25 2πε 0 l
∴ = ≈2.78 C=
ρl 9 b
ln  
a
λ
24. 2) ln x = x -1 for x ≈ 1
πε 0 r
b b−a
Solution : ⇒ − 1=
| E | = E 2 − E1 a a
2πε 0 l.a 2πε 0 .al
2λ λ λ C= =
= − = (b − a) (b − a)
2πε0
r
2πε0
r πε0 r
2 2 1 1.8 x10 −2 x 20 x10 −2
=
25. 4) 30 V 2 x 9 x109 (0.02 x10 −2 )
Solution : 1.8 x 2 x10−3
Q 2C = = 1 x 10-9 = 1 nF
x M 2 x 9 x 2 x105

60 V C C 28. 2) 28 A
Solution :
y
N 6 x 2 = 4 x I1 = 12 x I2
2C
I1 = 3A
Let Q be charge on each capacitor then I2 = 1A
Q Q Q i1 = 6A + 3A + 1A = 10 A
60 = + +
2C C 2C 4xi
Q(1 + 2 + 1) Q(4) 10 A =
60 = ⇒ 60 = ∴ Q = 30 C 7.2 + 4
2C 2C
112
Q 30 C i= = 28 A
VMN = = = 30 V 4
C C
144
4Cε 35. 1) µ0nNS
29. 4)
5 Solution :
Solution : Magnetic field inside the solenoid = µ0nI
ε Flux through the circular coil encircling the
i=
5R solenoid = µ0nIS.N = φ
4ε φ
Potential difference = M = = µ 0 nSN
5R I
4Cε 2)
m
(π + 2θ)
Charge stored = 36.
5R qB
30. 3) 12 W Solution :
Solution :
ε = 3 – 0.75 i
ε = 3 volts
r = 0.75 Ω
Maximum current drawn is when battery is
short circuited.
ε 3  π + 2θ  2πm  π + 2θ  m( π + 2θ)
i max = = x 4 = 4A t =T =  =
r 3  2π  qB  2π  qB
Pmax = εI = (3) (imax) = 12 W 37. 4) 8 m
13 Solution :
31. 3) A 2πm
2 P = V||T = V cos θ
Solution : qB
i0 13 4 2π x10 x10−6
i 0 = 22 + 32 = 13 ⇒ irms = = = 5x x m =8m
2 2 5 10 x10 −6 x 3.14
32. 3) W 3 38. 2) 40 ˆi + 20 ˆj m/s
Solution : Solution :
1 Using ICR (instantaneous centre of rotational
W = MB (1 – cos 60o) = MB the contact point P is the ICR is rolling. The
2
cylinder is in pure rotation about the ICR at
3 the instant under consideration. So from the
τ = MB sin 60o = 2W
2 relative motion equation, we have
33. 4) 2 A
Solution :
NA(B2 − B1 ) E
E= and i =
t R
(250) (0.2) x (0.06 − 0.01)
εav = = 10 V
0.25
∴ i av =
10
= 2.0 A VA = ω x rA = ω x (PC x CA)
5 ˆ x (5jˆ + (−4iˆ + 3j))
ˆ
= (−5k)
34. 4) 25 Ω
Solution : VA = (40iˆ + 20ˆj)m / s
ig (G + R) = V ∴G = 25 Ω
145
39. 4) 10.6 mA 44. 2) 0.012 rad
Solution : Solution :
dφ nλ
i d = ε 0 E = 8.85 x 10-12 x 3 x 1012 x 4 x 10-4 θ= ,n =2
dt a
= 12 x 8.85 x 10-4 A 2 x 6000 x10 −10
⇒ θ= = 12 x 10-3 rad
= 106.20 x 10-4A = 10.6 mA 10 −4

40. 4) 30 cm
Solution :
R 45. 4)
f = = 20 cm , m = 2 For real image ; m = -2
2
f Solution :
By using m = ,
f −u KEmax = hf - φ
−20 46. 2) 5 x 10-6 Pa
−2 = ⇒ u = − 30 cm Solution :
−20 − u
For virtual image ; m = +2 I 1.5 x103
P= =
−20 C 3 x108
So, +2 = ⇒ u = − 10 cm
−20 − u = 0.5 x 10-5 = 5 x 10-6
47. 4) 3200
o Solution :
41. 3) 60
Solution : ∆I r ∆I
A V = C . 0 = C .2 = 80
A + δm  60 + δ m  ∆I B ri ∆I B
sin 2  sin 
µ= 2 ⇒ 3=  2  ∆FC
A ⇒ = 40
sin 1 ∆I B
2
AP = AV.Ai = 80 x 40 = 3200
60 + δ m 3 5V0
⇒ sin = = sin 60o 48. 3)
2 2 4
⇒ δm = 60o Solution :
42. 4) 15 P − P0 V − VA
=
Solution : PB − PA VB − VA
L  D L D  2P 
| m 2 − m1 | = 1 +  −  
f0  fe  f0  fe  ⇒ P = 3P0 -  0  (V − V0 )
 V0 
L
= = 15 2P V
f0 ∴ P = 5P0 − 0 --- (1)
V0
43. 4) 2 mm
For one molecule ideal gas equation PV = RT
Solution :
Y RT 2P V
a sin θ = λ ⇒ a=λ P= = 5P0 − 0
D V V0
Dλ 3 x 6000 x10−10 x103 P0  2V 2 
⇒ y= = mm ∴ T=  5V − 
a 0.9 x10−3 R V0 
3x 6 dT
= = 2 mm If T is maximum then =0
9 dV
146
d  P0  2V 2   ⇒ v=
e2
∴   5V −  =0
dT  R  V0   2ε 0 nh
1
∴ V=
5V0 ⇒ E = − mv 2 ⇒ E ∝ m
2
4
∴ E = -2 x 13.6 = -27.2 eV
5V0
∴ At maximum T, the value of V is 50. 1) 6.66 T
4 Solution :
49. 1) – 27.2 eV 0.01 A0 = A0e-λt
Solution : 10-2 = e-λt
mv 2 1 e2 ⇒ 102 = eλt ⇒ 2 ln 10 = λt
= .
r 4πε0 r 2 2 ln 10 2 x 2.3
⇒ t= T= T = 6.66 T
nh nh e2 ln 2 0.69
mvr = ⇒ r= =
2π 2πmv 4πε 0 mv 2

CHEMISTRY
15 53. 1) α - D - glucose
51. 4)
16 Solution :
Solution : Maltose is a disaccharide made up of α-D-
Suppose 'x' g of each gas is present glucose units.
x 54. 1) BaCO3 > SrCO3 > CaCO3 > MgCO3
Number of moles of H2 = Solution :
2
Thermal stability of alkaline earth metal
x carbonates increases down the group. Thus
Number of moles of C2H6 =
30 order of stability should be
15 BaCO3 > SrCO3 > CaCO3 > MgCO3
Mole fraction of H2 =
16 55. 3)
1
Mole fraction of C2H6 =
16
Pressure exerted is directly proportional to Solution :
mole fraction.
and , are non-aromatic
∴ Fraction of total pressure exerted by
15
H2 =
16 (does not follow (4n + 2) π e- rule)
52. 4) [Pt(en)2Cl2]
But furan follows all rules including
Solution :

Cl en en (4n +2) π e-
56. 2) C6H6
en Pt en en Pt Pt en Solution :
Cl Cl C6H5CH3  oxidation
→ C6H5COOH  NaOH

Cl Cl Cl (A)
cis
Trans C6H5COONa  Soda lim e

→ C6H6 + Na2CO3
Geometrical
(B) (C)
Optical isomers
147
- +
57. 1) (CH3)3C - O Na + C6H5CH2 – Br 64. 4) CH3COCH3
Solution : Solution:-

2 – methyl - 2 – butanol
65. 2) cyclic silicate
58. 4) In covalent compounds fluorine can form Solution :
Cyclic silicates contains ( SiO3 ) n ions
2 n−
single bond only while oxygen forms double
bond. Each unit shares two oxygen atoms with other
Solution:- units.
Conceptual
66. 4) O-2 > F- > F > O (radius)
59. 4) [Co(CN)6]3-
Solution :
Solution:-
O-2 > F- > O > F is the correct order.
CN− is a strong field ligand and hence pairing
of e− takes place 67. 2) C2 has 1σ bond and 1π - bond
Co+3 = 3d6 4s° 4p° Solution :
C2 has two π bonds according to M.O.T.
68. 1) rich in dissolved oxygen
No unpaired e−, Hence lowest value of Solution :
magnetic behaviour The total amount of oxygen consumed by
60. 4) 2 : 1 microorganisms in decomposing organic
Solution:- matter present in definite volume of a sample
h 1 of water is called biochemical oxygen demand
λ= ; λα of the water.
2m (KE) m
Water is supposed to be pure if it has BOD
λ1 m2 2 less than 5 ppm whereas BOD more than
= =
λ2 m1 1 17 ppm results in highly polluted water.
61. 1) cyclohexylamine Thus, water having BOD less than 5 ppm is
Solution:- rich in dissolved oxygen.
Basic nature order 69. 4)
Solution :
It is a symmetrical alkane. It can be prepared
from single alkyl halide.
70. 4) CO, graphite
62. 3) 3 – hexyne Solution :
Solution:- C ≡ O, sp graphite, sp2
CH3 – CH2 – CBr3 + 6Ag + Br3C – CH2 – CH3
O = C = O, sp Fullerene, sp2
 ∆
-6AgBr
→ O = C = C = C = O, sp
3 - Hexyne 71. 2) Boiling point : Y < X < Z
63. 2) 2 – bromo – 2 – methyl butane Solution :
Solution:- V.P↓ with increase of i value
Colligative property ∝ i
i increases ⇒ π ↑, ∆Tb ↑ ⇒ B.P↑,
⇒ ∆Tf ↑ ⇒ F.P↓
148
72. 2) α - halo butanoic acid 80. 4) Thermosetting polymer
Solution : Solution :
rate of decarboxylation ∝ stability of carbanion Terylene is a fibre and not a thermosetting
73. 3) Emulsion = solid in liquid polymer because on heating they melt and do
Solution : not show plastic property.
Emulsion is liquid in liquid. 81. 2) A-S, B-R, C-P, D-Q
74. 4) Al Solution :
Solution : A) D2O → Moderator in nuclear reactor
C & CO can’t be used in the reduction of B) Lime → Removes the temporary hardness
Al2O3, Since Al is more electro positive. of water
75. 3) 33.6 L C) Zeolite → Removes the permanent
Solution : hardness of water
Electrolysis of dil H2SO4 (2F electricity) D) H2O2 → Used as antiseptic
Cathode : 2H+ + 2e- → H2 ⇒ 22.4 L (STP)
82. 1) ∆H < 0 and ∆S > 0
1
Anode: 2 OH- → O2+ H2O + 2e- ⇒11.2L(STP) Solution :
2
∆H < 0 and ∆S > 0
Total volume = 22.4 + 11.2 = 33.6L
83. 2) b, c and d
76. 4) A2B5
Solution :
Solution :
Temperature is state function but intensive
1
Number of atoms(A) per unit cell = 8 x = 1 property
8
84. 3) H2S
Number of atoms (B) per unit cell
Solution :
1 5
= (6 – 1) x =
2 2
(One atom (B) is missing)

85. 4) H2S > H2Se > H2O


Thus, Formula is A1B5/2 = A2B5 Solution :
1
77. 4) PCl5(g) ⇌ PCl3(g) + Cl2(g) Volatility ∝
BP
Solution : 86. 3) 1.77 x 10 -5
Addition of inert gas at constant pressure Solution :
shifts the equilibrium towards the more
ΛmC ΛmC
number of gaseous substances. Degree of dissociation (α) = (or)
78. 3) Basicity of all oxoacids of phosphorous is 3 Λmo Λm∞
Solution : For monobasic acetic acid,
Oxoacids of phosphorous have different molar conductance = equivalent conductance
basicity based on the number of –OH groups 7.36
∴α = =1.88 x10−2
attached to phosphorous in the respective 390.7
oxoacid. For weak acid solution
79. 2) A, B, C and D Dissociation constant (Ka) = Cα2
Solution : = 0.05 (1.88 x 10-2)2
All the given statements are correct. Ka = 1.77 x 10-5
149
87. 4) The unusual shortness and strength of the 93. 3) 69.3 s
bond may be explained by a Pπ - dπ interaction Solution :
between the atoms of boron and fluorine. Rate = K [Reactant]1
Solution :
Due to the absence of d-orbital in boron and Rate 10−3
k= = = 10-2 s-1
fluorine does not makes Pπ - dπ bond. [Reactant] 0.1
88. 2) CH2(COOH)2 0.693 0.693
Solution : t1/2 == = 69.3 s
k 10−2
CH ≡ CH  CH 3 MgBr
→ CH ≡ C-MgBr
+
94. 1) A, B, C, D, E
→ CH ≡ C-COOH 
CO 2 / H 3O HgSO 4 / H 2SO 4
→ Solution :
OH
|
Tautomerisation
⇀ 1
↽ • Bond length ∝
CH = CH − COOH Bond order
OHC-CH2-COOH  → HOOC-CH2-COOH
Ag 2 O
∆ • If bond order is 0, the molecule or ion does
(Malonic acid) not exist.
89. 4) 5.27
• Boiling Point :
Solution:- H
CH3COOH and Ba(CH3COO)2 is an acidic OH O O
buffer. N
NV
pH = pka+ log S S > O
N a Va
NSVS = no. of gram equivalents of salt = 0.8
NO2
NaVa = no. of gram equivalents of acid = 0.25
(Inter molecular (Intra molecular
0.8 H-bond) H-bond)
pH = 4.761 + log = 5.27
0.25
• Dipole moment :
90. 3) 3 x 10-23g, 3 x 10-23 cm3
Solution :
6.023 x 1023 molecules = 18 g
18
1 molecule = x 10-23 = 3 x 10-23 g [This type is an special case where
6.023
µtans > µcis]
Since d = 1g / cc, Mass = volume
91. 3) A-R, B-S, C-Q, D-P 95. 2)
Solution :
A) Sign of cathode → +ve Solution :
B) Sign of anode → –ve More no of α - Hydrogen, more hyper
C) Electrons flow from → –ve electrode to conjugation, more stable
+ve electrode
D) Current flow from → +ve electrode to –ve 1) 2)
electrode
92. 3) 200, 193 5α - H 7α - H
Solution :
3) 4)
V–C bond strength ∝ electron density at metal.
Hence bond strength of [V(CO)6]- > [V(CO)6]
∴ Bond length of V–C is [V(CO)6]- < [V(CO)6] 5α - H 1α - H
150
96. 3) 2/5 The carbonyl compound must contain an α-
Solution : hydrogen atom, for it to undergo aldol
+ condensation.
xK2Cr2O7 + 6X 
H
→ 2Cr+3 + product
nf = 6 n f = x 99. 1)
6
=3⇒x=2 O OH
x
+
Solution :
2KMnO4 + 5X H
→ Mn+2 + product

nf = 5 nf = 2 
NH 3
→  →
2
⇒ O OH
5

KOH /Br2


97. 2) O NH2
100. 2) Analgesic
Solution : Solution:-
E2 (Anti elimination) Aspirin is analgesic and antipyratic
98. 4) b, c and e only
Solution :

BOTANY
101. 2) Zeatin 126. 1) Dominant gene
102. 4) C2H4, ABA 127. 3) Mechanical support
103. 2) Birds 128. 1) Phellogen
104. 4) Carrot, Radish 129. 3) Gaseous exchange
105. 2) Chloroplast 130. 2) Leptotene
106. 3) Acrocentric 131. 3) Conidia
107. 3) Ribosomes 132. 4) Generative cell – Produce male gametes
108. 2) Transient and unstable 133. 4) D – Microspore mother cell, E – Tapetum
109. 3) A – II, B – I, C – IV, D – III 134. 2) Monocot stem
+
110. 4) NADP 135. 2) Nematode
2-
111. 2) B4O7 136. 2) Polycistronic
112. 4) Manganese and Chlorine 137. 3) Pea and Ficus
113. 1) Flaccid 138. 3) Bryophytes – Vascular plants
114. 3) Primer extension 139. 2) 14
115. 3) Point mutation 140. 2) 1, 3 bis PGA → 3 PGA
116. 4) A, B, C and D 141. 4) Statement – I is incorrect, statement – II is
117. 2) Bioinformatics correct
118. 1) Identification of names of species
119. 1) the stroma
142. 3)
120. 1) Sonalika and kalyan sona
121. 1) ssRNA
122. 4) All of the above 143. 2) A – III, B – IV, C – I, D – II
123. 1) Bryophytes 144. 4) a, b, c and d
124. 4) Wolffia – Smallest fern
125. 1) Co-dominance
151
145. 4) Gene producing ADA is isolated from bone 147. 4) Y chromosome has most genes (2968) and
marrow cells and introduced into the cells chromosome I has the fewest (231)
of early embryonic stage 148. 2) Fabaceae
146. 4) Active repressor 149. 1) Gene therapy
150. c) c, d, e only

ZOOLOGY
151. 4) 3 177. 2) A, B and D only
152. 3) Ctenophora and Echinodermata 178. 1) Both Statements I and II are correct
153. 4) Aptenodytes, Psittacula, Struthio 179. 1) Both A and R are true and R is the correct
154. 2) Both A and R are true and R is the not explanation of A
correct explanation of A 180. 3) A, B and D are correct
155. 2) Trygon 181. 2) The first cellular form of life did not
156. 1) A - iii, B - iv, C - i, D - ii possibly originate till about 2000 billion
157. 3) Statement I is incorrect, statement II is years ago
correct 182. 3) Ernst Heckel
158. 4) either columnar or cuboidal cells 183. 4) individuals mate selectively
159. 3) entry of food into larynx 184. 4) The malarial parasite reproduce sexually in
160. 1) Total volume of air a person can expire liver cells
after a normal inspiration is inspiratory 185. 4) Ig - A
capacity 186. 1) preparation containing antibodies to the
161. 2) A - iii, B - i, C - ii, D - iv toxin
162. 4) Both statements I and II are incorrect 187. 4) Bone marrow, lymph nodes are secondary
163. 4) A - iii, B - i, C - ii and primary lymphoid organs respectively
164. 1) DCT 188. 3) All bulls can gives rise to superior progeny
165. 4) Angiotensin - II 189. 3) Mortality < Natality
166. 2) bone  K−N
190. 3) dN/dt = rN  
167. 1) Flat bone on the ventral midline of thorax -
 K 
sternum 191. 2) Ophrys
168. 3) i, iii and iv only correct
192. 1) primary production
169. 3) Photopic vision and is the function of cones
193. 2) Invertebrates - 359, vertebrates - 338, plants
170. 4) Thyrocalcitonin of thyroid gland regulates
- 87
the blood Na+ levels 194. 4) both 1 and 2
171. 2) A - Adenohypophpysis, B - Neurohypophysis,
195. 1) The development of the fertile top soil takes
C - Pars distalis centuries
172. 1) Both Statements I and II are correct
196. 4) all the above
197. 3) Automobiles
173. 3)
198. 2) Recycled modified plastic
199. 4) endometrium→myometrium →perimetrium
174. 3) male germ cells and sertoli cells
200. 3) A - 33, B - 67
175. 4) Tubectomy
176. 2) zygote or early embryo upto 8 cell stage
152
SPP – 5 (Answer Key & Solutions)

PHYSICS
1. 2) 200 N mR 2
Solution : L = m(ωR) x 3R - ω
2
Given x = t3 – 2t2 – 10 t
5mR 2 ω
dx = in Cω direction
v= = 3t2 – 4t – 10 2
dt
6. 2) a – e, b – h, c – f, d - g
dv
a= = 6t – 4 Solution :
dt Solution not available
At t = 4 seconds 7. 3) 3.4 eV
a = 6 x 4 – 4 = 20 m/s2 Solution :
Fnet = ma = 10 x 20 The energy of the electron in the hydrogen
Fnet = 200 N −13.6
2. 3) 27o atom in the nth state is E n = 2 ev
n
Solution :
where ionization potential is 13.6 eV given,
u 2 sin 2θ then energy required to remove an electron
Horizontal range R =
g +13.6
Substituting the given values, we get from n = 2 is E 2 = eV = 3.4 eV
4
82 x 82 x sin 2θ 8. 4) 5 mH
560 =
9.8 Solution :
560 x 9.8 5488 φ = Li
⇒ sin 2θ = =
82 x 82 6724 dφ di
− =−L
⇒ sin 2θ = 0.82 ⇒ 2θ = sin-1(0.82) dt dt
⇒ 2θ = 55.1 ⇒ θ = 27o di
ε = −L
3. 3) L2A dt
Solution : ε 5
L= = = 5 x 10-3 = 5 mH
Magnetic moment = (current) (Area) di −(2 − 3)
⇒ (Magnetic moment) = AL2 −
dt 1x10 −3
4. 1) 4 ms-1
2
Solution : 9. 2)
S 5
Velocity of boat = V = Solution :
t If ball falls from height h1 and bounces back
Here, S = 12 m
t = time of fall of object from bridge h
up to height h2 then e = 2
2h 2 x 45 h1
= = = 3 sec
g 10
12
Therefore, V = = 4 ms-1
3
5
5. 1) mR 2 ω
2
Solution :
153
Similarly if the velocity of ball before and hc 1
after collision are v1 and v2 respectively then ⇒ = W0 + mv 2 --- (i)
400 x10−9 2
v
e= 2 hc 1
v1 and −9
= W0 + m(2v) 2 --- (ii)
250 x10 2
v2 h 1.8 9 3 On solving (i) and (ii)
So = 2 = = =
v1 h1 5 25 5 1 hc  1 1 
mv 2 =  −9
− −9 
--- (iii)
i.e., fractional loss in velocity 2 3  250 x10 400 x10 
v 3 2
= 1− 2 = 1− = From equation (i) and (iii)
v1 5 5 W0 = 2hc x 106 J
-1
10. 1) 2.5 Vm
15. 4)
Solution :

Solution :
The output of NOT gate with input A is = A
The output of NOT gate with input B is = B
The output of NAND gate with inputs
( A & B ) is = A + B
From the graph it is clear that at any point Y = A.B = A.B
between B and C, the electric field intensity is ∴ Y = A.B
same. Therefore the electric field at d = 5m is 16. 1) 1 A
equal to the slope of line BC. Solution :
dv (0 − 5) −5 The given circuit can be simplified as follows
E=− =− = − = 2.5
dx (6 − 4) 2
E = 2.5 Vm-1
11. 1) 0, 1
Solution :
Diode is in reverse bias so reading of A1 is on further solving equivalent resistance
zero. R = 15Ω
V 12 Hence, current from the battery
For A2 i = = = 1A
R eff 10 + 2 15
i= = 1 A
12. 1) Two α-particles and one β-particle 15
Solution : µε
A −A' 17. 2)
By using n α = and nβ = 2nα - Z + Z′ µ0ε0
4
13. 1) 4 : 1 Solution :
We know that the velocity of light in vacuum
Solution :
1
λ=
h
=
h v m 4
∴ 1= 2= c=
m1v1 m 2 v 2 v 2 m1 1 µ0ε0
14.
6
1) 2 hc x 10 J And the velocity of light in a medium
Solution : 1
v=
hc 1 µε
By using = W0 + mv2
λ 2
154
Also the refractive index 21. 4) 1 rad s-1 & 5 m
Velocity of light in vacuum Solution :
n=
Velocity of light in medium v = ω A2 − x 2
1 v1 = 3 m/s , x1 = 4 m
v µ0 ε0 µε v2 = 4m/s, x2 = 3m
= = =
c 1 µ0 ε0 3 = ω A 2 − 42 --- (i)
µε
18. 3) 1.75 cm 4 = ω A −3 2 2
--- (ii)
Solution : Solving (i) and (ii), we get
Given : Focal length of objective lens A = 5m and ω = 1 rad/s
f0 = 1.6 cm 22. 4) 2
Focal length of eye-piece fe = 2.5 cm Solution :
Distance between two lenses L = v0 + ue B
= 21.7 cm tan δ = V
BH
And final image distance ve = ∞
Now for eye-piece, from lens formula BV ' BV
tan δ ' = =
1 1 1 1 1 BH ' BH cos 45o
= + = +
f e u e v e (L − v 0 ) ∞ tan δ (1)
⇒ tan δ ' = = ∴ tan δ ' = 2
or v0 = L – fe = 21.7 – 2.5 = 19.2 1 ( 2)
Again for objective lens. 2
1 1 1 1 1 1 1) 0.1 s
= + = = + 23.
f0 u 0 v0 f0 u 0 L − fe Solution :
1 1 1 1
⇒ = − ⇒ u0 = 1.75 cm Tension in wire = kx = 50 x = 0.5 N
1.6 19.2 u 0 100
m
 3W  Mass per unit length of string µ =
 W1 +  l
3) 
4  −3
19. 10 x10 1
S
−2
= kgm-1
Solution : 50 x10 50
3L Speed of wave pulse on string,
Force at a height from its lower end
4 T 0.5
v= = = 25 ms −1 = 5 ms-1
3 µ  1 
= Weight suspended + weight of   of the  
4  50 
chain Time required by pulse to each other end.
 3W  l 50 x10 −2
= W1 +   t= = = 0.1 s
 4  v 5 ms −1
 3W  24. 4) 0.32 m
 W1 + 4  Solution :
Stress =  
 S  1 1 1 1 1 1
= + ⇒ = +
  f u v u v 0.4
20. 1) zero v
Solution : m = = 5 ⇒ v = -5u
u
Internal force do not change the velocity of the 1 1 1 1.6
centre of mass (or) speed of the centre of mass − = ∴u= = 0.32
is zero. u 5u 0.4 5
155
5 7 Total mechanical energy = mgh
25. 4) U = RT and γ =
2 5 KE 2
As, =
Solution : PE 1
The gas is diatomic of = 5 2 1
KE = mgh and PE = mgh
C 7 5 3 3
γ = P = and U = nCVT = RT
CV 5 2 Height from the ground at this instant,
26. 4) 0.21 A from positive to negative terminal h
h ' = and speed of particle at this instant,
Solution : 3
 2h  gh
v = 2g(h − h ') = 2g   = 2
 3  3
31. 2) 7.98 x 10-4 Cm-1
Solution :
Electric field intensity due to infinite line
λ
-5i2 – 10(i1 + i2) – 2i2 + 20 = 0 charge is E =
-10i1 – 17i2 + 20 = 0 2πε 0 r
-10 + 4i1 + 10(i1 + i2) = 0 Given, E = 7.182 x 108 NC-1
14i1 + 10i2 + 10 = 0 r = 2 cm = 2 x 10-2 m
10i1 + 17 i2 = 20 → x 10 λ=?
14i1 + 10i2 = 10 → 17 2 x 2πε0 rE
⇒ λ = 2πε0rE =
-138i1 = 30 2
30 1x 2 x10−2 x7.182 x108
i1 = − = -0.217 = = 7.98 x 10-4 Cm-1
138 2 x 9 x109
It is negative it means current flows from 32. 2) 500 K
positive to negative terminal. Solution :
27. 3) 2.45 ms-2 Q ∝ T4
Solution :
Q T4 E 
g h (R + h)2 ⇒ 1 = 14 ⇒ T24 =  2  T14
= Q 2 T2  E1 
g R2
4
gh 1 1  1000 
= T24 = x (1000) 4 =   ⇒ T2 = 500 K
g  h
2
16  2 
1 +  33. 4) 6 x 10-5 cm
 R
Solution :
28. 1) 2 T Distance of nth dark fringe from central fringe
Solution : (2n − 1) λD
l GM y=
T = 2π , g= 2 2 d
g R -3 3 λ x1
1 x 10 = x
29. 2) 200 J 2 0.9 x10−3
Solution : λ = 6 x 10-7 m ∴λ = 6 x 10-5 cm
W T
=1 − 2 ρr
Q1 T1 34. 1)
3ε0
h gh Solution :
30. 2) ,2
3 3 Charge enclosed in the sphere of radius r.
Solution :
156
4 F - µR = ma1
q = πr 3ρ
3 R = normal reaction = mg
4 3 F - µmg = ma1
πr ρ F = ma1 + µmg --- (1)
q 3 rρ
E= = = 2nd case :
4πε0 r 4πε0 r 3ε0
2 2

F2 – f = ma2
35. 3) 1.25 W
2F - µmg = ma2 --- (2)
Solution :
From the equations (1) and (2)
Given V = 50 sin (50 t)V
Maximum voltage 2(ma1 + µmg) = µmg + ma2
π µmg = ma2 – 2ma1
V0 = 50 V, I = 50 sin (50t + ) mA µg = a2 – 2a1
3
µ x 10 = 18 – 2 x 5 = 8
Maximum current, i0 = 50 mA = 50 x 10-3 A
i0 ⇒ µ = 0.8
Power dissipated , P = b 9

2
V0  38. 4)
 8
 2
Solution :
(50 x 50 x10−3 ) 2500x10 −3 If the source of sound moving towards the
= = = 1.25 W
2 2 stationary observer, the frequency heard by the
m1m 2 2 observer.
36. 2) l
m1 + m 2  v 
f '= f  
Solution :  v − vs 
I = m1r12 + m2r22 where, v = speed of sound
r1 r2 vs = speed of sound source
 v 
m1 m2 therefore f1 = f   --- (i)
O  v − vs ' 
l  v 
therefore f 2 = f   --- (ii)
 v − vs '' 
axis
m2 l m1l From equation (i) and (ii)
Here r1 = and r2 =
m1 + m 2 m1 + m 2 f1 v − vs '' 340 − 34 34(10 − 1)
2 2 = = =
 m2   m1  f 2 v − vs ' 340 − 68 34(10 − 2)
⇒ I = m1  l  + m2  l
 m1 + m 2   m1 + m 2  f1 9
=
m m l2 f2 8
∴ I= 1 2 3t
(m1 + m 2 ) 39. 2)
37. 2) 0.8 2C
Solution : Solution :
Here m = 10 kg, a1 = 5 ms-2, F1 = F 3
Distance = thickness = t, µ =
a2 = 18 ms-2, F2 = 2F, µ = ? 2
Net force on the body = F,= f t µt
Time = =
Where F = applied force C C
f = frictional force µ
From Newton’s second law F – f = ma 3t
Therefore time =
1st case 2C
F1 – f = ma1
157
 π   π   3π 1  15 R
40. 1)  −  :   :  −  41. 1)
 2 2  4 2 4
Solution : Solution :
Case 1 : Kinetic energy of a system particles can be
µ i µ πi separated into kinetic energy of motion of the
BA = 0 . ⊗ ; BB = 0 . ⊙ centre of mass (translation) and kinetic energy
4π r 4π r
of rotational motion about the centre of mass
µ0 i
BC = . ⊙ of the system of particle (K′)
4π r
mv 2
So net magnetic field at the centre of case 1 K = K′ +
B1 = BB – BC - BA 2
µ πi This is incase of rolling motion
⇒ B1 = 0 . ⊙ --- (i) 1 1
4π r K = Iω2 + mv 2 --- (1)
2 2
In equation (1) substitute I = Mk2
(k = the radius of gyration of the body)
v = Rω. we get
1 Mk 2 v 2 1
K= + mv 2
2 R2 2
Case 2 :
1  k2 
As we discussed before magnetic field at the mgh = mv 2  1 + 2  --- (2)
centre O in this case 2  R 
µ πi In equation (2) substitute
B2 = 0 . ⊗ --- (ii)
4π r k2 1
v = 5gR and 2 =
R 2
1  1
m(5gR)  1 +  = mgh
2  2
1 3 15R
m(5gR)   = mgh ∴ =h
Case 3 : 2 2 4
BA = 0 42. 2) 442.5 nm
π Solution :
µ 0 (2π − 2 )i n1λ1 = n2λ2
BB = . ⊗ 3 x 590 = 4λ2
4π r
µ i µ 3πi 3 x 590
BC = 0 . ⊙ = 0 . ⊙ λ2 = = 442.5 nm
4π r 4π 2r 4
So net magnetic field at the centre of case 3 43. 1) 170 µA
µ i  3π  Solution :
B3 = 0 .  − 1  ⊗ --- (iii) E = BlV = 1 x 5 x 10-2 x 10-2
4π r  2  = 5 x 10-4 volt
e 5 x10−4
I= = = 1.66 x 10-4 ∴I = 170 µA
R (1.7 + 1.3)
44. 3) 133
Solution :
Here V1 = 10 V, VBE = 0, VCE = 0, VCC= 10 V.
From equation (i), (ii) and (iii) RB = 400 kΩ = 400 x 103 Ω,
158
3
RC = 3 kΩ = 3 x 10 Ω τmax = pE = q(2a)E
Now, Vi – VBE = RB IB = 1 x 10-6 x 2.0 x 10-2 x 1 x 105 = 2x 10-3 Nm
∴ 10 – 0 = (400 x 103) IB or W = pE(cos θ1 – cos θ2)
= (10-6 x 2 x 10-2) (105) (cos 0o – cos 180o)
10
IB = = 25 x 10−6 A = 4 x 10-3 J
400 x103
48. 3) 2.5 kms-1
Also, VCC–VCE = IC RC or 10 – 0 = IC x 3 x 103 Solution :
10 gm Mm  R e  1
or IC = = 3.33 x10 −3 A =
16
 = x (4) =
2
3 x10 3 x
g e M e  R m  81 81
I C 3.33 x10−3 16
β= = = 133 gm = ge
I B 25 x 10−6 81
45. 1) 61.7 ∴ ve = 2g e R e = 2 x 9.8 x (6400 x1000)
Solution :
≈ 11.2 kms-1
Heat extracted out of system
16 1
= (0.2) (4200) (25) = 21000 J v m = 2g m R m = 2 x ge x R e
81 4
Q 21000
So amount of ice melt = = kg 2 2
L 3.4 x105 = 2g e R e = x 11 ≈ 2.5 kms-1
9 9
210
= g = 61.76 gm 49. 4) 15 µF
3.4
Solution :
C1V1 + C 2 V2
46. 3) C1 + C 2
10 x 50 + C 2 x 0
20 = = 200 + 20 C2 = 500
Solution : 10 + C2
During isothermal process
C2 = 15 µF
1
A → B, P ∝ 50. 2) 6.28 T
V Solution :
During isobaric process (B →C), Here V = (10 x 0.5 x 0.2)cm3
P = constant = 1 cm3= 10-6m3
and during isochoric process (C →A), H = 0.5 x 104 Am-1, M= 5Am2, B = ?
V = constant M 5
Hence the correct P – V diagram of the I = = −6 = 5 x 106 Am
V 10
complete process is (3) From B = µ0(I + H)
47. 4) 2 x 10-3 N –m, 4 x 10-3 J B = 4π x 10-7 (5 x 106 + 0.5 x 104)
Solution : = 6.28T
q = ± 1 x 10-6 C
2a = 2.0 cm = 2.0 x 10-2 m
E = 1 x 105 NC-1, τmax = ?
W = ?, θ1 = 0o, θ2 = 180o
159
CHEMISTRY
51. 2) i only 57. 3) 1,1-Dibromo-2-chloro-5-methylcyclohexane
Solution : Solution :
In set (ii) and (iii) the order are wrong. In case of substituted cyclic compounds, if
52. 4) π - electrons lowest sum is same for two different
Solution : numbering, then give lowest number for more
substituted carbon and write IUPAC name
Cl
Br
2
1 Br
5

1,1-Dibromo-2-chloro-5-methylcyclohexane
π electrons are perpendicular to the plane. 58. 2) Statement – I is true and statement – II is
53. 4) i, ii and iii only false
Solution : Solution :
Except PCl5, other molecules are hypovalent. o-nitro phenol is purified by steam distillation
54. 2) -257.033 kJ but o-nitro phenol sparingly soluble in water
Solution : due to intra molecular hydrogen bond
2) I > III > II
∆So = ∑ So(Pr oducts ) − ∑ S(reactants)
o 59.
Solution :
 1  Structure with more number of covalent bonds
= SoCO2  − SoCO + SoCO2 
 2  and without charge separation is most stable.
= 213.8 – [197.9 + ½ (205)] Among (II) and (III), structure II is less stable
= -86.6 JK due to less number of covalent bonds and
carbon with an incomplete octet.
∆Go = ∆Ho - T∆So
= -282.84 – 298 x (-86.6 x 10-3) 60. 1)
= -282.84 + 25.87 Solution :
= -257.033 Kj Out of the given trans alkene is more stable so,
55. 2) HBr formation from H2 and Br2 trans 2-butene on hydrogenation liberates less
Solution : amount of heat.
Reduction Stability of alkene ∝ No. of hyperconjugative
structures
61. 2) Anti > Gauche > Partial eclipsed > Fully
eclipsed
Solution :
Other two cases, no redox reaction.
56. 4) Both (2) and (3)
Solution :
Statement (1) is wrong.
The actual dihedral angle in gas phase is
111.5° and in solid phase is 90.2°. The The anticonformation is most stable in which
difference due to intermolecular forces. two CH3 groups are far apart as possible and
minimum repulsion between two groups occurs.
160
In fully eclipsed conformation two CH3 groups 67. 1) a-iii, b-ii, c-i, d-iv
are so close that the steric strain is maximum Solution :
Hence this conformation is most unstable. Concentration of Ag ore → leaching with NaCN
∴Anti>Gauche>Partial eclipsed > Fully eclipsed Blast furnace → Pig iron
62. 4) Ammonical AgNO3
Blister copper → 98% Cu
Solution :
Only 1-alkynes give white ppt. with Froth floatation method → CuFeS2
ammonical AgNO3 (Tollen’s reagent). 68. 3) 6, 6
1-Butyne gives white ppt with ammonical Solution :
AgNO3, but 1-Butene does not.
63. 4) 106
Solution :
ZxM
d=
NA a3
4x M
7.62 = No. of P-O-P No. of P-O-P
6 x10 (0.4518 x10−7 cm)3
23

bonds = 6 bonds = 6
0.09 x10 −21 x 7.62 x 6 x10 23 3) +4
=M 69.
4 Solution :
106 = M Highest Mn fluoride is MnF4.In this
o
64. 2) -1 C compound, oxidation state of Mn = +4
Solution : 70. 4) A, B
∆Tf = i Kfm Solution :
i = 1 + (n – 1) α Sucralose is stable at coocking temperature
= 1 + (3 – 1) 50/100 = 1 + (2 x 0.5) = 2 cetyl trimethyl ammonium bromide is a
∆Tf = 2 x 0.5 x 1 cationic detergent.
Tfo − Tfs = 1 ; Tbs = Tfo − 1 71. 3) SN2 and SN2
0 – 1 ⇒ -1oC Solution :
65. 4) MF2 > MCl2 (solubility of fluorides and Both the reactions are halogen exchange
chlorides of alkaline earth metals) follows SN2 mechanism
Solution : 72. 4) NaHCO3
In alkaline earth metals, fluorides are Solution :
relatively less soluble than the chlorides owing Phenols and alcohols can’t be distinguished by
to their high lattice energies. NaHCO3
66. 2) rate = K[O3]2 [O2]-1 73. 4) A < C < B
Solution :
Solution :
Slow step is rate deterring step
N
rate = k′[O3] [O] (k′ - rate constant)

Keq =
[O 2 ][O ] (K = eq. constant) < <
N N N
[ O3 ]
eq

H H H
K eq [ O3 ]
[O] = (A) (C) (B)
[O 2 ] lp of e-s lp of e-s Aliphatic
k [ O3 ] K eq [ O3 ]
'
on 'N' on 'N'
rate =
[O 2 ] is involved is not involved
in resonance in resonance
rate = k[O3]2 [O2]-1 [k′ Keq = K]
161
74. 3) oils and fats 83. 1) 4π bonds and the remaining 4 electron pairs
Solution : form a tetrahedron
Oils and fats give positive test for acrolein Solution :
75. 3) I > III > II O
Solution : Xe
Intermolecular forces of attractions : O O
Fibres > Thermoplastics > Elastomers O
Nylon-6,6 > Polystyrene > Neoprene Out of 8 valency electrons of Xe 4 bond pairs
76. 2) Clemmensen reduction can form a tetrahedron and the remaining 4
Solution : electron can involve in Pi bond formation
We cannot use Wolf Kishner reduction 84. 4) TiF6 and Cu2Cl2
because in Wolf Kishner reduction due to Solution :
basic medium elimination takes place. TiF62− , Ti + 6F = -2
77. 2) C6H5CONH2 Ti + 6 (-1) = -2
Solution : Ti (Z = 22) ⇒ (Ar) 4s23d2
O
|| Ti+4 = [Ar] 4so3do
C6 H 5 − C − NH 2 + HNO 2 → C6 H 5 COOH + N 2 + H 2 O Cu2Cl2 = 2Cu+, 2Cl-
78. 2) PhCH2OH, HCOOH Cu (Z = 29) = [Ar] 4s13d10
Solution: Cu+ = [Ar] 4so3d10
PhCHO + HCHO →i)OH −
PhCH2OH, TiF6−4 and Cu2Cl2 do not have unpaired
ii) H O+ 3

HCOOH electrons so they are colour less species


85. 2) An oxidizing agent
(Cannizaro reaction)
79. 1) HF > HCl > HBr > HI…. acidic strength Solution :
Transition metal with its highest oxidation
Solution :
state acts as oxidizing agent
Correct order of acid strength
Oxidizing power ∝ oxidation state
HF < HCl < HBr < HI
86. 2) Fe2O3
80. 4) NO2
Solution :
Solution : 127.4 gr VO will be 479.1 gr of Fe2O3
Photochemical smog is caused by NO2 2 gr will require 7.5 gr Fe2O3
81. 2) -121 kJ But, only 5.75g Fe2O3 is given
Solution : So, Fe2O3 is limiting reagent.
C6H10 + H2 → C6H12, ∆H = ? 5R
1 87. 2)
Given H2 + O2 → H2O ∆H = -241 kJ --- (1) 9
2 Solution :
17 nh
C6H10 + O2 → 6CO2 + 5H2O Angular momentum =
2 2π
∆H = -3800kJ ---(2) n x 6.6 x10−34
C6H12 + 9O2 → 6CO2 + 6H2 3.1652 x 10-34 =

∆H = -3920 kJ ---(3) n=3
82. 2) It has sp3 hybridization of S and tetrahedral  1 1  2
shape ν=R − Z
Solution :  n12 n 22 
Sulphur in SF4 undergoes sp3d hybridisation 1 1 5R
so option (2) is incorrect ν = R  2 − 2  x 22 =
2 3  9
162
88. 2) acidic, basic 91. 2) -5.705 kJ, Forward
Solution : Solution :
( P ) = 100 =10
2
NO 2
Reaction quotient =
( P ) 10 N 2 O4

∆G o
Reaction= 2∆ f Go
− 2∆ f G (oN 2O4 )
( NO 2 )

= 2 x 50 - 100 = 0
We know that, ∆G = ∆Go – 2.303 RT log QP
∆G = 0 – 2.303 RT log QP
= -2.303 x 8.314 x 298 log 10
89. 3) = -5705.8J = -5.705 kJ
∆G = -ve, forward direction feasible
92. 2) 1.24 V
Solution : Solution :
Boyle’s law PV = K ∆G o
P K Theoretical efficiency = x 100
= ------- (1) ∆H o
V V2 +∆G o
PV2 = KV ------- (2) 84 = x 100
−285
log P = log K – log V ------- (3)
−285 x 84
P 1 ∆Go = = -239.4 kJ
= K. 2 100
V V
Y = mx −∆G o − ( −239.4 )
E ocell = = x 1000 = 1.24 V
nF 2 x 96500

93. 3)

Solution :

90. 2) 12.50
Solution :
PbBr2(s) ⇌ PbBr2(aq) → Pb2+ + 2Br-
80 x s
Pb2+ =
100 18
OH
2s x 80
Br- =
100 94. 3)
Ionization of PbBr2(s) = 80% OH
Ksp = [Pb2+], [Br-]2 Solution :
2
 S x 80   2S x 80 
8 x 10-5 =   
 100   100 
S = 0.034 mol/L mol wt = 367 g/mol
S = 0.034 x 367 g/L
S = 12.48 g/L
S ≃ 12.5 g/L
163
Sucrose is dextrarotatory but after hydrolysis
gives dextrorotatary glucose laevorotatory
fructose, laevorotatory fructose is more, so the
95. 3) mixture is laevorotatory.
98. 4) 19.6 mg
Solution :
Solution : WK 2Cr2O7 E O2
COD = × × 106
E K 2Cr2O7 VH 2O
49 × 3.2 ×1000
W= = 19.6 × 10−3 g = 19.6 mg
8 ×10 6

99. 2) -0.4 ∆0 + P
Solution :
96. 2) A-iv, B-i, C-ii, D-v In [Co(H2O)3F3] ⇒ Co3+ ; [Ar] = 3d6 4s0
Solution : As H2O and F- both are weak field ligands,
A) Sheet silicates - (Si2O5)n2n- ∆0 < P
B) Ortho silicates - SiO44- Electronic configuration will be t2g4eg2
C) Cyclic silicates - (SiO3)n2n- CFSE = (-0.4 nt2g + 0.6 neg]∆0 + P
D) Three dimensional silicates - (SiO2)n
= [(-0.4 x 4) + (0.6 x 2)]∆0 = -0.4 ∆0 + P
97. 1) 1 : 1D-(+)-glucose; D-(-)-fructose
100. 3) 100
Solution : Solution :
On hydrolysis with dilute aqueous sulphuric
acid, sucrose gives a equimolar mixture of D- Coagulation value = no.of milli moles of electrotye
vol.of sol (in litre)
(+) glucose and D-(-)-fructore.
10 x 0.5
C12H22O11 + H2O  H 2SO 4
→ C6H12O6 + C6H12O6 = = 100
50 x10−3
Sucrose D-(+)glucose D(+) fructose
1:1

BOTANY
101. 3) Euglenoids 116. 3) cryIAc and cryIIAb
102. 1) Size of vascular bundles 117. 4) T.W. Engelmann
103. 2) Bicarpellary syncarpous ovary 118. 2) S phase
104. 4) CO2 + H2O 119. 4) It is aided by cytoplasmic streaming
105. 3) Cytokinin 120. 2) Glycogen
106. 1) Dioecy 121. 3) Mustard
107. 3) TTP 122. 3) 9 Peripheral triplets with no central
108. 3) Gel electrophoresis microtubules
109. 2) Haplontic life cycle 123. 3) Pseudomonas
110. 3) Complex tissues are a part of ground tissue 124. 4) code is overlapping
system 125. 1) Adenosine deaminase
111. 4) Vacuoles 126. 2) Karanrai – Brassica
112. 2) I IV III II 127. 1) Propionibacterium sharmanii
113. 3) Poales – Class 128. 1) Methanogens are anerobic microbes
2+ 129. 2) Cycas
114. 1) Zn
115. 2) Trichoderma polysporum
164
130. 2) Enzyme induces the substrate to alter its 141. 2) Golgi complex
shape for fitting more tightly 142. 3) Halophiles are eubacteria that are salt
131. 3) Anaphase I resistants
132. 4) Glutamic acid 143. 1) Rhizome – Ginger
133. 1) Both the statements are correct 144. 3) Endodermis
134. 2) iii iv ii i 145. 2) Rhodospirillum
135. 3) 6 and 4 146. 1) A – i, B – iii, C – iv, D – ii
136. 3) sporopollenin in exine 147. 4) Ori
137. 4) All of the above 148. 3) He studied 14 pairs of contrasting
138. 1) transcriptional level characters in pea plant
139. 4) Fungi can fix atmospheric nitrogen 149. 3) DNA polymerase
140. 2) Panchanan Maheshwari 150. 1) A

ZOOLOGY
151. 3) ctenophores 178. 3) can’t produce ovum and can provide
152. 3) Blind sac body plan suitable environment for fertilization and
153. 2) Pterophyllum development
154. 3) Meandrina and Pennatula 179. 4) O group
155. 2) Radial symmetry 180. 3) grandmother and mother are carriers for
156. 3) A - oviduct, B - vagina, C - spermatheca, colour blindness and the child is a male
D - gonapophysis 181. 1) Thalassemia
157. 3) Anal styles 182. 4) A - iii, B - i, C - iv, D - ii
158. 3) gap junctions 183. 3) A - iv, B - i, C - ii, D - iii
159. 2) 20 184. 4) origin & structure
160. 4) O2 185. 2) B - lymphocytes
161. 4) Both statements I and II are incorrect 186. 3) Cell mediated immune response
162. 1) Electrocardiogram is a machine 187. 4) Animal dander
163. 2) A - iv, B - i, C - iii, D - ii, E - v 188. 2) Cell mediated immunity
164. 3) Statement I is incorrect, statement II is 189. 1) ELISA
correct 190. 1) Both A and R are true and R is the correct
165. 4) Henle’s loop explanation of A
166. 2) troponin 191. 1) Both Statements I and II are correct
167. 1) acromion 192. 1) Both Statements I and II are correct
168. 3) B, C and D 193. 3) Low reproductive capacity
169. 2) ridge present in otolith organ -2
194. 1) Kcal m
170. 3) A - i, B - iv, C - ii, D - iii 195. 1) Both Statements I and II are correct
171. 3) Glycogenolysis 196. 2) A - iv, B - iii, C - ii, D - i
172. 2) sella tursica of cranial bone 197. 4) 0.1 to 0.2
173. 4) oxytocin from matermal pituitary 198. 3) Statement I is incorrect, statement II is
174. 4) both 1 and 2 correct
175. 2) A, D 199. 4) Suspended solids in the water are nitrates,
176. 2) 72 hrs ammonia and phosphates
177. 2) before full term maturity of foetus 200. 4) Sodium

You might also like